Transcript
Page 1: Objectives - Internal Medicine Define diastolic and systolic heart

Objectives - Internal Medicine

Define diastolic and systolic heart failureHeart failure is present when cardiac output fails to meet the metabolic demands of the body or meets those demands only if the cardiac filling pressures are abnormally high.

Systolic Heart Failure:In systolic dysfunction, the affected ventricle has a diminished capacity to eject blood because of impaired myocardial contractility or pressure overload (i.e. excessive afterload). Loss of contractility may result from destruction of myocytes, abnormal myocyte function, or fibrosis. Pressure overload impairs ventricular ejection by significantly increasing the resistance to flow. In impaired contractility, the end systolic pressure-volume relationship (ESPVR) is shifted downward such that systolic emptying ceases at a higher-than-normal end-systolic volume. When normal pulmonary venous return is added to the increased end-systolic volume that has remained in the ventricle because of incomplete emptying, the diastolic chamber volume increases, resulting in a higher-than-normal end-diastolic volume and pressure. While that increases in preload induces a compensatory rise in stroke volume (via the Frank-Starling mechanism), impaired contractility and the reduced ejection fraction cause the end-systolic volume to remain elevated. During diastole, the persistently elevated LV pressure is transmitted to the left atrium (through the open mitral valve) and to the pulmonary veins and capillaries. An elevated pulmonary capillary pressure (>20mm Hg) results in the transudation of fluid onto the pulmonary intersititum and symptoms of pulmonary congestion.

Diastolic Heart Failure:Abnormalities of impaired diastolic function include either impaired early diastolic relaxation (an active, energy-dependent process), increasing stiffness of the ventricular wall ( a passive property), or both. Acute myocardial ischemia is an example of a condition that transiently inhibits energy delivery and diastolic relaxation. Conversely, left ventricular hypertrophy, fibrosis, or restrictive cardiomyopathy causes the LB walls to become chronically stiffened. In diastole, filling of the ventricle occurs at higher-than-normal pressures because of reduced compliance of the ventricle. Patients with diastolic dysfunction often present with signs of vascular congestion because the elevated diastolic pressure is transmitted retrograde to the pulmonary and systemic veins.

Discuss the underlying pathophysiology of heart failureHeart may not provide tissues w adequate blood for metabolic needsSystolic or diastolic function abnormalities → ↑ pulmonary or systemic pressures → organ congestion

- Systolic dysfunction: ventricle contracts poorly, and doesn’t empty fully → ↑ diastolic volume and pressure, ↓ EF

- Diastolic dysfunction: difficulty filling ventricle → ↓ end-diastolic volume, ↑ end-diastolic P (EF normal)- LV failure: LV dysfunction → ↓ CO and ↑ pulmonary vein P → extravasation from capillaries into

interstitial space and alveoli = ↑ work of breathing- RV failure: ↑ BP → extravasation into periphery = edema, liver highly affected (↑ conjugated and

unconjugated bilirubin, PT, ALP, AST, ALT)

Page 2: Objectives - Internal Medicine Define diastolic and systolic heart

List the main risk factors for heart failure.

The 5 most common causes of CHF are: CAD (and other cariac risk factors), HTN, idiopathic, Valvular heart disease (AS, AR, MR) and alcohol (which may cause dilated cardiomyopathy). Patients are often asymptomactic for long periods of time because of mild impairement or because compensatory mechanisms are present (adrenergic nervous system, RAAS) to balance the dysfunction. Manifestations of HF occur when factors increase cardiac workload or tip the state of balance = decompensation. Think in terms of preload, afterload, cardiac output, contractility. Think of HEART FAILED (risk factors and precipitants of heart failure)

o Hypertension (increased afterload)o Environment/endocarditiso Anemia (cardiac output)o Rheumatic heart disease and other valve diseaseso Thyrotoxicosis (increased metabolic demand with insufficient CO)o Failure to take medicationso Arrhythmia (with tacchy, decreased diastolic filling time and increased heart O2 demand)o Infection, Ischemia or Infectiono Lung problems (PE (hypoxemia because of decrease O2 supply to heart), pneumonia, COPD- emphysema is

associated with right-sided failure)o Endocrine (pheo, hyperaldosteronism = increased metabolic demand)o Dietary indiscretions (too much salt = increased preload = congestion)

Also:

age (over 65) gender (male) ethnicity (higher rate in African american) genetics, family history chronic alcohol abuse (thiamine insufficiency) = impaired contractility CAD (associated with smoking, obesity)

Demonstrate an appropriate physical examination for heart failure

Symptoms Signs (Physical Findings)LEFT-SIDED

Dyspnea, orthopnea, paroxysmal nocturnal dyspnea, fatigue, dulled mental status (from decreased

perfusion), decreased urine output in daytime (decreased

renal perfusion), nocturia

LEFT-SIDED Cachexia, diaphoresis (increased SNS activity),

cool extremities, sinus tachycardia, tachypnea (+/- Cheyne Stokes breathing), pulmonary rales, loud P2, S3 gallop +/- S4, mitral regurgitation, pulsus alternans (alternating strong and weak

contractions in peripheral pulse), dullness in base of lungs on percussion (pleural

Page 3: Objectives - Internal Medicine Define diastolic and systolic heart

effusion)RIGHT-SIDED

Peripheral edema (especially calves to feet), weight gain (from fluid), abdominal discomfort (liver becomes

engorged), anorexia and nausea (edema of GI tract)

Jugular venous distention, hepatomegaly, peripheral edema (ankles and presacral

regions), right-sided S3 or S4, tricuspid regurgitation, dullness in base of lungs on percussion (pleural

effusion)

Implement emergency management for heart failure

Removal/correction of precipitating causeo Infection/arrhythmia/valvular disease/MI/PE…

Prevention of deterioration of cardiac functiono ACE inhibitorso ß-blockerso Digitalis glycosides

Control of congestive HF stateo Diuretic (+ weigh daily and adjust dose)o Liquid and salt restriction

Tx. of acute pulmonary edema LMNOP

o Lasixo Morphineo Nitrateso Oxygeno Position (sit upright)

Write a series of appropriate orders to treat heart failure

Acute CHF (presenting with pulmonary edema)1. O2 and intubation2. Nitroglycerin 0.4 mg sublingiual (may be repeated every 1-5 min)3. Potent IV diuretic. E.g. Furosemide 40-80 mg IV

a. Monitor electrolytes, especially K+4. Morphine 2-5 mg IV PRN

** Dopamine 5-10 µ/kg/min should be started for hypotensive pt or pt in need of additional inotropic support

Long term tmt of CHF (according to ACC/AHA HF classification)

Stage Description Tmt

A No structural heart disease and no symptoms but risk factors: CAD, HTN, DM, cardio toxins, familial cardimyopathy

1) Lifestyle modification (diet, exercise, smoking cessation)

2) Treat hyperlipidemia (Lipitor, Crestor)3) ACE inhibitor for HTN (Ramipril, Enalapril)

Page 4: Objectives - Internal Medicine Define diastolic and systolic heart

B Abnormal LV systolic function, MI, valvular heart disease but no HF symptoms

1) Lifestyle modifications2) ß-blockers (Bisoprolol, Carveditol, Metoprolol)

C Structural heart disease and HF symptoms

1) Lifestyle modifications2) ACE inhibitors3) ß-blockers4) Diuretics (HCTZ, Bumetamide, Furosemide)5) Digoxin

D Refractory HF symptoms to maximal medical management

1) Therapy under A, B, and C2) Mechanical assisted device3) Heart transplantation4) Continuous IV inotropic infusion5) Hospice care in selected pt

Identify the pharmacological agents likely to increase survival with heart failureLong Term Management of Heart FailurePharmacological Therapy:

Vasodilators:o ACEis: standard of care – slow progression and improve survival

All symptomatic patients functional class II-IV (grade A) All asymptomatic patients with LVEF <40% (grade A) Post-MI Target dose as used in mortality trials, or maximum tolerated dose

o Angiotensin II receptor blockers Second line to ACEi if not tolerated (grade B), or as adjunct to ACEi if β-blockers not

tolerated (grade A)o Hydralazine and nitrates

Second line to ACEi, decrease in mortality not as great as with ACE-i

β-blockers: standard of care – slow progression and improve survival should be used cautiously, titrate slowly because may actually worsen CHF class I-III with with LVEF <40% (grade A) stable class IV patients (grade A)

Aldosterone Antagonists: mortality benefit in severe CHF Spironolactone for class IIIb and IV CHF already on ACEi and loop diuretic (grade A) Hyperkalemia!! Monitor K+ after initiation and avoid if Cr>220 or K>5.2mmol/L

Diuretics: symptom control management for fluid overload Furosemide for potent dieresis Metolazone may be used with furosemide to increase dieresis

Page 5: Objectives - Internal Medicine Define diastolic and systolic heart

List the valvular heart diseases associated with syncope

Aortic stenosis: calcification of aortic cusps (resulting from congenital abnormality or rheumatic inflammation or age-related degenerative calcific)- Symptoms: asymp until <1cm

o Exertional dyspneao Angina pectoriso Exertional syncope may result from ↓ in arterial pressure caused by vasodilatation in exercising

muscles and inadequate vasoconstriction in nonexercising muscles in the face of a fixed cardiac output or from sudden ↓ in cardiac output by an arrhythmia

- If aortic stenosis and mitral stenosis coexist, ↓ in CO induced by MS lowers pressure gradient across aortic valve and masks many clinical findings produced by AS

Use a systematic approach to interpret an ECG, including determining rate, rhythm and axis.

- First thing required is to check the calibration = 1mm = 0.1 mV

RATE:

Page 6: Objectives - Internal Medicine Define diastolic and systolic heart

RHYTHM: a normal sinus rhythm must havea) HR between 60-100

Page 7: Objectives - Internal Medicine Define diastolic and systolic heart

b) P before every QRS and QRS after every Pc) Positive P wave in either leads 2, 3, avf (inferior leads)d) PR of less than 0.12 sec

AXIS:

Look at leads I and avF: if both are positive, axis must lie between -30 and +90 degrees If they are not both positive, look for the isoelectic lead Look for the most isoelectric lead The net QRS axis is perpendicular to the most isoelectric lead (ex: assume lead avl is the most isoelectric then

the net QRS axis is perpendicular to avl, either 60 degrees or – 120 degrees) Axis implies the net vector of depolarization of the ventricles LAD: causes include LVH, LBBB, left anterior hemiblock, inferior MI, elevated diaphragm, WPW RAD: causes include RVH, left posterior hemiblock, PE, COPD, septal defects, lateral MI, WPW

INTERVALS:- PR: normal is 0.12 – 0.2 sec- QRS: normal is approximately less than 0.12 sec- QTc: normal is approximately less than 0.44 sec

Apply ECG criteria for diagnosing chamber enlargement

ATRIAL HYPERTROPHYLEAD II LEAD V1

RA enlargement P wave taller than 2.5mm – best visualized in lead II

Page 8: Objectives - Internal Medicine Define diastolic and systolic heart

LA enlargement Greater than normal negative deflection (at least 1mm wide and 1 mm deep) – best visualized in V1

VENTRICULAR HYPERTROPHYRVH R>S in V1 + right axis deviation (lead I –ve, II positive)LVH S in V1 + R in V5 or V6 > 35mm + left axis deviation (lead I +ve, II –ve)

Interpret an ECG with atrial fibrillation (AF)

- atrial rate 350-600 discharges/minute (cannot see distinct p waves)- ventricular rate 140-160 bpm- ventricular (QRS) rhythm is irregularly irregular

Recognize the various forms of heart block on an ECG

AV CONDUCTION BLOCKS

1st degree AV block- ↑ PR interval- Found in otherwise healthy

aduls- No tmt required

2nd degree AV block- TYPE I

- Gradual ↑ PR interval preceeding QRS drop

- Due to failure of conduction of a P wave

- Frequently benign

- TYPE II

Page 9: Objectives - Internal Medicine Define diastolic and systolic heart

- Constant PR interval- Abrupt QRS drop- Block is usually distal to AV

node- ↑ risk of high grade 3rd

degree AV block

3rd degree AV block- Variable PR intervals- Narrow or wide QRS- Variable PP and PR intervals- No relationship between P

and QRS- Management – electrical

pacing

BUNDLE BRANCH BLOCKS

LBBB

- Wide QRS- Initial R wave absent in V1- Absent small Q wave in V6

RBBB

- Wide QRS- R’ in V1- Terminal deep S in V6

LAFB- Left axis deviation- Small Q in aVL and I- Small R in inferior leads (II, III, aVF)

LPFB- Right axis deviation- Small R in aVL and I- Small Q in inferior leads (II, III, aVF)

Bifascicular block- RBBB plus either LAFB (common) orLPFB (uncommon)- Features of RBBB plus frontal plane features of the fascicular block (axis deviation, etc.)

Interpret an ECG consistent with ischemiaOne of the most useful tools for diagnosing ischemia is obtaining an ECG during an ischemic episode. During myocardial ischemia, ST segment and T wave changes often appear. Acute ischemia usually results in transient horizontal or downsloping ST segment depressions and T wave flattening or inversions. Occasionally, ST segment elevations are seen, suggesting more severe transmural myocardial ischemia, and can also be obtained during the intense vasospasm of variant angina. In contrast to the ECG of a patient with an acute MI, the ST deviations seen in patients with stable angina quickly normalize with resolution of the patient’s symptoms. In fact, ECGs obtained

Page 10: Objectives - Internal Medicine Define diastolic and systolic heart

during periods free of ischemia are compeletely normal in approximately half of oatients with stable angina. In others, chronic “non-diagnostic” ST and T wave deviations may be present. Evidence of a previous MI (pathologic Q waves) on the ECG also points to the presence of underlying coronary disease.

Standard Exercise Testing:The test is continued until angina develops, signs of myocardial ischemia appear on ECG, a target HR is achieved (85% of max predicted HR) or the patient becomes too fatigued to continue. The test is considered positive if the patient’s typical chest discomfort is reproduced or if the ECG abnormalities consistent with ischemia develop (i.e. >1mm horizonatal or downsloping ST segment depressions). The stress test is considered markedly positive if one or more of the following signs of severe ischemic heart disease occur:

1. Ischemic ECG changes develop in first 3 minutes of exercise or persist 5 minutes after exercise has ceased.

2. The magnitude of the ST segment depressions is >2mm 3. Systolic blood pressure decreases during exercise (i.e. resulting from ischemic induced impairment of

contractile function)4. High-grade ventricular arrhythmias develop5. The patient cannot exercise for at least 2 minutes because of cardiopulmonary limitations

Recognize the ECG abnormalities associated with electrolyte abnormalities- Hyperkalemia: “tenting” of T waves, wider QRS with ↑ K concentration, P waves ↓ in amplitude and may

disappear, sinewave pattern → asystole if severe hyperK- Hypokalemia, IA antiarrhythmics (quinidine, procainamide, TCA), III (amiodarone, sotalol, ibutilide): QT

prolongation or prominent U waves, and ↑ susceptibility to Torsades - Hypothermia: prolonged repol, QRS “hump” at J-point = Osborn wave- Hypocalcemia: prolonged QT- Hypercalcemia: shortened QT- Digitalis: shortened QT, “scooping” of ST-T wave

Interpret an ECG consistent with pericarditis

o Characteristic ECG changes are 1) diffuse ST elevation initially that resolves to flattened and inverted T waves over days and 2) PR depression in several leads, 3) +/- T wave inversions

Page 11: Objectives - Internal Medicine Define diastolic and systolic heart

List the most common causes of AF

association with age (sharp increase in incidence after 70 yrs of age) idiopathic most frequent predisposing condition: hypertension underlying cardiac abnormality: valvular heart disease (especially rheumatic), heart failure and ischemic

heart disease Other precipitants: pericarditis, thyrotoxicosis, pulmonary emboli, pneumonia, acute alcohol ingestion,

post-op for 1/3 of patients who undergo cardiac surgery

Demonstrate the ability to perform an appropriate cardiac physical examination

General observation- Pale/diaphoretic/cyanotic/SOB/extremities (temperature, cap refill)/etc

Vital signs (TA, HR, RR, SatO2, Temp)

- Bed at 30°

Observation - JVP (while at the neck, auscultate and palpate carotids)- Scars

Page 12: Objectives - Internal Medicine Define diastolic and systolic heart

- Palpation- Heaves/thrills- Apex (location, diameter, force of impulse)…pt. in left lateral decubitus if difficult to find

Auscultation- Aortic, Pulmonary, Tricuspid and Mitral areas- Diaphragm and bell - B1B2 (B3B4 and murmur/regurg.)

o Emphasize mitral regurg by pt. in left lateral decubituso Emphasize aortic regurg by pt. sitting up and leaning forward

Periphery- Peripheral pulses- Pitting oedema

Resp exam must also be done (ex. Pulmonary oedema caused by CHF – crepitus in lung bases)Abdo exam may also reveal findings (ex. Hepatomegaly in right-sided CHF)

Discuss the various anti-arrhythmia medications available to treat AF

Tmt focuses on 3 aspects of the arrhythmia:1) Ventricular rate control = agents that ↑ conduction through AV node

a. ß-blockers or CCB (diltiazem or veramapil) b. Digitalis (Digoxin) less effective but may be useful if impaired ventricular contractile fonction

2) Rhythm control = drugs that modify impulse formation/propagation to prevent initiation of AFa. Pharmacologic cardioversion = Class IIIb. Maintenance of sinus rhythm = Class I, II, III, IVc. Electric cardioversiond. Catheter ablation

3) Prevention of thromboembolitic eventsa. For AF that persists > 48 hrb. Anticoagulation tmt ≥ 3 weeksc. Consider chronic warfarin anticoagulation (superior to ASA and clopidogrel)

ANTI-ARRHYTHMIA MEDICATIONS

Class Agent Mechanism of action Side effects

IA QuinidinePracainamideDisopyramide

- Moderate Na channel blockade- ↓ phase 0 upstroke- ↑ repolarization → ↓ conduction

- Torsade de pointe- Diarrhea- Lupus-like syndrome- Anti-cholinergic side effects (↓

memory, confusion, lightheadedness, constipation, blurred vision, dry mouth)

II PropanolMetoprolol

- ß-blocker- ↓ phase 4 depolarization

- Bronchospasm- Negative inotropy- Bradycardia- AV block- Impotence- Fatigue

III AmiodaroneSotalo

- Blocks K channel- ↑ phase 3 repolarization →↑ refractory

- Same as II +- Photosensitivity

Page 13: Objectives - Internal Medicine Define diastolic and systolic heart

period - Pulmonary toxicity- Hepatotoxicity- Thyroid disease- ↑ INR- Torsade pointe- Bradycardia- Heart block

IV VerapamilDiltiazem

- CCB- ↓ phase 4 spontaneous depolarization

→ ↓ AV node conduction

- Bradycardia- AV block- Hypotension

- Do not always maintain sinus rhythm during long term follow-up but ↓ # of AF episodes- Usually not used in asymptomatic pt because of the severity of their side effects (↓ efficacy, ↑ toxicity so

limited use)

** Class IC can also be used in pt with structurally normal heart (only)

IC PropafenoneFlecainideEncainide

- Marked Na channel block- Markedly slows phase 0 upstroke

- Exhacerbation of VT and heart failure- Negative inotropy- Bradycardia and Heart block

Prescribe appropriate medications for anticoagulation in a patient with AF.The decision to anticoagulate patients with a fib involves weighing the risks of an embolic event without therapy vs the risk of hemorrhage event on therapy. Improved methods of monitoring anticoagulation with the International Normalized Ratio (INR), and recent evidence of the efficacy and safety of low-dose warfarin (INR range 2.0 to 3.0) have clarified the role of anticoagulation in AF. Initiate therapy with the estimated daily maintenance dose (2-5 mg.). Elderly or debilitated patients often require low daily doses of warfarin (2-4 mg.). Patients are confused by alternating daily doses (e.g. 7.5 and 5.0 mg). Significant changes in INR can usually be achieved by small changes in dose (15% or less). 4-5 days are required after any dose change or any new diet or drug interaction to reach the new

antithrombotic steady state.

Demonstrate an appropriate approach in performing a history and physical examination for a patient with chest pain

Quick look test:- Patients with MI look pale and anxious- Pts with pericarditis, pneumothorax or PE involving pleural surface look apprehensive and breathe with

shallow/painful breaths - Pts with esophagitis or MSK problems look well

Airway and vitals:- BP: most pts with chest pain have normal BP

o Hypotension may be seen with MI, massive PE, aortic dissection → cardiac tamponade or tension pneumothorax

Page 14: Objectives - Internal Medicine Define diastolic and systolic heart

o Wide pulse pressure: suspect aortic insufficiency, which could be a complication of proximal aortic dissection

o Pulsus paradoxus: pericardial effusion with aortic dissection or pericarditis- HR:

o Tachy: severe chest pain, >100BPM could possibly mean a-fib, supraventricular tachys or ventricular tachy

o Brady: sinus or AV node ischemia, beta blockade or calcium channel blockade due to drugs- ECG analysis

Selective Hx and chart review:- How does pt describe pain? - Is pain same as usual angina- Pain worse with deep breathing or coughing? Pleuritic chest pain suggests pleuritis, pneumothoriax, rib

fracture, pericarditis, PE, pneumonia, costochondritis- Does pain radiate? - Associated nausea, vomiting, diaphoresis, lightheadedness? - Chest pain worse with swallowing? Suggests esophageal disorder or pericarditis

Selective physical:- Vitals- HEENT: white exudates in oral cavity or pharynx → thrush with possible concomitant esophageal

candidiasis- Resp:

o Asymmetrical expansion of chest → pneumothoraxo Deviation of trachea to one side → large pneumothorax on side opposite of deviationo Hyperresonance to percussion → on side of pneumothoraxo ↓ breath sounds → on side of pneumothoraxo Crackles → CHF secondary to acute MI, pneumoniao Consolidation → pulmonary infarction, pneumoniao Pleural rub → PE, pneumoniao Pleural effusion → PE, pneumonia, ruptured aortic dissectiono Chest wall: check for fractures, sensitive areas

- CVS:o Unequal carotid pulses → aortic dissectiono Unequal upper limb BP or ↓ femoral pulses → aortic dissectiono ↑ JVP → right ventricular failure secondary to MI or PE; tension pneumothoraxo Right ventricular heave → acute right ventricular failure secondary to PEo Left ventricular heave → CHFo Displaced apical impulse → away from side of pneumothoraxo Loud P2 → acute cor pulmonale, S3 → CHFo Mitral insufficiency murmur → proximal aortic dissectiono Pericardial rub (scratchy sounds that vary with position) → pericarditis

- Abdomen:o Guarding, rebound tenderness → perforated ulcero Epigastric tenderness → peptic ulcer diseaseo Generalized abdo pain → mesenteric infarction from aortic dissection

- CNS:o Hemiplegia → aortic dissection involving carotid artery

Distinguish between coronary versus non-coronary causes of chest pain

Page 15: Objectives - Internal Medicine Define diastolic and systolic heart

- history is essential: assess riwk factors, attempt to rule out potentially life threatening conditions (PE, tension pneumothorax, MI (HTN, hyperlipidemia, DM, smoking, genetics, race, age, gender), aortic dissection, etc.)- investigations: Troponin-t, CK-MB, EKG, CXR, CT, echo, CBC + diff

Angina/MI - retrosternal pain radiating to shoulder/jaw, dyspnea, diaphoresis, N/V, exertion or not, relieved by NTG

Pericarditis - sharp pain radiating to trapezius, aggravated by breathing and relieved by sitting forward

Aortic dissection

- sudden onset of pain, tearing, knife like, anterior or posterior, mid-scapular

Pneumothorax - unilateral, sharp, pleuritic, sudden onsetPE - pleuritic, sudden onset, tachypnea, tachycardia, hypoxemia…may also have calf pain,

erythema, swelling, also think of Virchow’s triad (alteration in blood flow, damage to endothelium, hypercoagulability)

Esophageal reflux

- retrosternal burning, acid taste in mouth, worse with eating (especially caffeine, chocolate, EtOH), better with antacids and PPI

MSK - localized pain, may radiate, tender to palpation (costochondritis, broken ribs, muscle tear)

- other causes of non-coronary pain include: pneumonia, pancreatitis, psychogenic, PUD

Describe the differences in the acute coronary syndromes: Unstable angina, non ST elevation MI and ST elevation MI

FEATURE UNSTABLE ANGINA NSTEMI STEMITypical symptoms Crescendo (increasing),

rest, or new onset severe angina

Prolonged crushing chest pain, more severe and wider radiation than usual angina

Serum biomarkers (i.e. Troponin T, CK)

None Yes Yes

ECG initial findings ST depression and/or T wave inversion

ST depression and/or T wave inversion

ST elevation and later on, Q waves (ST depression in reciprocal leads)

Describe the difference between stable angina and ACS

Stable angina ACSAnatomy Lumen narrowed by more than 70% by

atherosclerotic plaqueInappropriate vasoconstriction

Rupture of an unstable atherosclerotic plaque and platelet aggregation and thrombosisUnopposed vasoconstriction

Symptoms Aching/pressure chest pain that may radiate to neck, jaw and armLasts 5-15 minutesIs relieved by rest or nitroglycerinChest pain arises upon predicted level of exertionCharacter of each attack varies little with recurrent episodes

Aching/pressure chest pain that may radiate to neck, jaw and armLasts more than 15 minutesIs not relieved by rest or nitroglycerinChest pain arises with increasing frequency, severity or at rest

Page 16: Objectives - Internal Medicine Define diastolic and systolic heart

ECG No changes ST-segment elevations, Q waves, bundle branch block, T wave inversions Normal/unchanged does not rule out acute MI

Serum markers Negative Serial enzyme evaluation is needed for appropriate assessment

Positive CK-MB and TnTMay be negativeSerial enzyme evaluation is needed for appropriate assessment

Echocardiography Regional wall motion abnormalities may represent previous myocardial injury therefore a previous echo would be useful

Regional wall motion abnormalitiesRegional wall motion abnormalities may represent previous myocardial injury therefore a previous echo would be useful

Treatment NitroRest

ASALMWHClopidogrelß-blockernitrates oxygen pain control (morphine)ACE inhibitorStatinIf STEMI, reperfusion is required

Fibrinolytic Or

Primary PCI

Implement emergency treatment for acute coronary syndromes

Genertal (all pt) 1) General measuresa. O2b. Pain control (morphine)

2) Anti-ischemic medicationsa. ß-blockerb. Nitratec. +/- CCB

3) Additional therapya. ACE inhibitorsb. Statins

Antithrombic approach (UA and NSTEMI)

1) ASA2) Heparin (UFH or LMWH)3) Clopidogrel

** For high risk pt:4) GP IIb/IIIa inhibitors5) Cardiac cath

Reperfusion approach (STEMI)

1) ASA2) Heparin (UFH or LMWH)3) Clopidogrel

Page 17: Objectives - Internal Medicine Define diastolic and systolic heart

4) Firbinolytic drug or Primary PCI6) GP IIb/IIIa inhibitors

Recognize the complications of acute coronary syndrome(taken from Lilly)Complications:In unstable angina, the potential complications include death (5-10%) or progression to infarction (10-20%) over the ensuing days or weeks. Once infarction has transpired, especially STEMI, complications can result from the inflammatory, mechanical or electrical abnormalities induced by regions of necrosing myocardium. Early complications result from myocardial necrosis itself. Those that develop several days to weeks later reflect the inflammation and healing of necrotic tissue.

1. Recurrent Ischemia:-Postinfarction angina has been reported to occur in 20-30% of patients following an MI.

2. Arrythmias: -These occur frequently during acute MI and are a major source of mortality prior to hospital arrival. Mechanisms that contribute to arrhythmogensis after MI include:

a) Anatomic interruption of perfusion to structures of the conduction pathway.b) Accumulation of toxic metabolic products (e.g. cellular acidosis) and abnormal transcellular ion concentrations owing to membrane leaks.c) Autonomic stimulation (SNS and PSNS)d) Administration of potentially arrhythmogenic drugs (e.g. dopamine)

Ventricular Fibrillation: Largely responsible for episodes of sudden cardiac death during the course of an acute MI. Episodes of v fib that occur during the first 48 hours of MI are often related to transient electrical instability , and the long-term prognosis of survivors of such events is not affected. However, v fib occurring later than 48 hours after acute MI usually reflects severe LV dysfunction and is associated with high subsequent mortality rates.

Ventricular ectopic beats, ventricular tachycardia, and v fib during an acute MI arise from either reentrant circuits or enhanced automaticity of ventricular cells.

SVTs – sinus brady results from either excessive vagal stimulation or sinoatrial nodal ischemia usually in the setting of an inferior wall MI. Sinus tachy occurs frequently and may result from pain and anxiety, heart failure, drug administration (dopamine, etc), or intravascular volume depletion. Because sinus tachy increases myocardial oxygen demand and could exacerbate ischemia, treat its causes!!

Conduction blocks: result from ischemia or necrosis of conduction tracts or in case of atrioventricular blocks, may develop transiently as a result of increased vagal tone.

3. Myocardial Dysfunction:a) CHF:

-Acute cardiac ischemia results in impaired ventricular contractility (systolic dysfunction) and increased myocardial stiffness (diastolic dysfunction), both of which may lead to symptoms of HF.

b) Cardiogenic Shock:- Condition of severely decreased cardiac output and hypotension with inadequate perfusion of peripheral tissues that develops when more than 40% of the LV mass has infracted.

4. Right Ventricular Infarction:1/3 of patients with infarction of the LV inferior wall also develop necrosis of portions of the right ventricle, because the same coronary artery (usually RCA) perfuses both regions.

5. Mechanical Complications

Page 18: Objectives - Internal Medicine Define diastolic and systolic heart

a) Papillary Muscle Rupture – ischemic necrosis and rupture of an LV papillary muscle may be rapidly fatal because of acute severe mitral regurgitation, which causes the valve leaflets to lose their anchoring attachments.

b) Ventricular free wall rupture – an infrequent but deadly complication, rupture of the free LV wall through a tear in the necrotic myocardium may occur within the first 2 weeks following an MI. Hemorrhage into the pericardial space owing to LV free wall rupture results in rapid cardiac tamponade, in which blood fills the pericardial space and severely restricts ventricular filling. -on occasion a pseudoaneurysm results if rupture of the free wall is incomplete and held in check by thrombus formation that plugs the hole in myocardium.

c) Ventricular Septal Rupture - blood is shunted across the ventricular septum from the LV to RV, usually precipitating CHF because of subsequent volume overload of pulmonary capillaries. d) True ventricular aneurysm – a late complication of MI, a true ventricular aneurysm occurs weeks to months after acute event. It develops as the ventricular wall is weakened, but not perforated by the phagocytic clearance of necrotic tissue and it results in a localized outward bulge (dyskinesia) when the residual viable heart muscle contracts. Complication of LV aneurysm include (1) thrombus formation within this regions of stagnant blood flow; (2) ventricular arrhythmias associated with the stretched myofibers; and (3) heart failure resulting from reduced forward cardio output. 6. Pericarditis:

-may occur in the early post-MI period as necrosis and neutrophilic infiltrates extend from the myocardium to the adjacent pericardium. *Dressler syndrome: uncommon form of pericarditis – immune process directed against damaged myocardial tissue is suspected to play a role. The syndrome is heralded by fever, malaise, and sharp, pleuritic chest pain typically accompanied by leukocytosis, an elevated ESR, and a pericardial effusion.

6. Thromboembolism:Stasis of blood flow in regions of impaired LV contraction after an MI may incite intracavity thrombus formation. Subsequent thromboemboli can result in devastating infarction of peripheral organs (e.g. stroke).

Choose appropriate long-term therapy post-acute coronary syndrome- Beta blocker: metoprolol 25-50mg BID or atenolol 50-100mg DIE- Statin: atorvastatin 80mg DIE- Ca channel blockers: for pts w ongoing/recurrent symp of ischemia despite good beta blocker tmt OR a-fib- Blood transfusion if Hg<11g/dL- Antiarrhythmic drugs if needed- Venous thromboembolism prophylaxis (LMWH) if immobilized- Proton pump inhibitor: prophylaxis of GI bleed- ACE-I: especially for STEMI, consider for NSTEMI, no evidence for unstable angina

Recognize pericarditis as a cause of chest pain

Pericarditis is an inflammation of the pericardium (sac around heart). Causes include: o idiopathic which is most commono infection (Coxsackie virus, echovirus, S. Penumoniae, S. aureus, TB, histoplasmosis, blastomycosis)o post-MI (Dressler`s syndrome if 2-8 weeks, direct extension if earlier)o metabolic (uremia, hypothyroidism)o neoplastic (hodkin`s disease, breast, lung, renal cell, melanoma)o collagen vascular disease (SLE, scleroderma, RA)o meds (hydralazine)

Page 19: Objectives - Internal Medicine Define diastolic and systolic heart

o radiationo infiltrative disease (sarcoidosis)

Diagnostic triad: 1) pleuritic chest pain (may be sharp, dull, aching, burning or pressing) – often alleviated by leaning forward, aggravated by respiration, can radiated to trapezius2) pericardial friction rub (in atrial/ventricular systole or rapid ventricular filling phase)3) characteristic ECG changes (diffuse ST elevation initially that resolves to flattened and inverted T waves over days, PR depression)

Define, and describe the treatment for, the following: severe hypertension, hypertensive crisis and malignant hypertension Severe Hypertension (hypertensive urgency):

- Severely elevated blood pressure (usually DBP >115) with no evidence of end-organ damage- Most commonly due to non-compliance with anti-hypertensive meds- Treatment: gradually reduce BP over 24-48 hours to a level appropriate for patient

Malignant Hypertension (hypertensive crisis):

- Acute elevation of SBP and DBP (SBP > 200-220mmHg, DBP >120mmHg) associated with end-organ damage (CNS, eyes, heart, kidney)

Common S&S: angina, dyspnea, neuro deficits

- cardiac: angina, MI, pulmonary edema, orthostasis- neuro: occipital headache, cerebral infarct or hemorrhage, visual disturbance, encephalopathy (nausea,

vomiting, etc.)- renal: oliguria, or any sign of renal disease- GI: nausea and vomiting

Causes:1) Meds: cocaine, MAOIs, OCPs, withdrawal of beta-blockers, clonidine or alcohol2) idiopathic3) renal artery stenosis4) pheochromocytoma5) aortic coarctation6) hyperaldosteronism7) hyper and hypothyroidism

Treatment:

- BP should not be lowered to normal levels; rapid reduction in BP results in decreased perfusion to cerebral, renal and coronary systems; MAP (mean arterial pressure) should never be lowered more than 20% in the first hour.

- If patient is stable, BP can be lowered to 160 systolic and 100-110 diastolic; requires continuous infusion of short acting parenteral (IV) antihypertensive agent with constant intensive monitoring (ex. Labetolol: 20mg IV over 2 min; up to 40-80mg at 10 min intervals)

- Most common drugs used to treat: Labetolol (B-blocker) and Sodium nitroprusside (vasodilator – vascular smooth muscle)

- Other drugs used:o Vasodilators: nicardipine (CCB), enalapril (ACE-I), nitroglycerin IV; Adrenergic inhibitors: esmolol (B-blocker)

Page 20: Objectives - Internal Medicine Define diastolic and systolic heart

Recognize the clinical presentation of secondary hypertension

Age: develops before 20 or after 50Severity: dramatic rise in BPOnset: abruptFamily hx: not presentAssociated signs and symptoms: depend on cause of secondary hypertension

Type Clinical cluesChronic renal disease Increased creatinine

Abnormal urinalysisPrimary aldosteronism Decreased serum potassiumRenovascular Abdominal bruit

Sudden onsetDecreased serum potassium

Pheochromocytoma Paroxysms of palpitation, diaphoresis and anxietyEpisodic hypertension

Coarctation of the aorta BP in right arm > left armCXR: aortic indentation

Cushing syndrome Central obesity, hirsutism

Define hyperaldosteronism, renovascular hypertension, pheochromocytoma and Cushing’s disease

** All 4 cause secondary HTN.

Hyperaldosteronism (primary and secondary)- Conditions in which the adrenal gland releases too much of the hormone aldosterone

o Primary aldosteonism = dysfunction of the gland itself. Rare. Usually caused by a adrenal adenoma/hyperplasia

↑ aldosterone secretion↑ → Na+ retention → arterial pressure Hypokalemia is also a prominent feature (from Na+/K+ tubular exchange)

o Secondary aldosteronism = something outside of the adrenal gland mimics the primary condition. Generally related to high blood pressure. Also related to: cirrhosis of the liver, HF, and nephritic syndrome.

- S&S: fatigue, headache, high blood pressure, intermittent paralysis, muscle weakness, numbness

Renovascular hypertension: - Decreased perfusion of renal tissue due to stenosis of main or branch of renal artery activites the rennin-

angiontensin system.o ↑ circulating angiotensin II → vasoconstriction + ↑ aldosterone secretion↑ → Na+ retention +

stimulation of adrenergic NS → ↑ arterial pressure

Pheochromocytoma:- Rare catecholamine-secreting tumor derived from chromaffin cells- Hypertension is the most common sign

o Often severeo Sustained or only during criseso Hypertensive paroxysms or crises, often spectacular and alarming, occur in > 50% of cases

Cushing’s disease:

Page 21: Objectives - Internal Medicine Define diastolic and systolic heart

- Syndrome characterized by truncal obesity, HTN, fatigability and weakness, amenorrhea, hirsutism, purplish abdominal striae, edema, glucosuria, osteoporosis, and a basophilic tumor of the pituitary

- Due to increased production of cortisol by the adrenal gland- HTN:

o Not necessarily volume dependanto May be secondary to glucocorticoid-induced production of rennin-substrate(angiotensin-

mediated hypertension)

Recognize sleep apnea and describe its main features, diagnosis and management.Sleep apnea is a syndrome in which people have repetitive periods of apnea that occur during sleep. A period of more than 10 seconds without airflow is generally considered to constitute an apneic episode, and patients with OSA often have hundreds of such episodes during the course of a night’s sleep.

Clinical Features: Markedly deranged sleep pattern – loud snoring is particular prominent Patients may have snorting and agitation as a result of trying to breathe against obstructed airway (may

also have violent movements during obstruction) Complaints from sleep partner of being hit or injured Complaints of headache on waking – relating to cerebral vasodilatation associated with derangements in

gas exchange that occur during apneic episodes. Sleep deprivation – so overly somnolent during the normal waking hours – can be debilitating Intellectual impairment (reduced attention and cognitive ability), memory loss, personality disturbances

(as a result of chronic hypoxia and extreme fatigue) Polycythemia Secondary cardiovascular complications of OSA are believed to be mediated in part by increased

sympathetic nervous system activity. During the episodes of apnea, patients may have a variety of cardiac arrhythmias or conduction disturbances.

As a result of prolonged hypoxemia at night, pulmonary hypertension can result, and unexplained cor pulmonale may be presenting clinical problem. Thus peripheral edema may result.

Systemic hypertensions associated with OSA

Diagnosis:Doctors diagnose sleep apnea based on medical and family histories, a physical exam, and results from sleep studies.

Normal Sleep ApneaApneic Events few ManyPeriods of Deep Sleep (S4) 2-3 of typically > 15 mins duration Few to many, but of barely

measurable durationPeriods of Re-awakening Few, 2-3 manyO2 saturation level Erratic and long-lasting fluctuations

with density of apneic episodesHeart rate Constant with minor short-lasting

fluctuationsErratic and fluctuating with density of apneic events

Snoring Intermittent, sparse Excessive, persistent

Management:In patients with central apnea (no drive to breathe during the apneic period – no signal for respiratory center to initiate respiration), treatment consists of the use of respiratory stimulants, an electrical, implanted phrenic nerve

Page 22: Objectives - Internal Medicine Define diastolic and systolic heart

pacemaker to stimulate the diaphragm or mechanical ventilation, either invasive via a tracheostomy tube or noninvasive via face mask.

In obstructive apnea (drive to breathe still present during apneic episode, but transient obstruction of the upper airway prevents inspiratory airflow – so inspiratory muscles are active but attempts to initiate airflow are unsuccessful), treatment includes weight loss in obese patients, elimination of respiratory depressants (alcohol and sedative-hypnotic drugs).

First-line therapy used in most patients with OSA is nasal CPAP. A mask connected to an air compressor is placed over the nose at bedtime. The compressor maintains positive pressure in the upper airway throughout the respiratory cycle, thus providing a pneumatic splint to keep the airway open. A modification to this treatment is Bilevel Airway Pressure (BiPAP) which provides a lower pressure during expiration and greatly increases the comfort of the patient. CPAP can be used in conjunction with oxygen in extreme cases of chronic hypoxia in severe sleep apnea. Oral/dental appliances: to trap the tongue forward and/or to pull the jaw forward to keep the airway open. Surgical modes of therapy may be beneficial in selected patients. Uvulopalatopharyngoplasty is sometimes used but has many complications.

Demonstrate an appropriate approach in performing a history and physical examination for the hypotensive patient (including listing potential causes and treatment)Hx:- Fatigue, weakness, muscle cramps, thirst, postural dizziness- More severe: oliguria, cyanosis, abdo and chest pain, confusion, obtundationPhysical: assess volume level (only cardiogenic has volume overload)- Sunken eyes, furrowed tongue, dry mucous membranes, extremity weakness- Assess hypovolemia: BP (↓ systolic, ↑ diastolic), reflex tachycardia, dry mucous membranes, skin tenting/↓

cap refill- ↓ intravascular volume: ↓ JVP, postural hypotension, postural tachycardia- Larger/acute fluid loss: hypovolemic shock → hypotension, tachycardia, peripheral vasoconstriction,

hypoperfusion (cyanosis, cold/clammy extremities, oliguria, altered mental status)- Labs: it seems like this might be too much detail, but I’ve been quizzed on it twice…

o ↑ BUN, plasma creatinine (because ↓ GFR)Normally BUN:creatinine ratio 10:1

Prerenal azotemia: hypovol → ↑ urea reabsorption = ↑ BUN → 20:1 or higher Ratio can ↑ with ↑ urea production from hyperalimentation (high protein), glucocorticoid

therapy, GI bleedingo Urine: normally hypovolemia → ↑ Na reabsorption => Na <20mmol/L

Urine osmolality and specific gravity in hypovol >450mosmol/kg and 1.015 - ↑ AVP secretion Acute tubular necrosis: ↓ Na reabsorption Hypovol from vomiting: metabolic alkalosis => more HCO3 secreted => less Na reabsorbed;

However Cl is low (<20mmol/L) Diabetes insipidus: urine osmolality and specific gravity indicate inappropriate urine dilution

Diff dx:1) ECF volume contracted

a. Extrarenal Na loss:i. GI: vomiting, NG suction, drainage, fistula, diarrhea

ii. Skin/respiratory: fluid lost to thermoregulation – sweat, Na/H2O loss from skin iii. Hemorrhageiv. Burns, pancreatitis, peritonitis

b. Renal Na and water lossi. Diuretics

Page 23: Objectives - Internal Medicine Define diastolic and systolic heart

ii. Osmotic dieresis: eg. mannitoliii. Hypoaldosteronism: causes salt wasting

Note: aldosterone ↓ Na concentration in sweat iv. Salt-wasting nephropathies

c. Renal water lossi. Diabetes insipidus: central or nephrogenic

2) ECF volume normal or expandeda. ↓ CO

i. Myocardial, valvular, pericardial diseaseb. Redistribution

i. Hypoalbuminemia: hepatic cirrhosis, nephritic syndromeii. Capillary leak: acute pancreatitis, ischemic bowel, rhabdomyolysis

c. ↑ venous capacitancei. sepsis

Discuss the types of shock and clinical markers thereof

Le choc est un état pathologique résultant de trouble circulatoire et/ou métabolique majeur, TOUT choc a une hypotension artérielle (tension artérielle = débit cardiaque X résistance vasculaire périphérique) et une hypoxie tissulaire générale, aussi état mental diminué ou altéré, pâleur et cyanose, peau froide et moite, mauvais remplissage capillaire, RR plus de 32/min, PA systolique moins de 80, débit urinaire moins de 0.5 ml/kg/h, lactate sanguin plus de 5 mmol/L

- penser au corps comme une plomberie- cardiogénique = problème de pompe, diminution de débit cardiaque, perfusion et TA, compensation = vasoconstriction, tachy, augmentation extraction O2 par les tissus périphériques

- diminution de contractilité : ischémie, IM, arythmie, cardiomyopathie- trouble de remplissage : tamponage, péricardite constrictive, Tension penumothorax- obstruction du débit : EP, coarctation, sténose valvulaire

- hypovolémique : penser à une fuite dans le système = sang (trauma ou chirurgie), plasma ou eau (deshydratation, brulures, obstruction intestinale, pertes rénales anormales), donc diminution volume circulant ou précharge, diminution du débit cardiaque, compensation = vasoconstriction, tachy, augmentation extraction O2 par les tissus périphériques- choc septique : penser dilatation des tuyaux, syndrome infectieux avec 2 signes de SIRS (T plus de 38C, p plus de 90/min, RR plus de 32 /min, leucocytose ou leucopénie), hypoperfusion et dysfonction d’au moins un organe vital, DONC diminution de RVP, vasodilatation, hypotention, augmentation du shunt artério-veineux, diminution extraction O2 par les tissus périphériques, compensation = tachy, augmentation débit cardiaque

- anaphylaxie : penser tuyaux perforés, libération de substances vasoactives, extravasation du volumecirculatoire, bronchospasme, mixte de hypovolémie et distributive, fuite capillaire, diminution débit cardiaque et RVP, vasodilatation, hypotension, compensation = tachy

Page 24: Objectives - Internal Medicine Define diastolic and systolic heart

Perte estimative de liquide et de sang (pour une personne de 70 kg)

selon les premières manifestations Catégorie I Catégorie II Catégorie III Catégorie IVPerte de sang (ml) jusqu’à 750 750 à 1 500 1 500 à 2 000 > 2 000

Perte de sang (% volume sanguin) jusqu’à 15 % 15 à 30 % 30 à 40 % > 40 %

Pouls < 100 > 100 > 120 > 140TA normale normale à la baisse à la baisse

Tension différentielle normale ou à la hausse à la baisse à la baisse à la baisse

Fréquence respiratoire 14 à 20 20 à 30 30 à 40 > 35Diurèse (ml/h) > 30 20 à 30 5 à 15 Nulle

État du SNC légère anxiété légère anxiété anxiété /confusion

confusion /léthargie

Remplacement liquidien (règle du 3:1) soluté cristalloïde soluté

cristalloïdesoluté cristalloïde et sang

soluté cristalloïde et sang

Le rapport 3:1 constitue une ligne directrice fondée sur le fait que la plupart des patients ont besoin de 300 cc de solution d’électrolytes pour chaque 100 cc de sang perdu. (Extrait adapté du manuel ATLS, 1997).

CVP PAOP SVR/SCRI CO/CIHypovolenic ↓ ↕ ↓ ↑ ↓Cardiogenic ↑ ↑ ↑ ↓Distributive- Sepsis- anaphylaxis

↔ ↓↔ ↓

↔ ↓↔ ↓

↓↓

↕↑

Neurogenic ↔ ↔ ↓ ↔ ↑Obstructive- tamponade- tension ptx- massive PE

↑↕↑

↑↕↕

↑↑↑

↓↓↓

CVP = central venous pressurePAOP = pulmonary artery occlusion pressureSVR = systemic vascular resistanceSVRI = systemic vascular resistance indexCO/CI = cardiac output, cardiac index

Page 25: Objectives - Internal Medicine Define diastolic and systolic heart

Pathophysiology and hemodynamic profile of shock states

Physiologic variable

Preload Pump function Afterload Tissue perfusion

Clinical measurement

Pulmonary capillary wedge

pressure

Cardiac output Systemic vascular resistance

Mixed venous oxygen saturation

Hypovolemic ↓ ↓ ↑ ↓

Cardiogenic ↑ ↓ ↑ ↓

Distributive ↓ or ↔ ↑ ↓ ↑

Demonstrate an appropriate approach in performing a history and physical examination in the care of a patient in shock

-shock: systemic organ hypoperfusion, usually associated with hypotension.-strategy: gain vascular access quickly & aggressively replace volume

Monitor BP and organ perfusion (ex. Cerebral, cardiac and renal function) Central venous line for fluids Arterial line: measurement of oxygen exchange

-History & Physical Exam are specific to 4 classifications of shock In General: tachycardia (weak pulse), hypotension, +/- fever, +/- O2 Sat changes, +/- rapid, shallow

breathing, shifting level of consciousness1) CARDIOGENIC SHOCK (decreased cardiac output as a result of cardiac dysfunction)

a. Hx: pain? MI symptoms: sweating, palpitations, etc., symptoms of CHF (edema, orthopnea, PND), antecedents: cardiac, other?

b. P/E: sweating, MI signs (incl. ECG), cardiac exam: B1, B2, B3 (surcharge volumique), B4 (ventricules moins complaints), creps in base of lungs, edema (lower limbs)

2) HYPOVOLEMIC SHOCK (decreased intravascular volume)a. Hx: diarrhea? Vomiting? dehydration? Heat exhaustion? Hemorrhage/bleeding?b. P/E: muqueuses sèches, dry skin; improvement with normal saline infusion?

3) SEPTIC OR REDISTRIBUTIVE SHOCK (decreased systemic vascular resistancea. Hx: cuts, sites of infection? Bleeding site? Recent injury/trauma, surgery? Recent infection?

Timeline importantb. P/E: like hypovolemic, although may have fever, palpitations and tachycardia

4) OBSTRUCTIVE SHOCK (decreased cardiac output as a result of obstruction or anaphylaxis)a. Hx: allergies? Recent exposures?

5) P/E: stridor, laryngeal swelling, sweating, tachycardia, tacchypnea

Describe the anatomy and physiology of the peripheral vascular and lymphatic systems

Peripheral vascular system: blood caries nutrients, oxygen and waste products to and from the cells, exchange is done in the capillary bed.

Page 26: Objectives - Internal Medicine Define diastolic and systolic heart

Peripheral lymphatic system: provides drainage of surplus tissue fluid and leaked plasma proteins to the bloodstream , as well for removal of debris from cellular decomposition and infection .

Lymph nodes: small masses of lymphatic tissue through which lymph from the lymphatic vessels must be filtered.

Lymphoid tissue: in the walls of the digestive tract, in the spleen, thymus, lymph nodes

List the main causes of both unilateral and bilateral edemaDemonstrate an ability to elicit the various clinical signs and symptoms of edema in order to establish the etiological diagnosis

LOCAL EDEMA (unilateral or localized) GENERALIZED EDEMA

Page 27: Objectives - Internal Medicine Define diastolic and systolic heart

- Inflammation / infectiono Vasculitis, cellulitis, etc.

- Venous or lymphatic obstruction/stasiso Crhonic venous insufficiency/phebitiso DVTo Chronic lymphangitiso Filariasis

- Arterial insufficiency- Trauma/injury

- Increased hydrostatic pressureo Increased fluid retention

Cardiac cause (e.g. CHF) Hepatic causes (e.g. cirrhosis) Renal causes (e.g. acute and

chronic renal failure)o Vasodilators (especially CCB)o Refeeding edema

- Decreased oncotic pressureo Hypoalbuminemia

- Hormonalo Hypothyroidismo Exogenous steroidso Pregnancyo Estrogens

Clinical signs and sx1) Hx

o Atherosclerotic RF : smoking, HTN, DM, FHx, Lifestyle (diet, exercise), hyperlipidemiao CHF: peripheral edema, pulmonary congestion, exercise intoleranceo Precipitants of venous thrombosis

Recent prolonged immobilization (DVT) Post-operatively (especially ortho, thoracic, GI, GU)

Trauma (fracture of femur, pelvis, spine, tibia) Post-MI/CHF Long travel

Hormone-related: pregnancy, oral contraceptive pill, hormone replacement therapy Inheritable hypercoaguability (APLA syndrome, Factor V Leiden, Prothrombin G20210A, etc.) Underlying malignancy

o Risk factor for thromboembolitic events A fib, post MI <3 months, valvular disease, prosthetic valves, endocarditis, cardiomyopathy

o Vasculitis Skin changes, mucosal ulceration, joint pain, changes in vision and/or eye pain

o Symptoms of ischemia (6 Ps) Polar (cold) – comes first Pain – absent in 20 % Pallor Paresthesia – light touch lost first (small fibers) followed by other sensory modalities Paralysis/Power loss – further progression of ischemia Pulselessness

o Critical ischemia Acute = look for 6Ps + neuromotor dysfunction Chronic = night pain, rest pain, tissue loss (ulceration/gangrene)

2) PEo Check for pitting edema

Suggestive of orthostasis, chronic venous insufficiency or CHFo Firm, non pitting edema suggests lymphatic obstructiono Note prominent veins

Arterial insufficiency Venous statis Vasculitis- Cool, pale extremities ± cyanosis - Warm, thinkening and erythema - Livedo reticularis rash (bluish-

Page 28: Objectives - Internal Medicine Define diastolic and systolic heart

(only if severe)- Swelling, heaviness and aching in

legs (usually medial lower 1/3) - Arterial bruit may be present- ↓BP in the affected limb (and

pulse)- Loss of hair on the legs or feet- Painful, non-bleeding ulcers on

the feet or toes (usually black) that are slow to heal (severe)

- Shiny, tight skin (severe)- Thick toenails (severe)

over ankle and lower leg- Stasis ulceration of ankle or above

medial malleolus o Painlesso Develop slowlyo No distinct borders

- Varicosities/prominent veinsCHRONIC VENOUS INSUFFICIENCY- Warm, erythematous,

thickened skin, ↑ pigmentation, ± brown ulcers around ankles

SUPERFICIAL PHELBITIS- Warm, painfull, erythema

secondary to inflammation around vein

ACUTE DVT- Pain secondary to inflammation

in absence of superficial ∆, swelling of distal part of extremities

red discolouration, network pattern)

- Malar or discoid rash on face in SLE

- Raynaud’s phenomenon: episodes of sharply demarcated pallor and/or cyanosis, then erythema of digot

- Purpura- Look for active swollen joints- Inflammation of eye chambers- Venous/arterial

thrombosis/ulcers, ulcers, gangrene

3) Special testsArterial insufficiency Venous insufficiencyALLEN TEST- Test of good collateral flow through ulnar artery

before doing puncture of radial artery ofr arterial blood-gad

STRAIGHT LEG RAISE (Pallor on elevation)- Raison leg 40-60 degree for 30 sec., or until pallor

of the feet develops- Normal = mild pallor on elevation- Abnormal = marked pallor on elevation (suggestive)RUBOR ON DEPENDENCY- After straight leg raise- Ask pt to sit up and dangle both legs over the side

of the bed- Normal = Return to normal colour within 10-15 sec.- Abnormal = persistent pallor of the feet > 10 sec.

followed by rubor (marked redness) on dependency (after 1-2 min)

** seen in pt with critical ischemia

INCOMPETENT SAPHENOUS VEIN- Ask pt to stand and compress the vein proximally

with one hand and place the other hand 15-20 cm distally

- Briskly compresse/decompress the distal site- Normal = hand on proximal site should feel no

impulse- Abnormal = any impulse transmitted to proximal

siteBRODIE-TRENDELENBURG MANEUVER- Ask pt to lie in supine position and raise leg to 90

degree x15 sec (to empty vein)- Place tourniquet around pt upper thigh, ask pt to

stand and watch for venous filling with tourniquet still x60 sec

- Normal = superficial saphenous should fill slowly below tourniquet within 35 sec

- Abnormal = early filling indicates incompetence of deep and perforator veins. Filling of vein from above (retrograde flow) suggests superficial bvenous incompetence (after tourniquet has been removed)

Outline a useful investigation plan for the diagnosis of edemaLymphedema:

Page 29: Objectives - Internal Medicine Define diastolic and systolic heart

Lymphoscintigraphy – used to define anatomy and patency, evaluate dynamics of flow and reversal of flow, and determine the severity of obstruction

CT and MRI – delineate nodal architecture at increased cost but limited advantage over scintig. Doppler – presence of DVT is in the differential of unilateral swelling, and it may also occur concomitantly

with lymphedema – cost-effective test in appropriate situation.

Cardiogenic Pulmonary Edema: CBC – asses for severe anemia and may suggest sepsis or infection if markedly increased WBC Lytes – People with CHF often use diuretics so would be predisposed to lyte abnormalities Patients with CKD are at risk for hyperkalemia BUN and Cr – help to assess renal failure CXR – distinguish CPE from other pulmonary causes of severe dyspnea An enlarged heart, Kerley B lines, absence of air bronchograms and presence of pleural effusion suggest

CPE Arterial blood gases Oxygen sats ECG – LVH

Ankle edema in general: Pregnancy Meds – CCBs, NSAIds, estrogens LFTs, albumin, Cr, BUN, CK D-Dimer and ultrasound of calvesc to R/O DVT

Propose a treatment plan for edemaConditions associated with development of edema: heart failure, cirrhosis, nephritic syndrome, venous/lymphatic diseasePulmonary edema = lifethreatening

- Loop diuretic (furosemide 10-40mg): monitor degree of dieresis, fluid and electrolyte complications (hypokalemia, metabl alkalosis, hyponatremia, hyperuricemia, hypoperfusion - ↑ serum creat)

- Cirrhosis: spironolactone + furosemide, proceed slowly in absence of edema- Tense ascites: total paracentesis- Heart failure: monitor hypoperfusion (↑ serum creat)- Nephritic syndrome: may need higher than normal doses of loop diuretic bc binding of diuretic by albumin

in tubular lumen → inactive + ↓ total functioning nephrons - Idiopathic edema already on diuretics: discont diuretic 2-3 wks (could be diuretic induced)- Resistant edema from any cause: high-dose IV loop diuretic + thiazide (metolazone, hydrochlorothiazide)

Interpret a pulmonary function test

Page 30: Objectives - Internal Medicine Define diastolic and systolic heart

Different Tests:1) Total Lung Capacity (TLC)- volume of air in lungs after max inspiratory effort2) Vital Capacity (VC)- max volume of air that can be expelled from the lungs following max inspiration3) Residual Volume (RV)- volume of air remaining in lungs after a maximal expiratory effort4) FEV1 – forced expiratory volume in one second, commonly used to screen for airway disease, a measure of

flow rate5) Diffusion capacity (DLCO)- evaluation of the diffusing capacity for carbon monoxide indicates the adequacy of

alveolar-capillary gas exchange. Common causes of DLCO are emphysema, interstitial lung disease, and pulmonary vascular disease

Obstructive Restrictive- obstructive airflow, decreased flow rates (most marked during expiration), air trapping (increased RV/TLC) and hyperinflation (increased FRC, TLC)

- decreased lung compliance and lung volumes

asthma COPD Cystic fibrosis bronchiectasis

interstitial lung disease neuromuscular disease chest wall disease pleural disease parenchymal disease (penumonia)

FEV1(%) = FEV1/FVC. FEV1(%) is the percent of the forced vital capacity expelled in one second.

Approach:1. Look at the FEV1 first:

-if the FEV1(%) is normal or high, may be restrictive disease-if the FEV1(%) is decreased (<80%), then the patient has airflow limitation suggestive of obstructive disease

->if the FEV1 does not improve after bronchodilator administration, the patient has fixed airway obstruction (COPD)

2. Look at the lung volumes:- TLC, RV: suggests hyperinflation and gas trapping, consistent with obstructive disease- TLC, RV: suggests restriction and is diagnostic of restrictive lung disease

3. Look at the DLCO;-DLCO should be corrected for hemoglobin (DLCO-C) and alveolar volume (DLCO/VA)-if patient has restrictive disease, a low DLCO/VA implies interstitial lung disease-if DLCO is low but DLCO/VA is normal, restrictive disease due to low lung volumes (obesity, skeletal deformities)-if patient has obstructive disease, low DLCO-C suggests presence of emphysema

Page 31: Objectives - Internal Medicine Define diastolic and systolic heart

Interpret a plain film chest x-ray

1) Clichés: patient upright (PA et lateral) or patient supine/sitting (portable; AP – heart appears more magnified)

2) Pulmonary volumes: in a normal CXR, you can see 8-10 posterior ribs3) Heart size: in a normal CXR, the heart width is ½ the width of the total lung space4) Mediastinal or pulmonary hilar abnormalities?

a. Ex. Mediastinum too large: mass or vascular abnormlity (i.e. aneurysm)b. Hilar ‘swelling’: lymph nodes or dilated pulmonary arteries (NB: each hilum should be concave!)c. Cardiomegaly (i.e. in heart failure or congenital problem)

5) Look at each lung individually and compare the 2 sidesa. Lungs too white: alveolar consolidation - pneumonia, atelectasia, interstitial disease, mass,

pleural effusion (épanchement – look for signe du ménisque when patient is standing)i. NB: bronchogramme aérique: vessels become more black against interstitial

consolidation – pneumoniaii. Signe de silhouette: the contours of 2 adjacent structures with similar densities are not

well-defined b. Lungs too black: pneumothorax or emphysema (hyperinflation)

6) Are the diaphragmatic curves (both sides) well-defined?7) Examine the ribs, clavicles, shoulders and vertebrae

a. Fracture? Mass?8) Examine the soft tissue

a. Mass?9) Look under the diaphragm for abnormalities10) Rule of thumb: if you find one abnormality, keep looking for others

Interpret arterial blood gases in the context of pneumonia

ABG is not commonly done in the context of pneumonia, unless very diffuse or with severe underlying COPD

Assesses adequacy of gas exchange and ventilator insufficiency Respiratory acidosis may be present (due to parenchymal infiltration resulting in hypoventilation) or

respiratory alkalosis (due to hypoxemia causing hyperventilation) Increased A-a gradient

Page 32: Objectives - Internal Medicine Define diastolic and systolic heart

Pa02 < 95 mmHg which may not improve by giving 100% O2

Discuss the criteria for qualification of home oxygen

AUTOMATIC QUALIFICATION for the following conditions:o Pulmonary fibrosiso Cor pulmonaleo Pulmonary hypertension

GERENAL CRITERIA1) The condition must be stabilized and treatment regimen optimized before long-term oxygen therapy is

considered. Optimum treatment includes smoking cessation.

2) Pt must have chronic hypoxemia on room air at rest Pa02 ≤ 55mmHg, OR Sa02 ≤ 88%

3) Pt with persistent Pa02 in the range of 56 to 60 mmHg may be considered candidates for long-term oxygen therapy if any of the following medical conditions are present:

cor pulmonale pulmonary hypertension persistent erythrocytosis

4) Also, some pt with a persistent Pa02 in the range of 56 to 60mmHg may be candidates for long-term oxygen therapy if the following occurs:

exercise limited hypoxemia and documented to improve with supplemental oxygen nocturnal hypoxemia

HYPOXEMIA ON EXERTION = Pt who exhibit exertional hypoxemia only1) Pt must exhibit hypoxemia on exertion (exertional saturation ≤ 88%) on room air and improve exercise

tolerance with O2 Desaturation < 80% on walking, regardless of dyspnea or distance walk If pt able to walk > 5 min, must demonstrate an objective measured improvement in walking

performance on O2 so that the distance walked increase by 25% + one unit in the BORG score at the end-exercise point of the shortest test

If pt able to walk < 5 min, must demonstrate an objective measured improvement in walking performance so that the time walked increases ≥ 2 min + one unit in the BORG score at the near end-exercise point of the shortest test

Demonstrate an ability to perform an appropriate physical examination of a patient with COPDOn physical examination, patients with emphysema often appear thin (cachectic), and they frequently lean forward and rest on extended arms (tripod position). This position allows fixation of one end of the shoulder and neck muscles, allowing them to function more effectively as muscles of respiration. These patients are not cyanotic and do not demonstrate peripheral edema characteristic of RV failure. In contrast, patients with chronic bronchitis often are obese and sometimes cyanotic but generally appear to be in less respiratory distress than those with emphysema.

Page 33: Objectives - Internal Medicine Define diastolic and systolic heart

Examination of the chest discloses an increased anteroposterior diameter as a result of hyperinflation of lungs. Patients may be using accessory muscles of respiration, such as sternocleidomastoid and trapezius, and there may be retraction of intercoastal muscles with inspiration. When diaphragmatic excursion is assessed by percussion of the lung bases during inspiration and expiration, diminished movement is noted. Breath sounds are generally decreased in intensity, and expiration is prolonged. Wheezing may be heard or it may be brought out with forced exhalation. In patients with chronic bronchitis and profuse airway secretions, coarse gurgling sounds labeled as rhonchi are frequently appreciated.

Blue Bloaters patients often present with a hx of recurrent infection, chronic cough and abundant purulent sputum; patients can show signs of hypercapnia and severe hypoxemia resulting in cyanosis; patients may have peripheral edema due to RV failure; pulmonary hypertension and cor pulmonale (overload on RV) are complications of chronic bronchitis

Pink Puffers severe dyspnea; barrel chest; px may sit fwd in hunched-over position, with a pinched face and pursed lips; patients may overventilate and remain well oxygenated

Establish an appropriate treatment plan for exacerbations of COPD, taking into account the latest Canadian consensus guidelines

1) Oxygen: Sat 90-94% 2) Short acting Beta Agonist: nebulized albuterol 2.5mg q1-4h PRN or 4-8 puffs (90mcg/puff) w spacer →

NOTE: ↑ dose does not affect outcome 3) Short acting anti-cholinergic: 500mcg nebulized ipratroprium q4h4) Systemic corticosteroid: prednisone – 2 week therapy w taper at end

- First 24 hours: 80-125mg- If symptoms ↓: 40-60 mg/day

5) Ab: only treat severe exacerbation requiring mechanical ventilation, or exacerbation w ↑ sputum purulence + ↑ dyspnea OR ↑ sputum volume – see Ab choice for pneumonia

Describe the mechanism of action of the main pharmacological agents available to treat COPD

B2-agonsists Act on B2 receptors resulting in bronchodilation, improve lung function and symptoms (salbutamol for short-acting, salmeterol for long-acting)

Anticholinergics Block acetylcholine resulting in bronchodilation (synergistic with b- agonists), ipatropium for short acting, tiotripium for long acting

Glucocorticoids Systemic steroids work by decreasing the inflammatory immune response, used for exacerbations and NOT long term management

Methylxananthines Weak bronchodilator and offer modest improvement in pulmonary function, not used acutely (Theophylline)

- use a combination of short and long acting treatments

Discuss the various diagnostic tests for asthma

-requires documentation of 1) hyperactive airways, and 2) reversible obstruction to flow*since asthma attacks/symptoms come and go, you may need a bronchoprovocation test/challenge; NB: history maybe enough in most cases

Test Description and Expected Results, If Asthmatic

Page 34: Objectives - Internal Medicine Define diastolic and systolic heart

Spirometry (routine pulmonary function tests) Decreased Forced Expiratory Volume in 1 sec (FEV1), hyperinflation, improvement with bronchodilator

Provocation with bronchoconstrictor (histamine, metacholine, cold air, exercise); methacholine is a synthetic acetylcholine that is preferred because it has fewer systemic effects

Diagnosis depends on amount of stimulus to produce an effect: the concentration of methacholine required to produce a 20% reduction in baseline FEV1 is reported; NB: not alone diagnostic, but can help rule out asthma.

Lung volume measurements May show hyperinflation during active disease but it’s typically normal because asthma alone does not destroy acini

Arterial blood gas May be useful during acute exacerbations for determining gas-exchange status

Chest radiograph Usually only done if concern for infection; fleeting infiltrates and central bronchiectasus common in allergic bronchopulmonary aspergillosis

Blood tests May show eosinophilia and increased IgE (increased in atopy)

Skin prick Allergy tests for household items and antigens that might precipitate attacks

Asthmatics will show REVERSIBILITY of airway obstructiono FEV1 with increase ≥12% (>180 ml) 15-20 minutes after inhalation of B2-agonisto FEV1 decreases >10% after 6 minutes of exerciseo Methacholine challenge: FEV1 will decrease 20% with <8mg/ml methacholine compared to

control subjects

List the criteria for adequate control of the asthmatic patient

Classification of asthma by severity : clinical aspects and treatmentAsthma severity

Clinical features before tx Nighttime symptoms

Lung function Treatment

Mild intermittent

Symptoms < 3x/week Capable of normal activity Exacerbations brief

<3x/month

FEV1 > 80% predicted

FEV1/FVC normal

Short acting ß2 agonist prn

Mild persistent

Symptoms 3x/week, but < 1x/day

Minor limitation w normal activity

3-4x/month

FEV1 > 80% predicted

FEV1/FVC normal

Short acting ß2 agonist prn

Inhaled corticosteroid or sustained release theophylline, leukotriene modifier or cromoglycate

Moderate persistent

Daily symptoms Daily use SA ß2 agonist Limitation w normal

activity Exacerbations >2x/week;

may last days

>1x/week but not nightly

FEV1 60- 80% predicted

FEV1/FVC reduced 5%

Low-dose inhaled corticosteroid + long acting ß2 agonist

OR Medium-dose

inhaled corticosteroid

Severe Continual symptoms Often FEV1 <60% Medium or high-dose

Page 35: Objectives - Internal Medicine Define diastolic and systolic heart

persistent Extremely limited w normal activity

Frequent exacerbations

nightly predicted FEV1/FVC reduced

5%

inhaled corticosteroid + long acting ß2 agonist

Consider short course oral corticosteroid

Discuss the various treatments available to control asthma

1) Lifestyle/risk modification2) Medication3) Desensitisation

Medication

Management algorithm based on level of controlIf well-controlled for ≥ 3 mo, step down ↔ If not well-controlled, step up after addressing:- Avoidance- Adherence- Comorbities- Triggers

Step 1 Step 2 Step 3 Step 4 Step 5 Step 6Rapid medication As-needed SABA

Preferred controller medication

- Low-dose ICS Low-dose ICS +LABAOr Medium-dose ICS

Medium-dose ICS+ LABA

High-dose ICS+ LABA+Evaluation for omalizumab

Add OCS to Step 5 medication

Alternate controller medication

- LTRAorCromolynOrSustained release theophylline

Low-dose ICS +LTMOrSustained release theophylline

Consider adding LTMAnd/orSustained release theophylline

1 st line

Short-acting ß2 (bronchodilators) MDI or Inhaler prnSalbutamol (ventolin)- 1st line tmt for LT management of all severities of asthma + rapid tmt of exacerbations

Inhaled corticosteroids DPI, MDI or nebulizedBudenoside (pulmicort), Fluticasone (Flovent), Beclomethasone (Qvar)- For persistent asthma- Dosing depends on assessment of severity and control- Attemps should be made to ↓ dose of ICS every 2-3 mo to the lowest possible dose to maintain control

Page 36: Objectives - Internal Medicine Define diastolic and systolic heart

Long-acting inhaled ß agonists (LABA) Formoterol (Oxeze, Serevent), Salmeterol/Fluticasone (Advair), Budenoside/Formeterol (Symbicort)- Recommended for moderate and severe persistent asthma, in pt not adequately controlled with ICS- Should be used in combination with ICS: Salmeterol/Fluticasone, Budenoside/Formeterol)- Salmeterol and Formoterol, added to ICS have been shown to ↑ lung function , both day and nighttime, ↓

exacerbations, and minimize the required dose of ICS

Systemic corticosteroids- May be necessary to gain control of disease quickly via either oral or IV route- If chronic sx are severe, accompanied by nighttime awakening, or PEF < 70 % of predictive values, a short

course of systemic corticosteroids (prednisone 40-60 mg/d q 5-7 days) might be necessary- Should be administered to all pt during an exacerbation

o Speed the resolution of exacerbationo Ideal dose to speed recovery and limit sx not well defined. Various options.

2 nd line

Leukotrienes modifiers- Leukotriene-receptor tantagonists = Montelukast (10 mg PO qd) and Zafirlukast (20 mg PO bid)

o Recommended as alternative medication for mild persistent asthma- 5-lipoxygenase inhibitor = Zileuton (extended release 1200 mg bid)

o Recommended as an add-on to ICS for more severe forms of asthma- Improve lung function, quality of life and lead to fewer exacerbations but not as effective as ICS + LABA in

improving asthma outcomes

Discuss the treatment options for OSA 1) Modifiable factors: weight loss, ↓ alcohol/sedative use (↓ sleep quality), nasal decongestion, underlying

medical condition 2) Postural therapy: for pts with positional sleep apnea (ie. Sleep apnea only when lying on back). Suggest

“tennis ball technique”: put tennis ball in sock, and sew to pyjamas → prevents pt from rolling onto back 3) Mild apnea: dental appliance (expensive) 4) CPAP: in some pts, if CPAP is too high, central sleep apnea can be induced – therefore we want the lowest

pressure possible that gets the AHI <5/hr, without causing CSA 5) Peds: uvulopalatopharyngoplasty, tonsillectomy → not proven effective in adults 6) Tracheostomy: used as last resort

Describe the signs and symptoms of Obstructive Sleep Apnea (OSA) - Frequent arousals and fragmented sleep cause: daytime somnolence, personality/cognitive changes,

snoring – most obvious symptom - Hypoxemia, hypercapnia cause: headache upon waking, polycythemia, pulmonary/systemic HTN, cor

pulmonale/CHF, nocturnal angina, arrhythmias - Leg movements can be related to apneic events

… also…

- habitual snoring- witnessed apneas- nocturnal gasping or choking

Page 37: Objectives - Internal Medicine Define diastolic and systolic heart

- fatigue (lack of restful sleep and persistent hypoxia (later stage of pulmonary hypertension)- daytime hypersomnolence (excessive daytime sleepiness)- intellectual impairment (reduced attention and cognitive ability), memory loss, personality disturbances- swollen ankles / peripheral edema (cor-pulmonale – right heart failure resulting from pulmonary hypertension as a result of chronic hypoxia during the apneic events)- obesity (presumably from reduced daytime physical activity - fatigue)- pulmonary hypertension (resulting from chronic hypoxia in pulmonary vasculature)- systemic hypertension (chronic sympathetic stimulation after the apneic events)- chronic hypoventilation/CO2 retention (reset central respiratory control level)

Discuss the complications of OSA - Depression, weight gain (due to hormonal changes with ↓ sleep, AND ↓ energy because of less sleep), ↓

energy, ↓ quality of life, motor vehicle accidents risk x6, cardiac complications (HTN, CHF, cor pulmonale, nocturnal angina, arrhythmias), ↓ social function, complications during and post-surgery

Describe the underlying pathophysiology of pneumonia

-Most common routes of entry: aspiration of oropharyngeal secretionsEx. Legionelle, mycobateria, endemic fungi, mycoplasma pneumonia, Chlamydia pneumonia, most virus’

-Less common route: hematogenous or embolic spread from infection heart valves or venous clot (often multifocal with peripheral lesions susceptible to cavitation)

COMMUNITY-ACQUIRED PNEUMONIA Streptococcus pneumonia - most common (an encapsulated gramp +ve diplococcal bacteria) Mycoplasma pneumonia (facultative anaerobic organism)

NOSOCOMIAL PNEUMONIA 3 types: 1) hospital-acquired, 2) ventilator-associated, 3) health care –association (contact with health

care worker) Based on colonization of oropharynx and stomach with virulent pathogens and subsequent aspiration into

lower respiratory tract Gastric colonization by gram –ve organisms increased by neutralization of gastric acidity Within 1st 5 days: H. influenza, Strep. pneumonia, S. Aureus More than 5 days: P. aeruginosa, S. Aureus, anaerobics, acinobacter

PATHOGEN COMMUNITY-ACQUIRED NOSOCOMIALBACTERIAL 70-80% 90%S. pneumonia 60-75% 3-9%H Influenza 4-5% Up to 25%Legionella 2-5% 10-20%S Aureus 1-5% 10-20%Gram –ve bacilli Rare 50%ATYPICAL 10-20% RareM pneumonia 5-18% --Clam. Psittaci 2-3% --Cloxiella Burnetti 1% --VIRUS --

Page 38: Objectives - Internal Medicine Define diastolic and systolic heart

Influenza virus -- 8%Hantavirus -- rare

Recognize pneumonia on a chest x-ray

Lobular consolidation Suggests bacterial pneumonia

Interstitial infiltrates Suggest non-bacterial pneumonia (but may be seen in early staphylococcal pneumonia

Inlarged hilar lymph nodes Concomitant lung tumor Primary tuberculosis, viral or fungal pneumonias

Large pleural effusions Streptococcal pneumonia or tuberculosis

Negative CXR Cannot rule out pneumonia

Identify the severity criteria associated with pneumonia

Factors increasing severity of pneumonia: 1) age < 50 years2) neoplastic disease3) heart failure4) cerebrovascular disease, renal disease or liver disease5) altered mental status6) pulse > 125 beats/minute7) respiratory rate > 30 breaths/minute8) systolic blood pressure < 90 mm Hg9) temperature > 40 degrees C (104 degrees F) or < 35 degrees C (95 degrees F)10) Hx of cancer, CHF, CRF, chronic liver disease

Page 39: Objectives - Internal Medicine Define diastolic and systolic heart

** mortality < 0.5% if none of the above criteria** use Pneumonia Severity Index Calculator or see Pneumonia severity assessment to predict mortality if risk factors (see below)

Mortality/severity varies by etiology- Mortality ranges from < 2% for certain pathogens to > 30% for gram-negative bacteria- Legionella pneumonia also associated with ↑ severity

Prognostic factors associated with mortality are:- Male gender- Pleuritic chest pain- Hypothermia- Systolic hypertension- Tachypnea, diabetes mellitus- Neoplastic disease- Neurologic disease- Leukopenia, bacteremia- Multilobar infiltrate on x-ray

Pneumonia Severity Index/AssessmentSTEP 1

Presence of:

- Over 50 years of age Yes/No

- Altered mental status Yes/No

- Pulse ≥125/minute Yes/No

- Respiratory rate >30/minute Yes/No

- Systolic blood pressure <90 mm Hg Yes/No

- Temperature <35°C or ≥40°C Yes/No

History of:

- Neoplastic disease Yes/No

- Congestive heart failure Yes/No

- Cerebrovascular disease Yes/No

- Renal disease Yes/No

- Liver disease Yes/NoIf any yes, proceed to step 2. If all no, assign Risk Class I

STEP 2

Demographics:

- If Male +Age (yr)

- If Female +Age (yr) - 10

- Nursing home resident +10

Comorbidity:

- Neoplastic disease +30

- Liver disease +20

Page 40: Objectives - Internal Medicine Define diastolic and systolic heart

- Congestive heart failure +10

- Cerebrovascular disease +10

- Renal disease +10

Physical exam findings:

- Altered mental status +20

- Pulse ≥125/minute +20

- Respiratory rate >30/minute +20

- Systolic blood pressure <90 mm Hg +15

- Temperature <35°C or ≥40°C +10

Lab and radiographic findings:

- Arterial pH <7.35 +30

- Blood urea nitrogen ≥30 mg/dl (9 mmol/liter) +20

- Sodium <130 mmol/liter +20

- Glucose ≥250 mg/dl (14 mmol/liter) +10

- Hematocrit <30% +10

- Partial pressure of arterial O2 <60mmHg +10

- Pleural effusion +10

Interpretation of Pneumonia Severity Index Class Description of severityClass I Exhibit no severe clinical signs and aged less than 50 yrs old.

This class has a 30-day mortality rate of 0.1%, however, it is not relevant to residents of RACFs.Class II PSI score 1-70

This class has a 30-day mortality rate of 0.6% and are usually treated in the RACF home.Class III PSI score 71-90.

This class has a 30-day mortality rate of 0.9% and should be assessed for appropriateness of IV treatment in the facility or hospital.

Class IV PSI score 91-130. This class has a 30-day mortality rate of 9.3% and is usually treated in hospital.

Class V PSI score >130. This class has a 30-day mortality rate of 29.2% and should be treated in hospital.

List the main pathogens by age group for pneumonia

Page 41: Objectives - Internal Medicine Define diastolic and systolic heart

-listed in the table are the etiologic agents for adults – Strep, Mycoplasma pneumonia, and Haemophilus influenza are the 3 most common etiologic agents

Newborns (age 0-30days)-infections with Strep group B, listeria monocytogenes or gram-negative rods (e.g. E. coli, Klebsiella pneumonia) are a common cause of bacterial pneumonia. These pathogens can be acquired in utero, via aspiration of organisms present in the birth canal, or by postnatal contact with other people or contaminated equipment.-some organisms acquired perinatally may not cause illness until later in infancy – Chlamydia pneumonia, CMV, etc.- Community-acquired viral infections occur in newborns, although less commonly than in older infants – RSV is most common

Infants and Toddlers:- Viruses are the most common cause of pneumonia, accounting for approximately 90% of all lower respiratory infections. RSV is the most common viral pathogen, followed by parainfluenza types 1, 2, and 3 and influenza A or B. RSV infection occurs in the winter and early spring. Parainfluenza type 3 infection occurs in the spring, and types 1 and 2 occur in the fall. Influenza occurs in the winter. Other viruses that cause pneumonia less frequently in infants include adenovirus, enterovirus, rhinovirus, coronavirus, herpesvirus, and cytomegalovirus. A recent addition to this list is human metapneumovirus, which causes an illness similar to RSV and may be responsible for one third to one half of non-RSV bronchiolitis. -Bacterial infections in this age group are uncommon and are attributable to Streptococcus pneumoniae, H influenzae type B (less common in immunized children2 ), or Staphylococcus aureus.Children aged 5- Mycoplasma pneumoniae is the most common cause of community-acquired pneumonia and accounts for 20% of pneumonia cases in the general population, 9-16% of cases in early-school–aged children, 16-21% of cases in older children, and 30-50% of cases in college students and military recruits. C pneumoniae is also fairly common in this age group and presents in a similar fashion.School-aged children and adolescents:- Bacterial pneumonia (10%) is common, and these children are often febrile and appear ill. **In immunosuppressed individuals, opportunistic infections with organisms such as Aspergillus species, Candida species, Pneumocystis species, and cytomegalovirus can occur.Elderly: (in 30-50%, no specific pathogen is found)Colonization of the respiratory tract with potentially pathogenic gram-neg and gram-pos bacteria occurs more often in the elderly, owing in part to such factors as repeated antibiotic therapy, endotracheal intubation, smoking, malnutrition, surgery, and therapy that lowers gastric acidity, thereby raising pH.

S pneumonia (causing pneumococcal pneumonia – most common bacterial cause of community-acquired pneumonia in the elderly (15-50%).

Gram-negative bacilli (Klebsiella, Pseudomonas, enterobacter sp, proteus sp, E. coli. Anaerobic bacteria - cause 20% of community-acquired and 31% of nosocomial cases of pneumonia in the

elderly. Pneumonia caused by anaerobes usually results from aspiration. Elderly patients tend to aspirate

Page 42: Objectives - Internal Medicine Define diastolic and systolic heart

because of conditions associated with aging that alter consciousness, such as sedative use and medical conditions (eg, neurologic disorders, weakness).

Haemophilus Influenza Legionella – increases with age Viruses - influenza and parainfluenza viruses, respiratory syncytial virus, and possibly adenoviruses.

Influenza is the most important cause of pneumonia in the elderly. Its incidence in persons >= 70 is about 4 times that in persons < 40. About 90% of influenza-associated deaths in the USA occur in persons >= 65. Secondary bacterial pneumonia may complicate a course of influenza.

Select the antibiotic treatment of choice for pneumoniaTmt for pneumonia depends on infection conditions:

- Outpatient: o 1st choice: macrolides (erythromycin, azithromycin, clarithromycin) 250-500mg QIDo 2nd choice: doxycyclin

- COPD:o No antibiotic tmt or steroid use in past 3 months:

§ 1st choice: new macrolides (azithro, clarithro) § 2nd choice: doxycyclin

o Recent (within 3 months) Ab/steroid use: § 1st: levofloxacin, gatifloxacin § 2nd: amoxi-clavulanate + macrolide, OR 2nd generation cephalosporin + macrolide

o Microaspiration: § 1st: amoxi-clavulinate +/- macrolide; OR moxifloxacin § 2nd: levofloxacin + clindamycin OR metronidazole

- Nursing home pt:o 1st: levofloxacin/amoxi-clavulinate + macrolideo 2nd: 2nd generation cephalo + macrolide

- Pt on ward:o 1st: levo, gatifloxacin, moxifloxacino 2nd: cefuroxime/cefepime/cefotaxime/ceftriazone + macrolide

- Pt in ICU:o 1st: IV levofloxacin + cefotaxime/ceftriaxone/beta lactamase inhibitoro 2nd: IV macrolide + cefotaxime/ceftriaxone/beta lactamase inhibitoro P. aeruginosa:

§ 1st: cipro + ceftazidime/carbapenem/pip-tazo OR gentamycin/tobramycin/amikacin § 2nd: triple therapy w antipsudomonal beta lactamase inhibitor + aminoglycoside +

macrolide

Describe the complications of pneumonia

o Pleural effusion : an excess amount of fluid in the pleural spaceo Hypoxia : missing oxygeno Empyema : rare complication (person more likely to be younger of drug user), pus in the pleural space or an

effusion with organisms seen on a gram stain or culture – for example when pleural fluid is grossly purulent – positive culture NOT required for diagnosis, results from contiguous spread from lung infection (eg anaerobes) or infection through chest wall (eg trauma or surgery)

Page 43: Objectives - Internal Medicine Define diastolic and systolic heart

o Lung abcess : predisposing factors to this include alcoholism, seizures, poor oral hygiene and previous aspiration, chest CT most sensitive test to detect, increased mortaility with large cavity size, longer duration of symptoms, lower lobe location, association with malignant disease

o ARDS : respiratory failureo Sepsis and toxic shock : bacteremia and septicaemia, systemic infection spreading from the lungso Bacterial resistance : prolonged bacterial infection with resistance to antibioticso Pneumothorax: collection of gas in the pleural space resulting in the collapse of the lung on the affected side,

especially associated post-penumonia with Staph, klebsielle, Pseudomonas and Pneumocystiso Hemoptysis: coughing up blood

Interpret the results of a thoracocentesis for pleural effusion

-Thoracocentesis: fluid aspirated from pleural space; 2 types: exudate and transudate

TRANSUDATES-due to changes in osmotic and hydrostatic forces; usually small, may not require drainage -Ex. CHF, hypoalbuminemia, nephrotic syndrome, malnutrition, cirrhosis, intra-abdominal fluid, ascites, peritoneal dialysis

EXUDATES-due to alterations in vascular permeability; can be observed in inflammatory states, infection, or cancer-Ex. Infection, empyema, primary lung cancer, llymphoma, metastatic camcer, pulmonary embolism or infarct, lupus, rheumatoid arthritis, trauma, hemothorax, myexedema, uremia, asbestosis, lymphedema, drug-induced lupus

Exudate TransudateProtein More than 3g/dl Less than 3g/dlPleural: serum protein ratio More than 0.5 Less than 0.5Lactate dehydrogenase (LDH) Two thirds the upper limit of normalPleural: serum ratio of LDH More than 0.6 Less than 0.6

Establish an appropriate history and physical examination for a patient with hypoxemia

History Symptoms:

o Cough (+ character), dyspnea, SOBOE, sputum production and colour, hemoptysis, fever, chills, infectious contacts, travel, chest trauma

o Orthopnea, PND, leg swelling, exercise tolerance Past medical history

o COPD, asthma, lung cancer, other cancers, CHF, hypertension, neuromuscular disease Medication (+ compliance) including home oxygen Systems review

Physical examination General appearance: level of consciousness, respiratory distress (tachypneic, gasping), use of accessory

respiratory muscles, colour (cyanosis), diaphoresis, pain, oxygen need Vital signs Head and neck: peripheral and central cyanosis, mucous membranes, lymph nodes

Page 44: Objectives - Internal Medicine Define diastolic and systolic heart

Resp: o Inspection: chest deformities (barrel chest), signs trauma, clubbing, capillary refillo Palpation: areas of tenderness, tactile fremitus, o Percussion: areas of hyperresonance, or matte areas o Auscultation: air entry, breath sounds, wheezing, crackles, rhonchis, pectoriloquy,

bronchophony, egophony Cardiovascular:

o Inspection: JVP, peripheral oedema, o Palpation: apical impulse, fremituso Auscultation: S1, S2, extra heart sounds (S3, S4), murmurs

Discuss the differential diagnosis of chronic cough

Common causes:- Upper airway cough syndrome (postnasal drip)- Asthma- GERD

Other causes:- ACE inhibitors- Aspiration- Bronchiectasis- Cystic fibrosis- Chronic interstitial lung disease- Lung/laryngeal cancer- Pertussis- Psychogenic- Restrictive lung disease- TB, atypical mycobacterium, and other chronic lung infections

Will diagnose etiology in most patients:1) Directed history and physical2) Pulmonary function testing3) Sinus radiographs4) Barium esophagography, or 24 hour esophageal pH monitoring

Signs and symptoms of postnasal drip have low predictive value due to poor specificity. Not clinically possible to distinguish simple postnasal drip from postnasal drip due to chronic sinusitis

When considering only patients with cough at least 3 weeks, who are non-smokers and not on ACE inhibitors, with normal or nearly normal and stable chest radiographs:- 99.4% will have GERD, postnasal drip syndrome and/or asthma

Consider pertussis in:- Infants with apnea or severe coughing of any duration- Children or adults with prolonged cough (> 2 weeks), especially if

o Inspiratory whoopo Household exposure to prolonged cough illness

Page 45: Objectives - Internal Medicine Define diastolic and systolic heart

Propose an effective investigation plan to establish a diagnosis of chronic cough

Investigate each of the possible causes

Outline the various treatments available to control chronic coughNon-specific therapy: Cough supressors

Centrally acting: supress central cough centreo Codeine or other opioids (morphine common)o Dextromethorphan

Peripherally acting: benzonatate - anesthesizes stretch receptors in lung Inhaled Glucocorticoids: reduces inflammation; ?results in non-asthmatic/non-eosinophilic patients Ipratropium bromide: anticholinergic, blocks efferent limb of cough reflex and decreases stimulation of

cough receptorsCondition specific:

- Post nasal drip: oral antihistamine-decongestant, avoid allergic stimulus- Asthma: inhaled glucocorticoid + SABA- GERD: PPI + lifestyle- ACE-I: change tmt plan- Chronic bronchitis: stop smoking- Eosinophilic bronchitis: inhaled glucocorticoids

List the main risk factors for venothromboembolic disease

The main risk factors for venous thrombosis are part of Virchow’s Triad: Endothelial damage (leads to decreased inhibition of coagulation and local fibrinolysis, trauma, surgery,

prior DVT) Venous stasis (immobilization, eg post MI, CHF, stroke, post-op, which inhibits clearance and dilution of

coagulation factors) Hypercoagulability ( acquired or inherited)

1. Increases with age2. Surgery (especially ortho)3. Trauma (especially fractures of the spine, pelvis, femur)

Page 46: Objectives - Internal Medicine Define diastolic and systolic heart

4. Neoplasms (especially lung, pancreas, colon, rectum, kidney and prostate cancer)

5. Blood dyscrasias (myeloproliferative disorders)6. Prolonged immobilization (CHF, stroke, MI, leg injury)7. Hormone related (preggers, OCP, HRT, SERMs)8. Antiphospholipid antibody syndrome (APLAS)9. Hyperhomocystenemia10. Heart failure

Idiopathic Malignancies Previous DVT

Describe the usual signs and symptoms of venothromboembolic disease

Venothromboembolic disease encompasses both deep vein thrombosis (DVT) and pulmonary embolism (PE)

DVT:-Symptoms:

Lower extremity: pain and swelling, but may be asymptomatic if restricted to calf Upper extremity: superior vena caval syndrome (facial swelling, blurred vision, dyspnea); thoracic outlet

obstruction can compress brachial plexus and cause unilateral pain and hand weakness-Signs:

Lower extremity: tenderness, erythema, warmth, swelling below site of thrombosis, +/- Homan sign (pain with dorsiflexion), dilated superficial veins, low grade fever

Upper extremity: brachial plexus tenderness in supraclavicular fossa, atrophic hand muscles

PE:-Symptoms (acute):

Sudden onset dyspnea and pleuritic chest pain (sharp pain, worse with breathing and movement) Angina chest pain possible from right ventricular ischemia Hemoptysis from pulmonary infarction Syncope/presyncope from acute right-ventricular failure (cor pulmonale)

-Signs: Tachynpnea Tachycardia Inspiratory crackles Loud P2 Expiratory wheezing Pleural friction rub on auscultation Signs of DVT present in 10-20% of patients Arterial blood gas: +/- hypoxemia, respiratory alkalosis

Discuss the differential diagnosis of venothromboembolic disease

DVT Muscle strain/tear Lymphangitis / lymph obstruction Venous valvular insufficiency Ruptured popliteal cyst

Page 47: Objectives - Internal Medicine Define diastolic and systolic heart

Cellulitis Arterial occlusive disease

PE Cardiac: ACS, aortic stenosis, atrial fibrillation, CHF, MI, pericarditis, Resp: ARDS, asthma, COPD, pneumonia, pneumothorax, TB Other: MSK, acute anemia, shock

Order appropriate investigations for a patient with suspected venothromboembolic disease

1) ECG and CXR useful to look for other causes2) D-dimer

o Only useful if negative in low risk pt3) Duplex scan

o High sensitivity and specificity for proximal clot but only 73% sensitivity for calf DVT

4) CT angiography: high sensitivity and specificity for PE5) V/Q scan: useful when Ct angio not available6) Pulmonary angiography: gold standard but more invasive

Specific investigations for DVT1) D-dimer test only useful to rule out DVT if negative and low clinical suspicion of disease2) Doppler US

o Most useful dx testo Sensitivity and specificity: proximal DVT = 95%, calf DVT = 70%

3) MRI4) Plethysmography5) Venography

o Gold standard, but expensive, invasive and higher risk

Specific investigation for PE (if highly suspicious, go straight to spiral CT)1) D-dimers (products of thrombotic/fibrinolytic process)

o D-dimer results alone do not rule in or out DVT/PEo Need to use in conjunction with leg dopplers, other investigations

2) Spiral CT scan with contrast o Sensitive and specific for PE

3) Venous duplex ultrasound or doppler (high specificity)o With leg symptoms: (+) test can rule in a proximal or distal DVT, (-) test can only rule out a

proximal DVTo Without leg symptoms: (+) test rules in proximal DVT, (-) test does not rule out a DVT (a possible

non-occlusive DVT?)4) ECG (not sensitive or specific)

o sinus tachycardia most common, may see non-specific ST and T waves changeso RV strain, RAD, RBBB S1 Q 3 T3 with large embolus

5) CXRo Frequently normal and often nonspecific (e.g. areas of atelectasis, elevation of a hemidiaphragm,

pleural effusion)

Page 48: Objectives - Internal Medicine Define diastolic and systolic heart

6) V/Q scan (very sensitive but low specificity) o Order scan if: CXR normal/mild abnormalites, no COPDo Avoid if: CXR abnormal or COPD, inpatient, suspect massive PEo If normal, excludes dx of PE, although 60% are nondiagnostic

Establish an appropriate treatment plan for venothromboembolic disease

Treatment of DVT: Goal is to prevent extension of thrombus and PE Initial tx involves therapeutic doses of either unfractionned heparin or LMW heparin. Initial treatment

with an oral anticoagulant alone is unacceptable. Doppler Legs to evaluate extent of thrombus and have a baseline Long-term treatment – oral therapy with vitamin K antagonists (e.g. warfarin) is very effective for long-

term prevention of thrombosis. If first proximal DVT occurs in the context of a transient factor (e.g. surgery, trauma), the risk of recurrence is very low and a limited duration of therapy (3 months) is required. If first DVT occurs in the context of active malignant disease, which is an ongoing risk factor, longterm anticoagulation with LMW heparin has been shown to be more effective than warfarin. See http://www.cmaj.ca/cgi/reprint/175/9/1087 for extensive details on time of anticoagulation required post DVT event – if thrombophilic cause, lifelong anticoagulation recommended.

Addition of systemic thrombolysis leads to earlier patency of the occluded vein; however, it does not affect the rate of PE. There is a definite increase in the risk of hemorrhage so not recommended.

Placement of an inferior vena cava filter in addition to anticoagulation therapy has not been found to prolong survival among patients with DVT. While preventing PE, insertion of a filter increases the risk of recurrent DVT (and is only recommended when anticoagulation is contraindicated).

Treatment of PE: ED: heparin (either unfractionated or LMWH) reduces risk of mortality of PE because it slows or prevents

clot progression and reduces the risk of further embolism. Heparin does nothing to dissolve clot that has developed already, but it is still the single most important treatment that can be provided, because the greatest contribution to the mortality rate is the ongoing embolization of new thrombi. Treatment of DVT and PE: 1 mg/kg SC q12h or 1.5 mg/kg SC qd for 5 d; overlap w/warfarin until INR 2-3

Oxygen should be administered to every patient with suspected PE, even when the arterial PO2 is perfectly normal, because increased alveolar oxygen may help to promote pulmonary vascular dilatation.

IV fluids may help or may hurt the patient who is hypotensive from PE depending on which point on the Starling curve describes the patient's condition

Fibrinolytic therapy has been the standard of care for patients with massive or unstable pulmonary embolism (PE) since the 1970s. Unless overwhelming contraindications are evident, a rapidly acting fibrinolytic agent should be administered immediately to every patient who has suffered hypotension (even if resolved) or is significantly hypoxemic from PE. Fibrinolysis is indicated for any patient with a PE large enough to cause hypotension, even if the hypotension is transient or correctable. As noted above, early fibrinolysis may reduce the mortality rate by 50% for patients who have right ventricular dysfunction due to PE, even if they are hemodynamically stable

Interpret a complete blood count

Normal CBC:- RBC: male 4.32-5.72; female 3.9-5.03

o ↑ in: polycythemia vera, vigorous exercise, high altitude o ↓: anemia

Page 49: Objectives - Internal Medicine Define diastolic and systolic heart

- Hg: male 135-175; female 120-155o ↑: polycythemia vera, tobacco abuse, high altitude, stress, renal cell CA, cardiovascular disease,

dehydration, thalassemia traito ↓: anemia, hemolytic anemia, acute hemorrhage, marrow failure, CRF

- Hematocrit: male 38.8-50%; female 34.9-44.5%o ↑: dehydration, profound diuresis, hemoconcentration (burn, trauma, shock), polycythemia vera,

high altitudeo ↓: anemia, pregnancy

- WBC: 3.5-10.5o ↑:

Neutrophilia: stress, acute infection, tissue injury/inflam, metabolic condition (ARF, eclampsia, ketoacidosis), acute hemorrhage, steroids, lithium, beta agonists

Lymphocytosis, monocytosis Acute leukemia: thrombocytopenia, peripheral smear w immature cells (blasts),

hyperuricemia Chronic leukemia: normal platelet and Hg

o ↓: neutropenia, lymphocytopenia, monocytopenia, eosinopenia- Platelet: 150-450

o ↑: thrombocytosis – CML, polycythemia vera, myelofibrosis, essential thrombocythemiao ↓: thrombocytopenia

Platelet destruction: immune, prosthetic heart valve, TTP, sepsis, DIC, hemorrhage w extensive transfusion

↓ production: hereditary, infiltrative (leukemia, myelofibrosis, neuroblastoma…), suppression of megakaryocytes (alcoholism, radiation, infection by TORCH, meds, aplastic anemia)

Demonstrate an approach to the differential diagnosis of anemia- In general, hemoglobin must be <135

Microcytic (MCV <80) Normocytic (MCV 80-100) Macrocytic (MCV >100)Lead poisoningFe deficiency (Increased loss, decreased intake)ThalassemiaSideroblasticChronic disease

Acute blood lossHemolysisAnemia of chronic disease (cancer, thyroid, infection, inflammation)Aplastic anemiaMDSRenal/kidney failureDrugs

B12FolateEtOH abuseHypothyroidDrugs (methotrexate, chemotherapy)Pregnancy

- Also look at RDW for different populations of RBC and reticulocyte count- B12 and folate are megalobastic- CBC + differential- Personal medical history and presenting symptoms!!!

Demonstrate a detailed history focused on anemia

Page 50: Objectives - Internal Medicine Define diastolic and systolic heart

Obtain details of previous blood tests to establish duration of problem Family history: anemia, jaundice, cholestasis, splenectomy, bleeding disorders and abnormal hemoglobins Blood loss: pregnancies, abortions, menstrual losses Changes in bowel habits (red streaks, tarry stools, etc.) GI history: hemorrhoids, gastritis, peptic ulcers, hiatal hernia (and potential symptoms of these

conditions) Abnormal urine colour (can be present in renal, hepatic and hemolytic disorders) Thorough dietary history (foods patients eats and those patient avoids)

o Meal-by-meal descriptiono Consumption of clay or laundry starch (makes iron less absorbable)

Changes in body weight: may suggest malabsorption or infectious, metabolic or neoplastic disease Unusual symptoms associated with nutritional deficiencies

o Iron: suck or chew ice (pagophagia), dysphagia, brittle fingernails, impotence, fatigue, cramps in calves with exercise/effort

o B-12: early graying of hair, burning sensation of the tongue, decreased proprioceptiono Pernicious anemia: paresthesia, unusual sensations described as paino Folate: sore tongue, cheilosis, symptoms associated with steatorrheao Stools: color, bulk, frequency, odor, oily(?), whether stools sink or float (malabsoption)

Fever (infections, neoplasms, collagen vascular disease) Purpura, ecchymoses, petechia (thrombocytopenia, bleeding disorders) Dark urine associated with physical activity or particular time of day (march hemoglobinuria, paroxysmal

nocturnal hemoglobinuria) Other symptoms that might suggest underlying disease (cardiac, hepatic, renal, chronic infection,

endocrinopathy, malignancy)

Demonstrate the ability to perform a physical examination focused on anemia

VS: BP, HR, RR, Temperature, SatO2

General: cachexia, jaundiceSkin: pallor (lips, buccal mucosa, conjunctiva, palmar creases), koilonychias (brittle, ridged or spoon nails)HEENT: Glossitis, angular cheilosis, abnormal faciesLymphatic system: lymph nodes in head and neck, axilla and groin (size, shape, mobility, consistency, tenderness, warmth, number of enlarged nodes)CVS: signs of cardiac compromise (displaced apex, hyperdynamic precordium, bounding pulsesAbdomen: examine for hepatosplenomegaly, DRE

Discuss the use of appropriate investigations for anemia

Basic investigations:1) Rule out dilutional anemia (low Hb due to ↑ effective circulating volume)2) CBC with differential (pay special attention to MCV and RDW)3) Reticulocyte count4) Blood film/cell morphology

Additionnal investigations:

Page 51: Objectives - Internal Medicine Define diastolic and systolic heart

o Serum iron indices (Fe, TIBC, ferritin, soluble transferring receptor) for Iron deficiencyo Serum B12 and Folate for B12 and Folate deficiencyo Bilirubin, urobilinogen, LDH, Haptoglobulin, for Hemolytic anemia and B12 deficiency

General investigations to R/O secondary anemia- FOB/colonoscopy/endoscopy to R/O acute internal bleed- TSH and free T4 to R/O thyroid diseases- Hepatic function, Albumin, INR to R/O liver disease- Electrolytes, Creatinine, Urea to R/O renal disease

Specific investigationsMICROCYTIC ANEMIA

o Hb electrophorysis for ß-Thalassemia Minor = ↑ HbA2 to 2.5-5% (50% have also ↑HbF but non specific) Major = HbA 0-10% and HbF 90-100%

- DNA analysis using α-gene probe, for α-Thalessemia (electrophoresis not dx)o Bone marrow biopsy (for Iron deficiency – rarely done, and Sideroblastic anemia)

NORMOCYTIC ANEMIAo Hb electrophoresis for Sickle Cell Diseaseo Specific tests for intravascular hemolysis

Schistocytes on blood film Free Hb in serum and methemalbuminemia (heme + albumin) Urinalysis for hemoglobinuria (immediate) and hemosiderinuria (delayed)

o Specific tests for extravascular hemolysis Direct Coombs’ test (direct anti-globulin test)

Indication = Hemolytic disease of newborn, Autoimmune hemolytic anemia, Hemolytic transfusion reaction

Indirect Coomb’s test (indirect antiglobulin test) Indications: cross-matching of recipient serum with donor’s RBC, atypical blood

group, blood group antibodies in pregnant women, Autoimmune hemolytic anemia- Molecular analysis for spectrin gene (in hereditary spherocytosis or elliptocytosis)- G6PD assay for G6PD Deficiency

MACROCYTIC ANEMIAo Schilling test (can distinguish pernicious anemia from other causes)o Bone marrow

Demonstrate an understanding for the treatment of anemiaTreat anemia given the particular type.

Iron deficiency anemia – ferrous sulphate 750-1500 mg PO divided BID or QID for 3-6 months Pernicious anemia – B12 1000 mcg SC/IM qd x 1 week, then qweek x 1 month, then qmonth Folate deficiency – folic acid 1 mg PO qd until hematologic correction; max 5mg/day Hemolytic anemia (immune-mediated) – discontinuation of offending medication + supportive care +

corticosteroid Autoimmune hemolytic anemia – removal of insult or treatment of underlying condition + corticosteroids

+ whole-body plasmapheresis ±splenectomy Anemia of chronic disease – treatment of underlying disease, RBC transfusion, erythropoiesis-stimulating

agents (epoetin alfa)

Page 52: Objectives - Internal Medicine Define diastolic and systolic heart

Severe aplitic anemia (under age 40): matched related donor allogenic stem cell transplantation, immunosuppressive therapy, blood product transfusions, antibiotics

Sickle cell anemia – supportive care + prevention of complications (hydration, etc) + hydroxyurea (reduces the frequency of painful crises and the need for transfusions in patients with recurrent painful crises) + repeated blood transfusions; children with recurrent, severe complications unresponsive to first-line therapies should be considered for bone marrow transplantation.

Sickle cell anemia with vaso-occlusive crisis – analgesia + supportive care and correction of cause + hydration+ antibiotics + blood transfusion

Leukemia – treat based on type – may include chimo and bone marrow transplantation Chronic renal failure (stage 3-4 without uremia) with anemia: ACE inhibitor or ARB plus statins +

additional antihypertensive therapy + education about renal replacement therapy (hemodialysis, peritoneal dialysis, kidney transplant) + recombinant erythropoietin therapy (initiated once the Hb falls to <10g/dL and the patient has signs and symptoms of anemia)

Establish a practical approach to diagnose a bleeding disorderIndications of bleeding disorder:- Hx of excessive bleeding from trivial wound: >15min, spon recurrence 7 days post injury- Spontaneous nose bleed >10min- Unexplained blood in stool- Blood relative with VonWillibrand disease or hemophilia- Heavy menses with clots- Bruises with lumps- Liver/kidney disease, blood or bone marrow disorder, platelet problem- Unexplained anemia requiring Tx, or blood transfusionEvaluation: if Hx gives indication of bleeding disorderINR, PTT, platelet count/morphology- Normal PT, PTT, and Platelet Count/morphology

o Obtain Platelet Function Closure Time: quantifies platelet function w 2 different tests: Epinephrine and ADP → abnormal in VWD, thrombocytopenia, anemia w low hematocrit, ASA

PFCT Aggregation w epi membrane only: drug induced platelet dysfunction (ASA) PFCT Aggregation occurs w both membranes: VWD

- Partial Thromboblastin Time (PTT) abnormalityo PTT corrects with a PTT Mixing Study (mixed w normal blood)

Obtain Factor VIII, Factor IX, and Factor XI assays→ Consider VW testing if low Factor VIII

o PTT does not correct with a PTT Mixing Study (mixed w normal blood): obtain Lupus anticoag, FVIII inhibitor

- ProTime (PT) or INR abnormalo INR corrects with Vit K Supplement → Tx w Vit K, assess for malnutrition and malabsorption causeso NR does not correct w Vit K Supplementation → get FVII assay

- ProTime (PT/INR) and PTT Abnormal → advanced liver disease, DIC, anticoag use- Platelet abnormality → VWD, Glanzmann’s Thrombasthenia (missing GPIIb/IIIa), Bernard-Soulier disease,

storage pool disease, drug induced platelet dysfunction, thrombocytopeniaPrimary Secondary

Surface cuts Excessive, prolonged bleeding Normal/slightly prolonged bleedingOnset after injury

Immediate Delayed

Site of bleeding Superficial: mucosal (nose, gums, GI tract), skin

Deep: joints, muscles, GI, GU, excessive post-traumatic

Page 53: Objectives - Internal Medicine Define diastolic and systolic heart

Lesions Petechiae, ecchymoses Hemarthroses, hematomasLife-threatening Intracranial, retroperitoneal

Order the relevant laboratory tests for a bleeding disorder

The lab tests are done as general screening tests and to define specific platelet or clotting factor abnormalities

Tests:o Platelet count: look for small or large platelets, abnormalities in size, numbero Bleeding time (BT): measure of interaction of platelets with the blood vessel wall, prolonged bleeding time

may occur in thrombocytopenia, qualitative platelet abnormalities (ex: uremia), vWD, severe fibrinogen deficiency

o Prothrombin time (INR) : the production of fibrin via the extrinsic pathway and the final common pathway requires tissue thromboplastin (tissue factor), factor VII (ext). factors X, V, prothrombin (factor II) and fibrinogen – measures of oral anticoagulant activity

o Activated partial thromboplastin time (aPTT): measures intrinsic and common pathways of coagulation, used to measure the degree of heparin anticoagulation

o Thrombin time (TT): measures the time it takes for a clot to form in the plasma when a standard amount of thrombin is added to a sample

o Peripheral blood smear: essential for patients with low platelet counts to exclude the presence of pseudothrombocytopenia due to in vitro platelet agglutination in the presence of EDTA, use of other alternative anticoagulants (citrate or heparin) may circumvent this technical problem

o Platelet aggregation in response to ADP, epinephrine, collagen and ristocetino Platelet release assayso Coagulation factor assayso Assessment of factor XIII activity via clot solubility testingo Measure of fibrin split products and D-dimer levelso Less common: alpha-2-antiplasmin activity, euglobin clot lysis time, tissue plasminogen activator and

plasminogen activator inhibitor-1 antigens

Describe the indications for blood transfusions

1) Whole blood transfusionsa. Very few indications, except:

i. Cardiac surgery or situations of massive hemorrhage where more than 10 units of bloods are required in less than 24 hours

ii. Rapid, life-threatening therapy when red-blood cell component therapy is not available2) Platelet Transfusion therapy

a. Indication: to stop or prevent bleedingi. Prophylactic platelet transfusions for thrombocytopenia

1. <10,000 mcL: in non-bleeding patients: appropriate for patients undergoing chemotherapy and those with other conditions, without fever, sepsis and GI bleeding

2. <20,000mcL: for complicating circumstances or if patients are being managed on out-patient basis

3. <50,000 mcL: for patients undergoing invasive surgical procedures

Page 54: Objectives - Internal Medicine Define diastolic and systolic heart

4. <100,000: for patients undergoing neuro or ophthalmologic procedures due to catastrophic nature of bleeding in these locations

ii. Platelet transfusion for the acutely bleeding patient1. Platelet counts >50,000 are a reasonable goal for most cases2. >100,000 is reasonable for neurologic bleeds3. Patients on aspirin or other antiplatelet drug may benefit from transfusion

regardless of baseline4. *NB: in acute bleeding situations, most patients receive fluids and other blood

products to increase blood volume. This might create a ‘dilutional coagulopathy’. When platelet count is <50% baseline, infuse platelets liberally to replenish

3) Plasma and Coagulation Factor Therapya. Fresh Frozen plasma (contains all coagulation factors necessary to maintain homeostasis)

i. Liver failureii. Deficiencies in factors II, V, X, XI

iii. Reversal of warfarin therapy before invasive procedures or with bleedingb. Prothrombin Complex Coagulate (when patients can’t tolerate the large volumes of FFP) – II, VII,

IX, Xc. Virally inactivated factor VIII and IX for hemophilia A or B

4) Cryoprecipitate Transfusion Therapy (cryoprecipitate antihemophiliac factor) a. Description: produced from thawing FFP and removing precipitated portion; contains high levels

of fibrinogen, fibronectin, VIII, vonWillebrand factor and XIII; advantage: small volumeb. Uses:

i. Remplacement of fibrinogen in DICii. Patients with hypofibrinogenemia or dysfibrinogenemia

iii. Isolated comsumption of XIII in DIC or isolated XIII deficiency

Describe the potential complications associated with blood transfusions

ACUTE – Immune

Acute Hemolytic Transfusion Reactions ABO incompatibility resulting in intravascular hemolysis secondary to complement activation Most commonly due to incorrect patient identification Occurs immediately after transfusion Risk per unit of blood is < 1/250,000 Presents with fever, chills, hypotension, back/flank pain, dyspnea, hemoglobinuria Acute renal failure (<24hrs) and DIC Treatment

o Stop transfusiono Maintain BP with vigorous IV fluids +/- inotropeso Maintain urine output with diuretics, crystalloids, dopamine

Febrile Nonhemolytic Transfution Reactions Due to alloantibodies to WBC, platelets or other donor plasma antigens and release of cytokines from

blood product cells

Page 55: Objectives - Internal Medicine Define diastolic and systolic heart

Occurs within 0-6hrs of transfusion Risk per unit of blood is 1/100 Presents with fever +/- rigors, facial flushing, headache, myalgia, hypotension Treatment

o Rule out fever due to hemolytic reaction or infectiono If fever < 38°C, continue with transfusion but decrease rate and give antipyreticso If fever > 38°C, stop transfusion, give antipyretics and anti-histamine

Allergic Nonhemolytic Due to alloantibodies (IgE) to proteins in donor plasma which result in mast cell activation and release of

histamine Occurs mainly in those with history of multiple transfusion, or multiparous women Risk per unit of blood is 1/100 Presents mainly as urticaria and occasionally with fever Can present as anaphylactoid reaction with bronchospasm, laryngeal edema, and hypotension but this

occurs mainly in IgA deficient patients that have anti-IgA antibodies Treatment

o Mild: slow transfusion rate and give diphenhydramineo Moderate to severe: stop transfusion, give IV dyphenhydramine, steroids, epinephrine, IV fluids

and bronchodilators

Transfusion-Related Acute Lung Injury New-onset acute lung injury that occurs during transfusion or within 6 hrs of the completion of

transfusiono Insidious, acute onset of pulmonary insufficiencyo Profound hypoxemia (PaO2/FiO2 < 300 mmHg)o Bilateral pulmonary edema on CXRo Pulmonary artery wedge pressure<18mmHgo No clinical evidence of left atrial hypertension

Pathogenesis uncertain; perhaps due to binding of donor antibodies to WBC of recipient and release of mediators that increase capillary permeability in the lungs

Typically occurs 2-4hrs post transfusion and resolves in 24-72 hrs Risk per unit of blood is 1/5000

o Is currently the leading cause of transfusion-related morbidity and mortality Treatment: supportive therapy (oxygen)

ACUTE – Nonimmune

Bacterial Infection Gram +: S. aureus, S. epidermis, Bacillus cereus Gram - : Klebsiella, Serratia, Pseudomonas, Yersinia Overall risk is 1/100,000 for RBC and 1/10,000 for platelets Never store blood > 4 hrs after a bag has left blood bank Treatment: stop transfusion, blood cultures, IV Abx, fluids

Circulatory Overload Due to impaired cardiac function, and/or excessive rapid transfusion Presents as dyspnea, orthopnea, hypotension, tachycardia, crackles at base of lung, and increased venous

pressure Incidence is 1/700 Treatment: transfuse at lower rate, give diuretics and oxygen

Page 56: Objectives - Internal Medicine Define diastolic and systolic heart

Hyperkalemia Due to K release from stored RBC Risk increases with storage time and if blood is irradiated Decreased risk if given fresh blood Occurs in 5% of massively transfused patients Treatment:

o K < 6.5 : furosemide, Kayexalateo K 6.5 - 7.0 : add insulin to aboveo K > 7.0 : add Ca gluconate to above (to protect the heart)

Citrate Toxicity Occurs with massive transfusion in patients with liver disease – patients are unable to clear citrate from

blood Citrate binds to Ca and causes signs and symptoms of hypocalcemia Treatment: IV calcium gluconate (10mL of 10%) for every 2 units of blood

Dilutional Coagulopathy Occurs with massive transfusion (> 10 units) pRBC contains no clotting factors, fibrinogen, cryoprecipitate, or platelets treatment: FFP, platelets, and cryoprecipitate

DELAYED – Immune

Delayed Hemolytic due to alloantibodies to minor antigens such as Rh, Kell, Duff, and Kidd level of antibody at time of transfusion is too low to cause hemolysis; later the level of antibody increases

due to secondary stimulus and causes extravascular hemolysis occurs 5-7 days after transfusion presents as anemia and mild jaundice treatment: no specific treatment required; important to note for future transfusion

Transfusion-Associated Graft Versus Host Disease transfused T-lymphocytes recognize and react against host (patient) occurs 4-30 days following transfusion most patients already have severely impaired immune systems (ex. Hodgkins or leukemia) presents as fever, diarrhea, liver function abnormalities, and pancytopenia can be prevented by giving irradiated blood products

DELAYED-Nonimmune

Iron Overload due to repeated transfusions over long period of time (ex. ß-thalassemia major) can cause secondary hemochromatosis treatment: iron chelators after transfusion

Viral Infection Risk HBV < 1/82,000 HTLV < 1/1,000,000 HCV < 1/ 2,800,000 HIV < 1/ 4,000,000 Other infections include EBV, CMV, WNV

Page 57: Objectives - Internal Medicine Define diastolic and systolic heart

Demonstrate an approach to the diagnosis of renal failure (acute and chronic) based on prerenal, renal and postrenal classification

1) ACUTE RENAL FAILURE = abrupt ↓ in renal function leading to ↑ nitrogenous waste products

** 2 most common causes of acute kidney injury in hospitalized pt:1) Azotemia2) ATN

Prerenal failure can lead to ATN

Clinical features:- Azotemia: ↑BUN, ↑Cr- Abnormal urine: anuria, oliguria, polyuria

Page 58: Objectives - Internal Medicine Define diastolic and systolic heart

Clues leading to etiologyPre-renal Renal Post renal

↓BP, ↑HR + orthostatic ∆↑[urea] >> ↑[Cr]

Clinical contextUrinalysis positive for casts:- ATN = pigmented granular- AIN = WBC- GN = RBC

Known solitary kidneyOlder manRecent retroperitoneal surgeryAnuriaPalpable bladderU/S shows hydronephrosis

** To differentiate pre-renal from ATNUrinalysis = N

Urine [Na] < 20Urine [Cr]/ [Na] > 40Urine osm > 500 FeNa < 1%

Urinalysis = RBC, pigmented granular castsUrine [Na] > 40 mEq/lUrine [Cr]/ [Na] < 20Urine osm < 350 mOsm/kgH2OFeNa > 1%

CHRONIC RENAL FAILURE = abnormal markers (Cr, urea)- GFR < 10 ml/min for > 3 months OR- Kidney pathology seen on biopsy OR- ↓ renal size on U/S (kidneys < 9 cm)

Stage of CRFStage GFR (ml/min/1.73 m2)

1 ≥ 902 60-893a (moderate) 45-593b 30-444 (severe) 15-29

Page 59: Objectives - Internal Medicine Define diastolic and systolic heart

5 (end stage/dialysis)

< 15

Most common etiology of CRF: diabetes, HTN, glomerulonephritis

Describe the signs and symptoms of renal failure

Signs and symptoms depend on acuity of onset severity of insult, adaptation to nephron loss/dysfunction, treatment of reversible disease process

1) Volume Overload Due to increase in total body Na Weight gain, HTN, pulmonary or peripheral edema

2) Electrolyte AbnormalitiesHigh

K (decreased renal excretion, increased tissue breakdown) PO4 (decreased renal excretion, increased tissue breakdown) Ca (rare; happens during recovery phase after rhabdomyolysis-induced ARF)

Low Na (failure to excrete excessive water intake) Ca (decreased vitamin Dactivation, hyperphosphatemia, hypoalbuminemia) Academia (especially with sepsis or severe heart failure)

Uremia – N/V + diarrhea which may lead to dehydration, weight loss, nocturnal urination, foamy or bubbly urine, polyuria with pale urine, less frequent urination with dark urine, hematuria

Hyperphosphatemia – itching, bone damage, muscle cramps Hyperkalemia – abnormal heart rhythms, muscle paralysis Polycystic kidney disease – pain in back or side Decrease in EPO synthesis – *anemia is associated with CKD due to EPO when GFR declines to

<50mL/min/1.73m2 feeling tired or weak, memory problems, difficulty concentrating, anemia, low BP

Page 60: Objectives - Internal Medicine Define diastolic and systolic heart

Edema – periorbital and peripheral edema develop as a result of salt and water retention as the GFR declines

Pruritis, nausea, anorexia – due to accumulation of toxic waste products (i.e. urea)

List the most useful supplementary tests for the diagnosis of renal disease- Blood: BUN (byproduct of protein breakdown) and creatinine (byproduct of normal muscle function) - ↑ in

kidney disease- Urine:

o Protein determination: quantifies protein; reported as protein to creat ratio, >100 is abnormalo Microalbumine: microalbumin/creat ratio >30mcg of albumin per gram of creat = microalbumineria =

early sign of kidney diseaseo Dipstick: detects protein, blood, glucose, WBCo Sediment examination: WBC, RBC, bacteria, yeast, crystals, renal tubular cells

- GFR: measures plasma filtration, ↓ with chronic kidney diseaseo Creatinine clearance: 24h urine collection + blood sampleo Cockcroft-Gault formulao eGFR by MDRD equation: estimates GFR based on age, gender, ethnicity, serum creat

- Ultrasound: determines size of kidneys, cysts, masses, kidney stones, quantify scarring, urine flow blocage in kidney/ureters/bladder

- IVP (IV pyelogram): X-rays of kidneys/ureters/bladder w injection of IV contrastSize, shape of kidneys, kidney stones, cysts/tumorsNOT done if function ↓ bc of contrast

- CAT: detects kidney stones, blockage, cysts, masses- MRI: gadolinium contrast not used in moderate to severe renal impairment bc nephrogenic systemic fibrosis- Biopsy via ultrasound or CT

Interpret urinalysis and urine sediment

Urinalysis: Specific gravity

a. Ratio of the mass of equal volumes of urine/H2Ob. Range is 1.001 to 1.030c. Values < 1.010 reflect dilute urine, values > 1.020 reflected concentrated urined. Value usually 1.010 in the end stage renal disease

pH (urine pH is normally between 4.5 – 7; if persistently alkaline, consider…)a. renal tubular acidosisb. UTI with urease producing bacteria (ex: Proteus)

Glucosea. Freely filtered at glomerulus and reabsorbed in proximal tubuleb. Causes of glucosuria: 1) hyperglycemia >9-11 mmol/L leads to filtration that exceeds tubular

resorption capacity, 2) increased GFR (ex: pregnancy), 3) proximal tubule dysfunction (Fanconi’s) Protein

a. dipstick only detects albumin…other proteins may be missedb. microalbuminuria is not detected by dipstickc. sulfosalicylic acid detects all protein in urine by precipitationd. gold-standard is the 24 hour urine collection for total protein

leukocyte esterasea. enzyme found in WBC and detected by dipstick

Page 61: Objectives - Internal Medicine Define diastolic and systolic heart

b. presence of WBCs indicates infection or inflammationc. +ve dipstick for leukocyte esterase and nitrites is 94% specific for diagnosing a UTI

Nitritesa. Nitrates in urine are converted by bacteria to nitritesb. High specificity, low sensitivity for UTI

Ketonesa. Positive in alcoholic/diabetic ketoacidosis, prolonged starvation, fasting

Hemoglobina. Positive in hemoglobinuri (hemolysis), myoglobinuria (rhabdomyolysis) and true hematuria (RBCs

seen on true microscopy)

Urine sedimentCells Casts Crystals

Erythrocytes – normal 2-3 RBC/HPF, hematuria = more, dysmorphic RBCs and casts suggests glomerular bleeding, isomorphic RBCs and no casts indicates extraglomerular bleeding (ex: bladder Ca)Leukocytes – normal = up to 3 WBCs/HPF, more = pyuria, inflammation or infection, is sterile pyuria could be renal TB, viral, prostatitis, calculi, etc.Eosinophils – consider allergic interstitial nephritis, atheroembolic diseaseOval fat bodies – cells filled with lipid droplets, seen in heavy proteinuria

Cylindrical structures formed by intratubular precipitation of Tamm-Horsfall mucoprotein; cells may be trapped in the matrix-hyaline casts: physiologic (concentrated urine, fever, exercice)-RBC casts: glomerural bleeding-WBC casts: pyelonephritis, interstitial nephritis-Pigmented casts: ATN, glomerulonephritis-Fatty casts: Proteinuria of >3.5g/day

Uric acid – consider acid urine, hyperuricosuria (ex: gout)Calcium phosphate – alkaline urineCalcium oxalate – consider hyperoxaluria, ethylene glycol poisoningSulfer – sulfa containing antibiotics

Page 62: Objectives - Internal Medicine Define diastolic and systolic heart

Etiology Urinalysis Urine SedimentPre-RenalDecreased Effective arterial volume; Renal vasoconstriction; Large vessel (i.e. Renal artery stenosis, thrombosis)

Bland Transparent hyaline casts

IntrinsicATN +/- RBCs and protein Pigmented granular “muddy”

castsAIN (Allergic, Infectious, Infiltrative) WBCs +/- RBCs WBC casts; + eos in abx; +

lymphocytes in NSAIDsSmall veseel (i.e. cholesterol emboli, DIC) +/- RBCs + eos in cholesterol emboliGN RBCs RBC Casts

Post-RenalBladder neck; Ureteral; Tubular Bland; +/- RBCs if stones

Discuss in detail the role of the renin-angiotensin-aldosterone system

RAAS Major regulator of aldosterone secretion Renin juxtaglomerular cells secrete renin in response to:

o Decrease in circulating volume AND/ORo A reduction in renal perfusion pressure

Renin is the rate-limiting enzyme that cleaves angiotensin (made by the liver) to angiotensin I Angiotensin I is converted to angiotensin II by ACE (angiotensin converting enzyme) in the lungs and other

tissues Angiotensin II is a potent vasopressor (vessel constrictor) and stimulates aldosterone production (but not

cortisol production) Angiotensin II is the predominant regulator of aldosterone secretion, but secretion is also influenced by

plasma potassium concentration, plasma volume and ACTH levels

Page 63: Objectives - Internal Medicine Define diastolic and systolic heart

Describe the advantages and disadvantages of angiotensin-converting enzyme inhibitors (ACE inhibitors) and angiotensin receptor blockers (ARBs)

Examples Mechanism Side Effects Contraindications

ACE inhibitors

EnalaprilBenazeprilCaptoprilFosinoprilLisinopril

- Suppression of sympathetic and renin-angiotensin systems by inhibiting formation AII, increases sodium elimination-Inhibits degradation of bradykinin

- Reversible dry cough, hyperkalemia, azotemia- Hypotension- Renal dysfunction- Cardiogenic shock- Second-third degree heart block

- Hypotension- Significant renal failure- Bilateral renal stenosis- Pregnancy

Angiotensin II Receptor Blocker

CandesartanEprosartanIrbesartanLosartan

- Blocks AII formed by other pathways- vasodilation- Used if ACE inhibitors are not tolerated (i.e. due to cough)

- Similar to ACE inhibitors (renal function, serum K and BP) but without dry cough

- Pregnancy

Describe the main treatments available for renal failure

ARF CRF1) Preliminary measures Tmt focuses on avoidance of risk factors (listed above)

Page 64: Objectives - Internal Medicine Define diastolic and systolic heart

- Pre-renal = correct pre-renal factorso ↑ vol/cardiac performance

- Renal o Exclude reversible causes (d/c nephrotoxic

drugs, optimise lytes, hold ACEI/ARB- Post-renal

o Consider obstruction: stones, strictures, neuropathy

o Treat with foley, indwelling bladder cath, nephrostomy, stenting

2) Treat complications- Fluid overload

o NaCl restrictiono High dose loop diuretics

- Hyperkalemiao Dietary K restriction if mildo Calcium guconate + insulin + glucose +

bicarbonate if severe and/or accompanied by ECG ∆

- Adjust dosage of meds cleared by kidney3) Definitive therapy depends on etiology4) Consider dialysis- Pt suffering from oliguric RF and not expected to

recover promptly- Severe acidosis, hyperkalemia or volume overload

refractory to tmt- Drug or alcohol intoxications (methanol, ethylene

glycol, or salicylates)- Uremic pericarditis- Encephalopathy

** Renal transplant is NOT a therapy for ARF

1) Diet restriction- Protein restriction with adequate caloric intake- K, Na, H2O, PO4 restrictions- Avoid antacids (extradite Mg)2) HTN- ACEI/ARBs (target BP < 130/80)- Diuretics

o loop diuretics, thiazides less effective if GFR < 30 mL/min

3) Anemia - Erythropoietic hormones (epoetin)- Correct Iron deficiency if present4) Renal osteodystrophy and secondary

hyperparathyroidism- Egocalciferol ou Cholecalciferol (25-OH vitamin D)

o To replenish vit D stores- Dietary PO4 binders (Tums, sevelamer)- Active vitamin D (1,25-dihydroxyvitamin D)

o To suppress PTH if serum PTH elevated- Only in dialysis pt: cinacalcet to suppress PTH

release5) Metabolic acidosis- Sodium bicarbonate (do not use citrate)6) Preparation for renal replacement therapy- Hemodialysis, peritoneal dialysis, and eligibility for

renal transplantation

List the criteria for emergency dialysisIndications for Dialysis in chronic renal failure:

Volume overload unresponsive to medication Hyperkalemia unresponsive to medication Severe metabolic acidosis unresponsive to medication Neurologic signs and symptoms of uremia (encephalopathy, neuropathy, seizures) Uremic pericarditis Refractory accelerated HTN Clinically significant bleeding diathesis Persistent severe nausea and vomiting Plasma Cr>1060 µmol/L or BUN>36mmol/L

Relative Indications: Anorexia Decreased cognitive functioning Profound fatigue and weakness Severe anemia unresponsive to EPO

Page 65: Objectives - Internal Medicine Define diastolic and systolic heart

Persistent severe pruritis Restless leg syndrome

Define nephrotic and nephritic syndromesAcute nephritic syndrome:- 1-2g/day of proteinuria, hematuria w RBC casts, pyuria, HTN, fluid retention- Inflammatory damage to glomeruli: ↓ GFR → ↑ serum creat, uremic symptoms with salt and water retention

→ edema, HTNSubtypes:- Type I: idiopathic, subacute bacterial endocarditis, systemic lupus, Hep C +/- cryoglobulinemia, mixed

cryoglobulinemia, Hep B, CA (lung, breast, ovary)- Type II: idiopathic, C3 nephritic factor-associated, partial lipodystrohpy- Type III: idiopathic, complement receptor deficiency

Nephrotic syndrome: - Pts usually present with edema, frothy urine (high protein content)- Heavy proteinuria (>3g/day), HTN, hypercholesterolemia, hypoalbuminemia, adema/anasarca, microscopic

hematuria- If left undx or untreated, can lead to ↓ GFR and renal failure - Subtypes: minimal change disease (most common in children), focal segmental glomerulosclerosis,

membranous glomerulonephritis (30% of nephrotic syndromes – most common in adults), membranous glomerulonephritis, diabetic neuropathy, glomerular deposition diseases (light chain deposition, renal amyloidosis, fibrillary-immunotactoid glomerulopathy, Fabry’s disease – X-linked genetic)

- Dx: biopsy – unless obvious diabetic nephropathy- Complications:

o ↑ risk for ischemic heart disease because prothrombotic state (antithrombin III, protein S and C lost in urine; hyperprotenemia causes ↑ liver synthesis of fibrinogen), HTN, hyperlipidemia

o ↑ risk of infection: IgG and other immune proteins lost in urine

Identify the main causes of electrolyte and acid-base disorders

Overall electrolyte imbalance: Decreased intake, Increased excretion, Abnormal distribution/metabolismDDx:

Infectious: vomiting/diarrheaInflammatory: renal abnormalitiesDrugs: diuretics, poison, drug overdosesNeoplasm: ectopic secretion of hormones, paraneoplastic complication (hypercalcemia – bone mets)Endocrine/Metabolic: diabetes, parathyroid, thyroid, etcSystemic disease: CHF, cirrhosisFluid loss: blood, third spacing, GI losses (diarrhea/vomiting)

Hyponatremia Hypernatremia Dehydration: volume loss via

bleed/vomiting/diarrhea, third spacing (peritonitis, burns)

Drugs: Diuretics Endocrine: SiADH, adrenal insufficiency,

hypothyroidism

Water loss (hemoconcentration) – diarrhea, GI, lactulose, sweat

Diabetes Insipidus Iatrogenic: hypertonic saline infusion Cushing’s syndrome, hyperaldosteronism

Page 66: Objectives - Internal Medicine Define diastolic and systolic heart

CHF, nephritic syndrome, cirrhosis, GI enteropathy

Page 67: Objectives - Internal Medicine Define diastolic and systolic heart

Hypokalemia:

Hyperkalemia DDx:

Hypocalcemia: Hypoalbuminemia – cirrhosis, nephrosis, malnutrition, burns, chornic illness, sepsis, hemochromatosis, pregnancyPrimary hypoparathyroidism: DiGeorge syndrome, polyglandular autoimmune diseaseIatrogenic hypoparathyroidism: post-thyroidectomy

Page 68: Objectives - Internal Medicine Define diastolic and systolic heart

Hypomagnesium – chonic hypomagnesium impairs PTH secretion (usually low Mg stimulates PTH) Hypercalcemia:Primary hyperparathyroidism (major cause) – adenoma, hyperplasia, carcinoma, Secondary hyperparathyroidismFHH: familial hypocalciuric hypercalcemiaDrug-induced: lithium

Hypomagnesium: Diuretic therapyAlcohol abuseExcessive calcium or vit D intake, IV fluids or parenteral nutrition without Mg replacementGI Losses: diarrhea, bowel fistula, acute pancreatitisEndocrine: diabetes mellitus, hyperaldosteronism, hyperthyroidism, hyperparathyroidismDrugs: diuretics, beta-agonists, aminoglycosides, cancer chemotherapy, theophylline

Hypermagnesium:Renal failureLithium therapyExcessive magnesium intake: antacids, laxatives, cathartics, excessive parenteral MgImpaired elimination: anticholinergics, narcotics, chronic constipation, bowel obstructionRhabdomyolysisEndocrine: adrenal insufficiency, hyperparathyroidism, hypothyroidism

Alkalosis:Metabolic alkalosis:

GI loss (vomiting/NG tube), renal lossExogenous alkali (oral or parenteral)Diuretics: decreased bicarb excretion, decreased ECF volumePosthypercapneia: renal compensation for respiratory acidosis, retention of bicarb

Respiratory alkalosis: Hypoxemia: pulmonary disease (pneumonia, edema, PE, interstitial fibrosis), severe anemia, heart failure, Hyperventilation: hepatic failure, gram –ve sepsis, CNS disorders, drugs (ASA, theophylline,

catecholamines, psychotropics), pregnancy, anxiety, pain

Acidosis: Can also be Respiratory Acidosis:

Hypoventilation: central apnea, drugs, trauma, stroke, supplemental O2 in chronic CO2 retainers (COPD patients)

Obstruction Parenchymal disease: COPD, pneumothorax, pulmonary edema, pneumonia, ARDS Neuromuscular disorders: myasthenia gravis, Guillain-Barré syndrome, poliomyelitis, muscular

dystrophies, ALS, myopathies, chest wall disease

Anion Gap Metabolic Acidosis: MUDPILES CATMethanolUremiaDiabetic ketoacidosis (or alcoholic acidosis) ParaldehydeIsoniazid or ironLactateEthylene glycolSalicylates

Page 69: Objectives - Internal Medicine Define diastolic and systolic heart

Carbon monoxide, cyanideAlcoholic ketoacidosisToluene

Non-anion gap Acidosis: HARDUPHyperalimentationAcetazolamideRenal Tubule AcidosisDiarrheaUreteroenteric fistulaPancreaticoduodenal fistula

Interpret an anion gap

-normal concentration of HCO3 = 24mEq/L-normal pCO2 = 40mmHg-each acid/base disorder has an appropriate compensation

1) Evaluation of compensation

2) Calculate plasma anion gapa. AG = Na – (HCO3 + Cl)

i. Range = 10-14, baseline = 12ii. Can be altered by plasma albumin level: for every 10g/L fall in albumin, lower baseline

by 33) Identify the primary acid-base disturbance and assess whether a simple or mixed disturbance is present

a. Increase in AG < decrease in HCO3: co-existing non-anion gap metabolic acidosisb. Increase in AG > decrease in HCO3: co-existing metabolic alkalosis

4) Measure serum electrolytes and lab data5) Calculate osmolar gap (to help with ddx of metabolic acidosis: normal or wide anion gap?)

a. Osmolar gap = measured osmolality – calculated osmolalityi. Calculated osmolality (mmol/L) = (2 x Na) + urea + glucose

ii. If OG < 10: normal OG; If OG > 10: consider wide anion gap causes of metabolic acidosis

Causes of Metabolic Acidosis

Page 70: Objectives - Internal Medicine Define diastolic and systolic heart

1) Normal anion gap - examplesa. Bicarbonate losses, small bowel drainage, diarrhea, renal tubular acidosis, aldosterone

deficiency, Addison disease, diabetes mellitus (recovery phase) + others

2) Wide anion gap – examplesa. Reduced excretion of acids, renal failure, overproduction of acids, ketoacidosis, diabetic,

alcoholic, starvation + others

List the main causes of metabolic acidosis

Causes of high-anion gap metabolic acidosis

Causes of non-anion gap acidosis

Lactic acidosis Ketoacidosis

o Diabetico Alcoholico Starvation

Toxinso Ethylene glycolo Methanolo Salicylates

Renal failure (acute and chronic)

GI bicarb losso Diarrheao External pancreatic/small bowel drainageo Ureterosigmoidostomy, jejuna/ileal loopo Drugs

Calcium chloride (acidifying) Magnesium sulfate (diarrhea) Cholestyramine (bile acid diarrhea)

Renal acidosiso Hypokalemia

Proximal RTA Distal RTA

o Hyperkalemia Generalized distal nephron dysfunction

Page 71: Objectives - Internal Medicine Define diastolic and systolic heart

(mineralocorticoid deficiency/resistance, decreased Na+ delivery to distal nephron, tubulointerstitial disease, ammonium excretion defect)

Drug-induced hyperkalemia (w renal insufficiency)o K+ sparing diuretics (amiloride, triamterene,

spironolactone)o Trimethoprimo Pentamidineo ACE inhibitors and ARBso NSAIDso Cyclosporine

Other o Acid loads (ammonium chloride,

hyperalimentation)o Loss of potential bicarb: ketosis w ketone

excretiono Expansion acidosis (rapid saline administration)o Hippurateo Cation exchange resins

RTA = renal tube acidosis

Describe the physical signs and symptoms of acidosis and sodium and potassium imbalances

Metabbolic acidosis- Most signs and symptoms are caused by the underlying disease or condition that is causing the metabolic

acidosis- Usually:

o Tachypneao Confusion or lethargy o Shock or death (severe)

Na imbalanceHYPONATREMIAS&S depend on degree of hyponatremia and most importantly, velocity of progression from onset

- Acute hyponatremia (<24-48 hr) more likely to be symptomatic- Chronic hyponatremia (>24-48 hr) less likely to be symptomatic due to adaptation- Neurologic symptoms predominate (secondary to cerebral edema)

o Headacheo N/V, anorexiao Malaise/lethargy/weaknesso Muscle crampo Somnolence/disorientationo Personality changeso ↓ reflexeso ↓ LOC

- Complicationso Seizureo Coma

Page 72: Objectives - Internal Medicine Define diastolic and systolic heart

o Respiratory arresto Premanent brain damageo Brainstem herniationo Deatho Risk of brain cell shrinkage with rapid correction of hyponatremia

HYPERNATREMIAAcute hypernatremia more likely to be symptomatic

- ± polyuria, thirst, signs of hypovolemia- S&S due to brain cell shrinkage

o Altered mental statuso Weaknesso Neuromuscular irritabilityo Focal neurologic deficito Seizure/coma/death

- Complications:o ↑ risk of vascular rupture resulting in intracranial hemorrhageo Rapid correction may lead to cerebral edema due to ongoing brain hyperosmolarity

K imbalanceHYPOKALEMIA

- Usually asymptomatic (particularly when mild)o N/Vo Fatigue/generalized weaknesso Myalgia/muscle crampso Constipation

- If severe:o Arrhythmiaso Muscle necrosiso Paralysis with eventual respiratory impairment (rare)

- ECG ∆ = more predictive of clinical picture than K levelso U waves (most important)o Flattened/inverted To ST depression, ↑ QT intervalo If severe: ↑ PR, wide QRS, arrhythmias

HYPERKALEMIA- Usually asymptomatic but may develop:

o N/Vo Palpitationso Muscle weakness/muscle stiffness/paresthesia/areflexiao Ascending paralysiso Hypoventilation

- Impaired renal ammoniagenesis and metabolic acidosis- ECG ∆

o Peaked and narrow To ↓ amplitude and eventual loss of Po ↑PR interval, wide QRS and eventual merging with T waveo AV blocko Ventricular fibrillation, asystole

Page 73: Objectives - Internal Medicine Define diastolic and systolic heart

List the most useful additional tests for the diagnosis of electrolyte disordersThis objective is unclear as the most useful tests for electrolyte disturbances would obviously be to test for the lytes themselves. Perhaps they were asking for additional manifestations and presentations of electrolyte disturbances on labs….Test for lytes in blood (Na+, K+, Cl, Ca, PO4, Mg)

Decrease in GFR may lead to decreased renal calcitrol production – and thus reduced ionized Calcium in blood resulting in stimulated PTH secretion. Test PTH.

Electrolytes disturbances, mainly fluctuations in K+, may cause changes on ECG Serum H+ and HCO3 – Once the GFR falls below 25 mL/min, organic acids accumulate, producing a delta

metabolic acidosis. Hyperkalemia can also inhibit tubular HCO32– reabsorption, as can extracellular volume

expansion and elevated levels of parathyroid hormone (PTH). Eventually, as the kidneys fail, the level of serum HCO3

2– falls severely, reflecting the exhaustion of all body buffer systems, including bone. Arterial blood gases BUN

Discuss medical treatment of the main causes of electrolyte and acid-base disordersElectrolyte disorders:- Sodium:

o Hyponatremia: diuretics, diarrhea, heart failure, renal disease Mild: small adjustments to treat cause

Hypovolemia: NSHypervolemia: water restriction + treat cause (ACE-I, loop diuretic + replace lytes)

Severe: ↓ water intake, ↑ Na by 1mEg/hr (correcting too fast can lead to osmotic demyelination syndrome)

o Hypernatremia: controlled water replacement Free water deficit = TBW (kg x 0.6) x (serum Na/140) – 1)

Hypervolemia: replace free water deficit w 5% D/W +/- loop diuretic Euvolemia: 5% D/W or 0.45% saline Hypovolemia, nonketotic hyperglycemic coma: 0.45% saline or 0.9% NS + 5% D/W

If severe acidosis (pH <7.1), add bicarb- Potassium: mvmts depend on insulin (K into cells), beta-adrenergic (K into cells), acid-base status (acute metab

acidosis moves K out of cells)o Hypokalemia:↑ loss from kidneys/GI

Tmt: admin K and treat causeo Hyperkalemia: ↑ K intake, drugs impairing secretion, acute/chronic kidney disease, metabolic

acidosis (DKA)Tmt:

Mild: Na polystyrene sulfonate (cation exchange resin) or loop diuretic Mod/severe: IV insulin, glucose, Ca (if loss of P wave or wide QRS) – protects cardiac muscle,

possibly beta2-agonist (eg. Albuterol), hemodyalisis (for pts w renal failure OR ineffective emerg tmt)

- Calcium: o Hypocalcemia:

Tetanus: IV Ca gluconate Post op hypoparathyroidism: oral Ca Chronic: Ca PO and vitamin D

o Hypercalcemia Mild: asymp defer tmt until determine dx → treat underlying cause

Symptomatic: PO4 oral (binds Ca during meals) OR isotonic saline + lasix – monitor K and Mg q4h

Page 74: Objectives - Internal Medicine Define diastolic and systolic heart

Moderate: isotonic saline + lasix OR treat cause: ↓ bone resorption: bisphosphonates, calcitonin, steroids, chloroquine CA associated: bisphosphonates

Acid-base disorders:- Metabolic acidosis: treat underlying cause, may require hemodialysis (renal failure, ethylene glycol, methanol,

salicylate poisoning)o HCO3: when acidosis results from loss of HCO3 or ↑ inorganic acids (normal AG)

NOT to be used in ↑ AG bc intracellular acidosis not corrected, can worsen situation- Metabolic alkalosis: treat cause, IV NS for Cl-responsive metabolic acidosis

o Severe alkalosis may require more urgent correction via hemodyialysis- Respiratory acidosis: ventilation, HCO3 CONTRAINDICATED- Resp alkalosis: not life-threatening → treat cause

Identify the ECG findings associated with hyper/hypokalemia

HYPERkalemia ECG changes: peaked and narrow T waves, decreased amplitude and loss of P waves, prolonged PR interval, widened QRS complex approaching a sine wave pattern, AV block, ventricular fibrillation and asystole

HYPOkalemia ECG changes: U waves most important, flattened or inverted T waves, depressed ST segments and prolongation of QT interval…if severe PT prolongation, widened QRS complex and arrhythmias

Recognize the metabolic disorders of adrenal insufficiency (Addison’s disease)

Glucocorticoid (i.e. cortisol) insufficiency can be:

Page 75: Objectives - Internal Medicine Define diastolic and systolic heart

1) primary: resulting from destruction of adrenal cortex, or2) secondary: resulting from ACTH hyposecretion

Addison’s disease autoimmune destruction of adrenal glands (70% of patients have anti-adrenal antibodies) most common cause of primary adrenal insufficiency (65% of all cases); can be fatal if untreated usually both glucocorticoid (cortisol) and mineralcorticoid (aldosterone) secretions are diminished (mild

forms can cause decrease of one or the other) adrenal medulla function is usually spared

CAUSES OF ADRENAL INSUFFICIENCY

PRIMARY ADRENAL DISORDERSAutoimmune Addison’s, polyglandular autoimmune syndrome type I or IIInfectious TB, fungal, CMV, HIVVascular Bilateral adrenal hemorrhage, sepsis, coagulopathy, thrombosis/ embolism,

adrenal infarctInfiltration Metastatic CA, sarcoidosis, amyloidosis, hemochromatosisCongenital Congenital adrenal hyperplasia (ex. 21-hydroxylase deficiency), other rare

diseasesIatrogenic Bilateral adrenalectomy, drugs: metyrapone, ketoconazole, mifepristoneMineralcorticoid Deficiency ONLY Heparin therapy, critical illness, converting-enzyme inhibitorsSECONDARY ADRENAL DISORDERSSecondary Adrenal Insufficiency Hypothalamic-pituitary dysfunction, exogenous glucocorticoids, after

removal of ACTH-secreting tumourHyporeninemic Hypoaldosteronism Diabetic nephropathy, tubulointerstitial diseases, obstructive uropathy,

autonomic neuropathy, NSAIDs, beta-adrenergic drugs

List the main organisms responsible for bacterial meningitis

50% - Strep pneumonia (most common cause in adults >20 yo)25% - Neisseria meningitides (most important cause in 2-20 yo)10% - group B strep10% - Listeria monocytogenes

*Staph aureus most common cause following neurosurgical procedures*Haemophilus influenza was an important cause until introduction of Hib vaccine in 1987

Interpret the results of a lumbar puncture

Condition Colour Protein Glucose CellsNORMAL Clear < 0.45 g/l 60% of serum glu

> 3.0 mmol/lWBC: 0-5RBC: 0Neutro: 0

INFECTIOUSViral infection Clear or opalescent Normal or slightly

↑< 0.45 g/l

Normal <1000x106/lMainly lymphoSome PMNs

Page 76: Objectives - Internal Medicine Define diastolic and systolic heart

Bacterial infection Opalescent or yellow

May clot

> 1 g/l ↓(<25% serum glu OR

<2.0 mmol/l)

>1000x106/lPMNs

Granulomatous infection (TB, fungal)

Clear or opalescent ↑ but usually < 5 g/l

↓(<2.0-4.0 mmol/l)

<1000x106/lLympho

Implement empiric treatment of bacterial meningitis with antibiotic therapyThe empiric approach to antibiotic selection in patients with suspected bacterial meningitis is directed at the most likely bacteria based on the patient’s age and host factors.

From age 2-18, N. meningitis with a portal of entry in nasopharynx was most common cause (60% cases), followed by S. Pneumonia and H. influenza.

In adults up to age 60, S. pneumonia was responsible for 60% of cases, followed by N. meningitis (20%), H. influenza (10%).

In adults ≥60, almost 70% due to S. pneumonia, 20% to L. monocytogenes.

**Selected 3rd generation cephalosporins (cefotaxime and ceftriaxone), are the beta-lactams of choice in the empiric treatment of meningitis. These drugs have consistent CSF penetration and potent activity against the major pathogens of bacterial meningitis, with the notable exceptions of Listeria monocytogenes and some penicillin-resistant strains of S. pneumoniae. With the increase of penicillin-resistant pneumococci, vancomycin should be added to cefotaxime or ceftriaxone as empiric treatment until culture and susceptibility results are available.

-Ceftazidime, a 3rd gen cephalosporin with broad invitro activity against gram-negative bacteria including Pseudomonas aeruginosa, is much less active against penicillin-resistant pneumococci.

Immune Status Most likely bugs TreatmentNo immune deficiency Strep pneumonia, N. meningitis,

Haemophilus influenza and group B strep most likely in healthy people up to age 50 with community acquired infection

Ceftriaxone 2g IVq12hOR

Cefotaxime 2gIVq4-6hPLUS

Vancomycin 30-60mg/kg IV per day in 2 or 3 divided doses

PLUS In adults over 50, Ampicillin 2g

IV q4h

Impaired Cellular Immunity(lymphoma, chemo, or high-dose glucocorticoids)

Listeria monocytogenes and gram negative bacilli (Pseudomonas aeruginosa) as well as Strep pneumoniae

Vancomycin 30-60mg/kg IV per day in 2-3 divided doses

PLUS Ampicillin 2gIV q4h Cefepime 2gIV q8h

OR Meropenem 2gIVq8h

Nosocomial Infection Gram negative bacilli (Klebsielle and Pseudomonas) are most common followed by strep, staph aureus, and coag-negative staph

Vancomycin 30-60 mg/kg per day in 2-3 divided dosesPLUS

Ceftazidime 2g IV q8hOR

Cefepime 2gIV q8hOR

Page 77: Objectives - Internal Medicine Define diastolic and systolic heart

Meropenem 2gIVq8h

Demonstrate an understanding of the expected clinical findings in a patient with meningitis- S&S: fever, headache, stiff neck, vomiting → do lumbar, regardless of if it seems viral

o Seizures in 30% o CN abnormalities, focal neuro deficits in 20% o Change of consciousness: irritability → confusion → drowsiness → stupor → coma o Dehydration, vascular collapse cause shock o Meningococcal : petechiae, purpuric rasho Signs of ICP : change in LOC, papillary edema

- Lumbar puncture:Condition Pressure Cells Predom cells Glucose ProteinNormal 100-200 0-5 Lympho 50-100 20-45Bact meningitis >300 100-10000 PMN <40 (<50%

serum)>100

Aseptic meningitis N or ↑ 10-1000 Lympho N N or ↑ (<100)Subacute or chr N or ↑ 25-2000 Lympho ↓ ↑ or ↑↑

Demonstrate the ability to order appropriate investigations in order to establish a diagnosis of endocarditisFever + new murmur = endocarditis until proven otherwise

The diagnosis of endocarditis uses Duke’s criteria:

Page 78: Objectives - Internal Medicine Define diastolic and systolic heart

- CBC + diff (looking for anemia, rheumatoid factor, ESR)- Blood culture X 3 (at least)- Urinalysis (looking for RBC casts)- Echocardiogram (look for vegetations, regurgitation, abcess)- EKG

Demonstrate an understanding of the expected clinical findings in a patient with bacterial endocarditis

General: Clinical signs and symptoms from streptococcus usually manifest 2-5 weeks after precipitating event (i.e.

dental extraction) If causative organism is s low-growing, symptoms can appear up to 6 months after infection before

definitive diagnosis is made Symptoms and signs relate to systemic infection, emboli, metastatic infective loci, congestive heart failure

or immune complex-associated lesions

Most common symptoms: Fever (90% of patients; may be absent in elderly or with presence of CHF or previous Abx use) Chills Weakness Shortness of breath Night sweats Loss of appetite Weight loss

Peripheral Manifestations Petechiae: red, non-blanching lesions (vasculitis or emboli) Splinter hemorrhages – nail beds: linear red-brown streaks (vasculitis or emboli) Osler nodes: 2-5mm painful nodules on pads of fingers or toes (vasculitis) Janeway lesions: macular red painless patches on fingers or toes (emboli) Roth spots: oval pale retinal lesions surrounded by hemorrhage (vasculitis)

Other possible symptoms/signs: MSK pain (especially back) in 50% Heart murmurs (85%), changing murmurs (5-10%) With endocarditis involving aortic or mitral valve, CHF in 2/3 of patients Splenomegaly (25-60%; usually only seen if symptoms are prolonged) Clubbing (10-15%; usually only seen if symptoms are prolonged)

Potentially serious signs: Emboli (with mitral or aortic valve infection)

o stroke may be the initial signo splenic artery: ++ LUQ pain radiating to left shoulder with friction rub and left pleural effusiono frequent sites of emboli: renal, cerebral, coronary, mesenteric arteries

Neurologic (1/3 of patients)o CNS embolization with acute neurologic deterioration (2-4x mortality); **always consider

endocarditis in DDX of young patients with strokeo Headacheo Seizureso Toxic encephalopathy, meningoencephalitis

Page 79: Objectives - Internal Medicine Define diastolic and systolic heart

mplement empiric treatment of bacterial endocarditis

IV Abx depending on clinical scenario and suspected organism – minimum 4 weeks Prophylaxis Amoxicillin 2g po 30-60 minutes prior for the following procedures:

o Dentalo Respiratory tracto Procedures on infected skin/skin structures/MSK structures

List the main classes of pharmacological agents used to treat HIV

Class Mechanism1) Nucleoside reverse transcriptase

inhibitors (NRTIs)Incorporated into growing viral DNA Competitively inhibits reverse transcriptase and terminates viral DNA growth

2) Nucleotide reverse transcriptase inhibitors

Similar to NRTIs( except are chemically preactivated and thus, require less biochemical processing in the body to become active)

3) Non-nucleoside reverse transcriptase inhibitors (NNRTIs)

Non-competitively inhibit function of reverse transcriptase, preventing viral RNA replication

4) Protease inhibitors (PIs) Prevent maturation of infectious virions by inhibiting the cleavage of polyproteins

5) Fusion inhibitors Inhibit viral fusion with T-cells by inhibiting gp41 or CCR5, thus preventing infection of healthy cells

6) Integrase inhibitors Inhibit integration of HIV DNA into the human genome thus preventing HIV replication

List the common opportunistic infections associated with advanced HIVMost common:

Pneumonia is seen with an increased frequency in patients with HIV infection; they appear to be particularly prone to infections with encapsulated organisms – S. pneumonia and H. influenza.

Pneumocystis pneumonia (PCP), once the hallmark of AIDS, has dramatically declined in incidence following the development of effective prophylactic regimens and the widespread use of combination ARV therapy. It is, however, the single most common cause of pneumonia in patients with HIV infection in the United States and can be identified as a likely etiologic agent in 25% of cases of pneumonia in patients with HIV infection.

P. jiroveci causes PCP but has extra-pulmonary manifestations – otic involvement, ophthalmic lesions of the chorioid, a necrotizing vasculitis, bone marrow hypoplasia and intestinal obstruction. Pneumocystis jirovecii (carinii) pneumonia remains the leading initial AIDS-defining illness and an important cause of death in patients with AIDS.

M. tuberculosis resurgence associated with HIV. HIV infection increases the risk of developing active TB by a factor of 100.

Atypical microbial infections –M. avium or M. intracellulare species – MAC (mycobacterium avium complex) – late complication of HIV infection, predominantly occurring in patients with CD4+ T cell counts of <50/µL. The most common presentation is disseminated disease with fever, weight loss, and night sweats.

Fungal infections of the lung. i.e. Coccidiodes immitis is a mold endemic in the southwest United States. It can cause a reactivation pulmonary syndrome in patients with HIV infection. Aspergillus infection may have

Page 80: Objectives - Internal Medicine Define diastolic and systolic heart

an unusual presentation in the respiratory tract of patients with AIDS where it gives the appearance of a pseudomembranous tracheobronchitis. Primary pulmonary infection of the lung may be seen with histoplasmosis *(most common prevalent endemic mycosis in the US). The most common pulmonary manifestation of histoplasmosis, however, is in the setting of disseminated disease, presumably due to reactivation. In this setting respiratory symptoms are usually minimal, with cough and dyspnea occurring in 10–30% of patients. The chest x-ray is abnormal in ~50% of patients, showing either a diffuse interstitial infiltrate or diffuse small nodules.

Toxiplasmosis is associated with cerebral abscesses and severe morbidity and mortality in AIDS patients. Oral lesions – thrush, hairy leukoplakia and apthous ulcers Candida – mucosal candidal infections, such as oropharyngeal thrush and vaginitis, are commonly diagnosed

in HIV-infected patients **Invasive esophageal candiasis is an AIDS defining diagnosis that occurs in 10% of immunosuppressed

people. Cryptococcus – most commonly presents as a meningeoencephalitis with fever, malaise and headache.

Page 81: Objectives - Internal Medicine Define diastolic and systolic heart

Define what is meant by a fever of unknown originTypes: daily or intermittent fever >38.3 w no source by clinical evaluation

A. Classic fever of unknown origin: 3 weeks, hospital evaluation for 3 days or outpt eval for 7 days or 3 outpt visits

B. Nosocomial fever of unknown origin: hospitalized >1day w/o fever on admission, fever evaluation of 3+ days

C. Immune-deficient fever of unknown origin: neutrophil <500, fever eval 3+ daysD. HIV-associated fever of unknown origin: outpt fever >4weeks or inpt fever >3days

Establish a differential diagnosis for a fever in a returning traveler

Questions to ask a returning traveler:- Pre travel vaccinations (vaccines possible for HepA and HepB, ), MMR, tetanus/diphtheria, varicella,

pertussis, polio, Japanese encephalitis, thyphoid fever, yellow fever, rabies)- Malaria prophylaxis- Countries visited- Food exposure: dairy, uncooked meat, water, seafood- Drugs, tattoos- Sexual history- Animal or insect bites- Swimming in beaches near storm drains, after heavy rainfalls- Traveler’s diarrhea

DDx: Malaria: Plasmodium falciparum, ovale, malariae, vivax HIV-1 infection Viral Hepatitis: HepA, HepB, HepC, HepE Dengue fever Typhoid or paratyphoid fever (enteric fever)

Page 82: Objectives - Internal Medicine Define diastolic and systolic heart

tb Mononucleosis (due to EBV or CMV) Rickettsial infection Schistosomiasis African trypanosomiasis (sleeping sickness) Diphtheria Rabies Drugs Vasculitis: giant cell arteriris, polyarteritis nodosa Neoplasm (lymphoma, renal cell carcinoma, hepatocellular carcinoma)

THERE IS AN AWESOME TABLE IN TORONTO NOTES SHOWING WHERE YOU GET WHAT!!!!!

Implement treatment for cellulitis in the diabetic patient

Cellulitis: Infection of the deeper layers of the skin Predilection for lower extremities where venous stasis predisposes to infection (impaired lymphatic

drainage) In diabetes, most common organisms: Streptococci and staphylococci; anaerobic and gram –ve rods if

associated with infected ulceration of skin

Treatment – Cellulitis in Diabetes Foot ulcers complicated by cellulitis: agents against anaerobes and enteric gram –ve rods For mild infections: oral antibiotics, for serious infections: hospitalization + parenteral (IV therapy)

(ampicillin-sulbactam; clindamycin + levofloxacin) Optimal control of blood sugar Xray on patients with foot ulcers to rule out osteomyelitis Elevate affected limb to enhance venous drainage

Demonstrate an understand of the principles of treatment for a patient in septic shock

Supportive: O2, IV fluids, IV Abx (empirical, depends on suspected source) CVS support: IV fluids, +/- dopamine, +/- norepinephrine; ICU transfer Activated protein C

o Modulates coagulation and inflammation in severe sepsiso Evidence suggests use in severe sepsis

IVIg: some evidence for Streptococcal toxic shock syndrome but limited by small size of trials in literature Hydrocortisone and fludricortisone in ACTH non-responders

Describe the various indications and modalities for isolation of a patient

AIRBORN PRECAUTIONS- Use these guidelines when caring for patients who have infections (e.g. tuberculosis, chickenpox, measles)

that spread through the air as small airborne droplets or dust particles containing the pathogen (5 microns or smaller)

- Place any infected patient in a negative-pressure isolation room with the door closed

Page 83: Objectives - Internal Medicine Define diastolic and systolic heart

- Wear an high efficiency respiratory mask (the mask, which filters inspired air, should filter particles one micron in size, have a 95% filter, and provide a tight facial seal (less than 10% facial seal leak), when entering the room.

- A patient on Airborne Precautions must wear a surgical mask (filters expired air) when leaving the room

DROPLETS PRECAUTIONS- Use these precautions when caring for patient who have infections (e.g. mumps, pertussis, meningitis,

Influenza, adenovirus in children) that spread by large particle droplets containing microorganisms (larger than 5 microns in size that are spread by coughing, sneezing, or talking)

- Wear a high filtration mask when working within 3 feet of patients - Place patient in a private room or with another patient with the same infection. - Special ventilation is not required and the door may remain open - Patient is to wear a surgical mask when leaving the room

CONTACT PRECAUTIONS- Use these precautions for patients who are infected or colonized by a microorganism that spreads by direct

contact (skin to skin) with the client or by indirect contact with a contaminated object (e.g., antimicrobial resistant microorganism (ARO) (e.g. MRSA), C. difficile, Respiratory syncytial virus (children and immunosuppressed adults), enteric pathogens (diapered or incontinent) (e.g. Shigella, E.coli 0157.H7, Rotavirus, Hepatitis A), major wounds, abscesses, or cellulitis (no dressing or dressing doesn’t adequately contain drainage), disseminated mucocutaneous/neonatal Herpes simplex or localized immunocompromised/disseminated Zoster infections

- Place patients with any of these infections in a private room - Masks are to be worn with respiratory infections with AROs - Wear gloves when entering the patient’s room and change gloves as needed - Wear a gown if you anticipate that your clothing may come in contact with the patient, or environmental

surfaces in the client’s room.; remove the gown before leaving the patient’s room - Use dedicated equipment when treating a patient with antimicrobial resistant microorganisms - During transfer of a patient requiring “Contact” precautions make certain staff in the receiving area have been

notified and understand the precautions to take.

Describe the factors affecting the emergence of drug-resistant organismsThe widespread use of antibiotics is playing a significant role in the emergence of drug-resistant bacteria. The volume of antibiotic prescribed is the major factor in increasing the rates of bacterial resistance. Inappropriate prescribing of antibiotics has been attributed to a number of causes including: people who insist on antibiotics, physicians simply prescribe them as they feel they do have the time to explain they are unnecessary, physicians who do not know when to prescribe antibiotics or else are overly cautious for medical legal reasons. Patient compliance with recommended treatment is another major problem. Patients forget to take medication, interrupt their treatment when they begin to feel better, or may be unable to afford a full course, thereby creating an ideal environment for microbes to adapt rather than be killed. In some countries, low quality antibiotics (poorly formulated or manufactured, counterfeited or expired) are still sold and used for self-medication or prophylaxis.Hospitals are a critical component of the antimicrobial resistance problem worldwide. The combination of highly susceptible patients, intensive and prolonged antimicrobial use, and cross-infection has resulted in nosocomial infections with highly resistant bacterial pathogens. Resistant hospital-acquired infections are expensive to control and extremely difficult to eradicate. Failure to implement simple infection control practices, such as handwashing and changing gloves before and after contact with patients, is a common cause of infection spread in hospitals throughout the world. Hospitals are also the eventual site of treatment for many patients with severe infections due to resistant pathogens acquired in the community. In the wake of the AIDS epidemic, the prevalence of such infections can be expected to increase.Veterinary prescription of antimicrobials also contributes to the problem of resistance. In North America and Europe, an estimated 50% in tonnage of all antimicrobial production is used in food-producing animals and poultry.

Page 84: Objectives - Internal Medicine Define diastolic and systolic heart

The largest quantities are used as regular supplements for prophylaxis or growth promotion, thus exposing a large number of animals, irrespective of their health status, to frequently subtherapeutic concentrations of antimicrobials. Such widespread use of antimicrobials for disease control and growth promotion in animals has been paralleled by an increase in resistance in those bacteria (such as Salmonella and Campylobacter) that can spread from animals, often through food, to cause infections in humans.Drugs are used in animals that are used as human food, such as cows, pigs, chickens, fish, etc., and these drugs can affect the safety of the meat, milk, and eggs produced from those animals and can be the source of superbugs. For example, farm animals, particularly pigs, are believed to be able to infect people with MRSA. The resistant bacteria in animals due to antibiotic exposure can be transmitted to humans via three pathways, those being through the consumption of meat, from close or direct contact with animals, or through the environment. In the USA federal agencies do not collect data on antibiotic use in animals but animal to human spread of drug resistant organisms has been demonstrated in research studies. Antibiotics are still used in U.S. animal feed—along with other ingredients which have safety concerns.

4 mechanisms by which microorganisms exhibit resistance to antimicrobials:1. Drug inactivation or modification – e.g. enzymatic deactivation of Pen G in some penicillin-resistant

microorganisms through production of B-lactamases2. Alteration of target site – e.g. alteration of PBP – the binding target site of penicillins – in MRSA3. Alteration of metabolic pathway; e.g. some sulfonamide-resistant bacteria do not require para-

aminobenzoic acid (PABA), an important precursor for the synthesis of folic acid and nucleic acids in bacteria inhibited by sulfonamides. Instead, like mammalian cells, they turn to utilizing preformed folic acid.

4. Reduced drug accumulation: by decreasing drug permeability and/or increasing active efflux of drugs across cell surface.

Prescribe appropriate treatment for C. difficile diarrhea, including the need for isolationGeneral measures:

Discontinue antibiotico Diarrhea resolves in 25% of cases with only D/C of causative agento If antibiotic required, choose one with lower risk

Start Ab immediately if: older pt, multiple comorbid conditions, Ab can’t be D/C, severe disease (persistent diarrhea, dysentery – fever, leukocytosis)

Avoid meds that could worsen condition: opioids, antidiarrheal agents Do not retest for toxin post-tmt if asymp → may be positive but doesn’t need tmt

Management: If in hospital: isolation – gloves and jacket, MUST wash hands with soap after exposure Metronidazole 500mg TID for 10-14 days PO or IV Vancomycin (only good if PO) 125-500mg PO QID for 10-14 days

o Pts at high rish of fulminant diseaseo 2nd line agent to metronidazole – very expensive

Low dose in most pts, high dose in severe illnessRecurrence: recurrence risk doubles with each episode

Vancomycin taper: start at 125mg q6h for 1 week → BID for 1 week → DIE for 1 week → every other day for 1 week → every third day for 2 weeks

Probiotics (florastor): 250mg PO BID or TID for 1 monthFulminant disease: high mortality rate

Metronidazole IV 500mg q8h AND Vancomycine 500mg PO QID AND Vancomycin enema 500mg in 100cc NS q6h

Page 85: Objectives - Internal Medicine Define diastolic and systolic heart

Recognize the risk factors, clinical presentation, treatment and appropriate isolation procedures for Tb

Risk factors:- Exposure/infection

o Travel or birth in country with high TB prevalence (eg: Asia, sub-saharan Africa)o Aboriginalo Crowded living conditionso Homelesso Personal or occupational contacto IVDU

- Progression from latent infection to active diseaseo Immunocompromised/immunosuppressedo Concurrent local disease (lungs)o Skin test conversion within past 2 yearso Meds – biologics

Clinical presentation:- Primary infection usually asymptomatic- Secondary infection or reactivation usually presents with constitutional symptoms: fatigue, anorexia,

night sweats, weight loss and site dependent symptomso Pulmonary TB (chronic productive cough +/- hemoptysis, CXR consolidation or cavitation,

lymphadenopathy)o Military TB (widely disseminated spread especially to lungs, abdo organs, marrow, CNS, CXR with

multiple small 2-4 mm millet seed-like lesions)o Extra-pulmonary TB (lymphadenitis, pleurisy, pericarditis, hepatitis, peritonitis, meningitis,

ostemyelitis (vertebral = Pott’s), adrenal infection (causing Addison’s disease), renal, kidney)

Treatment:- Empiric therapy: INH + rifampin + pyrazinamide + ethambutol- Pulmonary TB: INH + rifampin + pyrazinamide for 2 months THEN INH + rifampin for 6 months (Rule: 3 for

2 and 2 for 4 = always makes a total of 6, total treatment is for 6 months)- Extrapulmonary TB: same as pulmonary TB but increase to 9-12 months + corticosteroids for meningitis,

osteomyelitis

Demonstrate the ability to perform a physical examination focused on the complications of diabetes

MICROVASCULAR COMPLICATIONS

Nephropathy – NO PHYSICAL EXAM No physical exam signs, but important labs to check:

o Microalbuminuria (30% of patients 10-15 years after diagnosis) – associated with 10-20x increased risk of diabetic nephropathy

o Followed by a decline in GFR as albumin excretion rate increases; may eventually require dialysis or renal transplant

o Yearly screening for microalbuminuria – ratio of albumin (protein): creatinine (random spot urine test)

Retinopathy – DILATED FUNDOSCOPY May already be present at time of diagnosis

Page 86: Objectives - Internal Medicine Define diastolic and systolic heart

Non-proliferative Stageo Dilation of venules, microaneurysms (dots or flames)o Hard exudates (yellow deposits of protein and lipids

Proliferative Stageo Cotton wool spots (white; superficial retinal infarcts resulting in hypoxic necrosis of retinal nerve

fibers)o Neovascularization on retina, optic disc or iris – these fragile vessels can hemorrhage and causes

fibrosis Other signs

o Premature development of senile cataractso Snowflake lens opacities in young patientso Retinal detachment

Requires annual dilated fundoscopy

Neuropathy – NEURO, MSK, SKIN, PERIPHERAL VASCULAR, GI, CARDIAC EXAM Peripheral polyneuropathy (most common)

o Bilateral, symmetrical, distal, primarily sensory with glove and stocking distributiono Pain, numbness, hyperesthesias, paresthesias progress to sensory losso Loss of proprioception

Abnormal gait, repeated trauma to tarsal bones (may develop Charcot joints) Foot trauma + arterial insufficiency = foot ulcers (may progress to osteomyelitis and

gangrene) Mononeuropathy

o Usually present acutely and are self-limiting; signs depend on specific nerve affected Radiculopathies

o Self-limited painful sensory syndromes of spinal nerves Diabetic Amyotrophy

o Muscle atrophy and weakness: anterior thigh and pelvic girdle Autonomic Neuropathy (SNS or PNS)

o GI: gastric dysmotility, gastroparesis, diarrhea, constipation, bloating, etc.o Orthostatic hypotensiono Cardiac rhythm disturbances can cause syncope and cardiac arresto Bladder: incontinence, retentiono Erectile dysfunction and decreased arousal

MACROVASCULAR COMPLICATIONS

Include: Hypertension, myocardial ischemia and infarction, TIAs, strokes, peripheral vascular disease EXAMS: VITAL SIGNS, CARDIAC, PERIPHERAL VASCULAR, NEURO (if applicable, for stroke)

Describe the involvement of the various target organs in the diabetic patient

Microvascular Retinopathy

o Nonproliferative diabetic retinopathy appears late in the first decade of the disease, marked by retinal vascular microaneyrysms, blot hemorrhages, cotton wool spots loss of retinal pericytes, increased retinal vascular permeability, alterations in retinal

blood flow, abnormal retinal microvasculature, which lead to retinal ischemia

Page 87: Objectives - Internal Medicine Define diastolic and systolic heart

severe nonproliferative diabetic retinopathy has a greater chance of evolving to proliferative retinopathy

o Proliferative diabetic retinopathy Neovascularization in response to retinal hypoxia Newly formed vessels rupture easily leading to vitreous hemorrhage, fibrosis and retinal

detachment Nephropathy

o Hemodynamic alterations in the renal microcirculation : glomerular hyperfiltration, increased glomerular capillary pressure,

o Structural changes in the glomerulus: increased extracellular matrix, basement membrane thickening, mesangial expansion, fibrosis

o After 5-10 years of DM-I, about 40% of patients have microalbuminuria, later proteinuria presents with a steady decline in GFR

o In DM-II, microalbuminuria may be present at time of diagnosis, usually accompanied by hypertension

Neuropathyo Occurs in about 50% of patients with long-standing DM-I or DM-IIo Correlates with duration of diabetes and glycemic controlo Most common form is distal symmetric polyneuropathy and presents as distal sensory loss

although hyperesthesia, parathesia(numbness, tingling, sharpness, burning) and pain may also occur

GI/GU dysfunctiono Gastroparesis and altered bowel motility (diarrhea or constipation) are due to parasympathetic

dysfunction secondary to hyperglycemia and to hyperglycemia itselfo GU dysfunction including cystopathy(inability to sense a full bladder and failure to void

completely), erectile dysfunction and female sexual dysfunction are due to diabetic autonomic neuropathy

Macrovascular Coronary artery disease

o DM-II increases the CVS death rate by 2X in men and 4X in women Peripheral vascular disease

o Peripheral vascular disease, neuropathy and poor wound healing all contribute to making DM the leading cause of nontraumatic lower extremity amputation

Cerebral vascular disease/ischemic strokeo DM increases the risk of stroke by 3X

Describe the mechanism of action of hypoglycemic drugs and insulin

Drugs that stimulate insulin secretion (Insulin secretagognes)1. Sulfonylureas

o Glicazide o Glimepiride (Amaryl)o Glyburide (DiaBeta,

Micronase, Glynase)o Chlorpropamide (Diabinese)o Tolbutamide (Orinase)

o Stimulates basal insulin secretion and increase meal-stimulated insulin release

o Do not correct the impaired early-phase insulin response

o Bind specific receptors on the surface of pancreatic ß cells

o Activation of these receptors close K+ channels → depolarization of ß cells

o Depolarization permits Ca2+ enter the cells and actively promote insulin release

2. Meglitinideso Repaglinide (Prandin)

o Induce an early insulin response to meals, lowering postprandial

o Bind to sulfonylurea receptor and close ATP-sensitive K+ channels

Page 88: Objectives - Internal Medicine Define diastolic and systolic heart

o Nateglinide (Starlix) glucose levelso Short-acting, and should be taken

only with meals

(same cascade as above)o Rapidly absorbed from the

intestine, reaching peak plasma levels within an hour

o Metabolized in the liver and have a plasma half-life of about 1.5 hours

o Drugs that principally lower glucose levels1. Biguanides

o Metformino ↓ hepatic glucose productiono May ↓ glucose absorptiono Enhance insulin-mediated

glucose uptake

** The only oral antiglycemic drug not associated with weight gain.Do not cause hypoglycemia when used alone but can potentiate hypoglycemic effects of insulin and sulfonylureas

o Primary action on the livero Reduce hepatic gluconeogenesis

by activating adenosine monophosphate-activated protein kinase (AMPK)

o AMPK acts as an intracellular energy sensor and in the liver has a critical role in regulating gluconeogenesis

o Has a half-life of 1.5-3 hourso Does not bind to plasma proteinso Not metabolized in humans and

excreted unchanged by the kidneys

2. Thiazolidinedioneso Rosiglitazoneo Pioglitazone

o Enhance insulin sensitivityo Lower blood glucose and

circulating insulin levels by:o Increasing peripheral glucose

uptakeo Enhancing fat cell sensitivity

to insulino Lowering hepatic glucose

output

** Cardiovascular safety under scrutiny

o Stimulate adiponectin secretion, which sensitize tissues to the effects on insulin

o Inhibit resistin secretion, which reduces insulin resistance

o Increase glucose transporter expression (GLUT 1 and GLUT 4)

o Decrease fatty acid levelso Decrease hepatic glucose outputo Increase differenciation of

preadipocytes into adipocyteso Also have an effect on lipids and

other cardiovascular risk factors:o Increase total cholesterol, LDL and HDL, reduction in free fatty acids

Drugs that affect glucose absorption1. Alpha-glucosidase inhibitors

o Acarboseo Delay the absorption of

carbohydrates and reduce postprandial glycemic excursion

o Competitive inhibitors of intestinal brush border α-glucosidase

o Potent inhibitors of glucoamylase, α-amylase and sucrose

o Bind 1000 more avidly to the intestinal dissacharides than do products of carbohydrate digestion or sucrose

2. Dipetidyl Peptidase-4 Inhibitors o Slow the breakdown of glucagon-like petitd 1 thereby enhance insulin secretion and suppress

o Oral DPP-IV inhibitoro Prolong the action of

endogenously released GLP-1

Page 89: Objectives - Internal Medicine Define diastolic and systolic heart

glucagon secretiono Reduce HbA1 by approximately

0.7%

** Reduce dose in renal failure

o The drug is removed by the kidney

3. Enexatide o Lower blood glucose and HbA1c levels

o Also cause weight loss

** Must be given by SC injections

o GLP-1 receptor agonist resistant to DPP-IV action

Anti-obesity agentsOrlistat o Weight loss

o Can produce modest improvement in glycemic control

** Should not be used as a monotherapy for glycemic control

o Intestinal lipase inhibitor

Insulin4 types of insulin (see table below) o Lower blood sugar by moving

sugar from the bloodstream into the cells so that it becomes readily useable for energy

o Insulin receptor = a tyrosine kinase

o Binding of insulin to the alpha subunits causes the beta subunits to phosphorylate themselves (autophosphorylation) → activation of the catalytic activity of the receptor → phosphorylation (insulin receptor substrate 1 or IRS-1) → recruitment and activation of other enzymes → mediation of insulin's effects

Insulin type Onset(approximation)

Peak time(approximation)

Duration(approximation)

Rapid acting, Lispro, Aspart, Glulisine insulin

15 minutes 30 to 90 minutes 3 to 5 hours

Short acting, Regular (R) insulin

30 to 60 minutes 2 to 4 hours 5 to 8 hours

Intermediate acting, NPH (N) or Lente (L) insulin

1 to 3 hours 8 hours 12 to 16 hours

Long acting, Glargine, Detemir insulin

1 hour none 20 to 26 h

Implement emergency treatment for diabetic ketoacidosis*I didn’t see any objective really describing what ketoacidosis is so I gave a brief intro. Skip if you want.Diabetic ketoacidosis (DK) is one of the most serious acute complications of diabetes. Runaway catabolism due to lack of insulin action plus increased counterregulatory hormones leads to life-threatening metabolic acidosis. In DKA, metabolic acidosis is often the major finding, while the serum glucose concentration is generally below

Page 90: Objectives - Internal Medicine Define diastolic and systolic heart

800mg/dL (44mmol/L). However, serum glucose concentrations may exceed 50mmol/L in patients with DKA who are comatose.

It is characterized by a triad of hyperglycemia, anion gap metabolic acidosis, and ketonemia. The most common precipitating factors are infection (often pneumonia or UTI) and discontinuation of or inadequate insulin therapy. Others include –acute major illness (MI, CVA, pancreatitis), new onset diabetes type 1, meds affecting carbohydrate metabolism (glucocorticoids, thiazide durietics in high dose), cocaine, etc. DKA usually develops over a 24-hr period and earliest symptoms of marked hyperglycemia are polyuria , polydypsia, and weight loss. As the degree of duration of hyperglycemia progresses, neurologic symptoms, including lethargy, focal signs, and obtundation, which can progress in later stages, can be seen.

Initial Evaluation: Airway, breathing, and circulation (ABC) status Mental status Possible precipitating events (eg source of infection, MI) Volume status

Initial Labs: Serum glucose Serum electrolytes (with calculation of anion gap), BUN, plasma creatinine CBC with differential Urinalysis and urine ketones by dipstick Plasma osmolality Serum ketones (if urine ketones are present) Arterial blood gas if the serum bicarb is substantially reduced ECG

Therapy includes fluid replacement to correct for both hypovolemia and hyperosmolality, insulin administration to correct for hyperglycemia, metabolic acidosis and K+ repletion. Vigorous IV fluid replacement is recommended and should correct estimated deficits within first 24 hours, with care to avoid an overly rapid reduction in serum osmolality. Isotonic (0.9 percent) saline is initially infused at a rate of 15 to 20 mL/kg per hour, in the absence of cardiac compromise, for the first few hours. This is followed by one-half isotonic (0.45 percent) saline at 4 to 14 mL/kg per hour if the serum sodium is normal or elevated; isotonic saline is continued if hyponatremia is present. We add dextrose to the saline solution when the serum glucose reaches 200 mg/dL (11.1 mmol/L). The need for potassium repletion may influence the timing of one-half isotonic saline therapy, since the addition of potassium to isotonic saline creates a hypertonic solution that can worsen the underlying hyperosmolality.

Initial with low-dose intravenous insulin in all patients with moderate to severe DKA who have a serum potassium is ≥3.3 meq/L. Patients with an initial serum potassium below 3.3 meq/L should receive aggressive fluid and potassium replacement PRIOR to treatment with insulin. If the serum potassium is ≥3.3 meq/L, we give a continuous intravenous infusion of regular insulin at 0.14 U/kg per h; at this dose, an initial intravenous bolus is not necessary. An alternative option is to administer an IV bolus (0.1 U/kg body weight) of regular insulin, followed by a continuous infusion at a dose of 0.1 U/kg per hour. The dose is doubled if the glucose does not fall by 50 to 70 mg/dL (2.8 to 3.9 mmol/L) in the first hour.

We initiate a multiple-dose subcutaneous insulin schedule when the ketoacidosis has resolved and the patient is able to eat. The intravenous insulin infusion should be continued for one to two hours after initiating the subcutaneous insulin, to avoid recurrence of hyperglycemia.Patients with DKA typically have a marked degree of potassium depletion due to both renal and, in some patients, gastrointestinal losses. However, because of potassium redistribution from the cells into the extracellular fluid, the initial serum potassium concentration is often normal or elevated, an effect that will be reversed by insulin therapy. We recommend that replacement with intravenous KCl be initiated when the serum potassium

Page 91: Objectives - Internal Medicine Define diastolic and systolic heart

concentration is ≤5.3 meq/L. Patients with an initial serum potassium below 3.3 meq/L should receive aggressive fluid and potassium replacement PRIOR to treatment with insulin to prevent initial worsening of the hypokalemia.

In summary:Patients must be admitted to the ICU for fluid resuscitation (3-4L in 8 hours) with NS and IV insulin. Identify and treat the precipitating cause (e.g. antibiotics). Sodium, potassium, phosphate, and glucose (change NS fluids to D5NS when glucose <250mg/L) must be monitored and repleted every 2 hours. Change IV insulin to an SQ insulin sliding scale once the anion gap normalizes.

Recognize and manage hyperglycemia and hypoglycemiaHyperglycemia: target glucose 4.4-6.7 during day; 5.6-7.8 at bedtime; A1C <7%Tmt:- Exercise- Monitoring glucose, A1C- Insuline for DM1, uncontrolled DM2- Oral antihyperglycemics:

o Secretagogues: Sulfonylureas (acetohexamide, chlorpropamide, tolbutamide, tolazamide, glyburide,

glipizide): ↑ beta-cell insuline secretionAdverse effects: hypoglycemia, WG

Short acting insulin secretagogue (repaglinide, nateglinide): stim insulin secretion during meals => ↓ postprandial hyperglycemia, w lower risk of hypoglycemia, less WG

o Insulin sensitizers: Biguanides (metformin): ↑ suppression of hepatic glucose production by insulin = peripheral

insulin sensitizers, ↓ lipid levels, ↓ GI absorption, antithrombotic effectAdverse effects: lactic acidosis, NO hypoglycemia, GI distress (diarrhea, nausea, pain), B12 malabsorptionCI: renal insufficiency, heart failure, alcoholism, dehydrationHelps WL

Thiazolidinediones = TZDs (pioglitazone, rosiglitazone): ↓ peripheral insulin resistanceAdverse effects: WG, fluid retention, anemia, ↑ LDL, hepatotox → monitor liver function

o Intestinal enzyme inhibitors: Alpha-glucosidase inhibitors (acarbose, miglitol): ↓ postprandial plasma glucose, taken with

mealAdverse effects: GI (flatulence, diarrhea, bloating,

GLP-1 agonists (exenatide): enhance glucose-dependent insulin secretion, slow gastric emptying, ↓ appetite, promote WL, stim beta cell proliferation

Hypoglycemia: most commonly complication of drug tmt in DM (esp insulin) - S&S: sweating, nausea, warmth, anxiety, tremors, palpitations, hunger, paresthesias, headache,

blurred/double vision, confusion, difficulty speaking, seizures, coma- Tmt: oral sugar or IV dextrose, can give parenteral glucagon

List the main causes of hypothyroidism and hyperthyroidism

HYPERthyroidism HYPOthyroidismo Grave`s disease (most common –autoimmune

dysfunction results in production of IgG which bind TSH-r on thyroid and stimulate secretion

Primary (90% of cases)o Hashimoto`s thyroiditis (most common, auto-

immune destruction)

Page 92: Objectives - Internal Medicine Define diastolic and systolic heart

of T3/T4)o Toxic nodular Goitre – aberrant hormone

productiono Toxic noduleo Thyroiditis (subacute, silent, postpartum)o McCune-Albright syndromeo Jod Basedowo Extrathyroidal sources of hormone

(endogenous or exogenous, TSH secreting tumors)

o Excessive thyroid stimulation (ex: increased hCG in pregnancy)

o Iatrogenic (thyroidectomy, underdose thyroid replacement, radioactive thyroid ablation)

o Amiodarone-induced, or lithiumo Iodine deficiency (dev countries)

Secondaryo Hypothalamic or pituitary failure

Demonstrate an appropriate physical examination of a patient with hyperthyroidism

Symptoms Nervousness, heat intolerance, fatigue and weakness, palpitations, increased appetite, weight loss,

oligomenorrhea

Physical Exams & Signs Tachycardia: pulse and cardiac auscultation Atrial fibrillation: pulse and cardiac auscultation Wide pulse pressure (difference between systolic and diastolic BP >40 mmHg): blood pressure Brisk reflexes: neuro exam – reflexes Fine tremor: neuro exam – observation Proximal limb girdle myopathy: MSK exam (strength, gait, range of motion) Chemosis (swelling/edema of the conjunctiva): eye exam – observation from side of patient Graves Disease – specific

o Periorbital edema, proptosis, extraocular muscle weakness, optic nerve damage: eye exam – observation from side and front of patient; neuro exam – CN II (fundoscopy), III, IV, VI

o Pretibial myxedema (thickened skin over tibia without pitting edema) : MSK – observationo Onycholysis (separation of finger nails from nail beds), clubbing: skin exam - observation

Interpret thyroid function tests

TSH Hyperthyroidism

o Primary: TSH is low (negative feedback from increased circulating T3 and T4 )o Secondary: TSH is high (which increases T3 and T4)

Hypothyroidismo Primary: TSH is high (less negative feedback because decreased circulating T3 and T4 )o Secondary: TSH is low

Free T3 and T4

Levels depend on amount of TBGo TBG increases with pregnancy, oral contraceptive use, acute infectious hepatitis, biliary cirrhosiso TBG decreases with androgens, glucocorticoids, cirrhosis, hyponatremia, phenytoin, ASA,

NSAIDS, nephritic syndrome, severe systemic illness

Page 93: Objectives - Internal Medicine Define diastolic and systolic heart

Levels are independent of TBG and measure biological activity

Thyroid Autoantibodies Thyroglobulin antibodies (TgAb), thyroid peroxidase (microsomal antibodies), TSH receptor inhibiting

antibodieso Increased in Hashimoto’s disease

Thyroid stimulating immunoglobulin (TSI)o Increased in Graves’ disease

Plasma Thyroglobulin Used to monitor residual thyroid activity post-thyroid ablation, ex for thyroid cancer recurrence Undetectable levels = remission Normal/elevated levels = probable persistent, recurrent, or metastatic disease

Serum Calcitonin Not routinely done to investigate most thyroid nodules Ordered if suspicious of medullary thyroid carcinoma or family history of MEN IIa or IIb syndromes

Thyroid Imaging/Scans Normal gland 15-20g Thyroid U/S

o To measure size of gland, solid vs. cystic nodule Thyroid scan (Technetium-99)

o Differentiates between hot (functioning) and cold (non-functioning) noduleso To distinguish between three major types of high-uptake hyperthyroidism

Graves’ disease – diffuse uptake Toxic multinodular goiter – multiple discrete areas Solid toxic adenoma – single intense area of uptake

o Test of structure – order if there is a thyroid nodule and patient is hyperthyroid Radioactive iodine uptake (RAIU)

o Measures turnover of iodine by thyroid gland in vivoo In areas of low iodine intake and endemic goiter, 24h RAIU may be as high as 60-90%o In areas of high iodine intake, normal 24h RAIU will be 8-30%o RAIU is high in Graves’ disease or toxic nodular goiter and low in subacute thyroiditis, active

phase of Hashimoto’s thyroiditis and excess iodine intakeo Test of function – order if patient is hyperthyroid

Thyroid BiopsyFine needle aspiration cytology differentiates between benign and malignant disease

Describe the clinical manifestations of hyperthyroidism and hypothyroidism

Hypothyroidism HyperthyroidismSymptoms most are nonspecific and develop gradually.

- Cold intolerance- Fatigue/somnolence- Poor memory- Constipation- Menorrhagia- Myalgias

- Heat intolerance- Weight loss- Weakness- Palpitations- Oligomenorrhea- Anxiety

Page 94: Objectives - Internal Medicine Define diastolic and systolic heart

- Hoarseness- Mild weight gain

Signs - Goiter (no bruit)- Slow tendon reflex relaxation- Bradycardia- Facial and periorbital edema- Dry skin- Myxedema (non pitting)

(rare)- Hypoventilation- Pericardial or pleural effusion- Deafness- Carpal tunnel syndrome- Ascites

- Goiter (± bruit)- Brisk tendon reflex- Tremor- Proximal weakness/myopathy- Stare and eyelid lag- Cardiac abnormalities: sinus tachycardia,

atrial fibrillation, exacerbation of CAD or HF

- Hot and sweaty hands

** S&S specific to Grave’s disease- Proptosis or pretibial edema- Diffuse non tender goiter

** Suggesting other causes than Grave’s disease:- Recent pregnancy- Neck pain- Recent iodine administration

Discuss the pharmacological treatment for hyperthyroidism and hypothyroidism Symptomatic hyperthyroidism is treated with propranolol, hydration, rest, and adequate nutrition.

Cooling measures are required for severe hyperthermia. Do not administer beta-blocker therapy to a patient with significant history of asthma. Calcium-channel blocklers can be used for the same purposes when beta blockers are contraindicated or poorly tolerated. Mild cases of hyperthyroidism can then be treated with propylthiouracil or methimazole (antithyroid drugs – these drugs inhibit multiple steps in synthesis of T4 or T3).

o Antithyroid medications inhibit formation and coupling of iodotyrosines in thyroglobulin, which are necessary for thyroid hormone synthesis

o A second therapeutic action of propylthiouracil, but not methimazole, is the inhibition of conversion of T4 to T3. T3 is a more biologically active form of thyroid hormone. A quick reduction in T3 is associated with a clinically significant improvement in thyrotoxic symptoms

o Methimazole is a more potent and longer-acting drug. Often, patient compliance is better with methimazole taken once or twice daily than with propylthiouracil given 3 or 4 times daily. Propylthiouracil often is the drug of choice in severe thyrotoxicosis because of the additional benefit of inhibition of T4 -to-T3 conversion. Administer propylthiouracil every 6-8 hours. The reduction in T3, which is 20-100 times more potent than T4, theoretically helps to reduce the thyrotoxic symptoms more quickly than does methimazole.

More severe cases require radioactive 131I thyroid ablation. Thyroidectomy is indicated for large goiters, pregnant patients or obstruction of the trachea. Patients who have undergone radioactive ablation or thyroidectomy become hypothyroid and are treated with PO levothyroxine.

Hypothyroidism is treated with levothyroxine (synthroid) – 1.6mcg/kg/d PO. Patients with myxedema coma need IV levothyroxine and IV hydrocortisone. Mechanical ventilation and warming blankets are required for hypoventilation and hypothermia, respectively.

o Levothyroxine – in active form, influences growth and maturation of tissues. Involved in normal growth, metabolism, and development. Produces stable levels of T3 and T4. Administered as a

Page 95: Objectives - Internal Medicine Define diastolic and systolic heart

single dose in the morning on an empty stomach. May be administered PO/IV/IM. Has long half-life (7-10 d), and parenteral dosing is rarely needed (except when PO is unavailable, patient is on continuous enteral feeds, or in emergency, such as myxedema coma). Initial subtherapeutic doses are recommended to avoid the stress of rapid metabolic change in elderly patients and in those with coronary artery disease or severe COPD.

o Liothyroxine (cytomel) – synthetic form of natural hormone T3 converted from T4. Used when a rapid effect is desired perioperatively or for nuclear medicine studies. Not intended for sole maintenance therapy.

Recognize radioiodine as a treatment for hyperthyroidism1) Antithyroid medications (methimazole, propylthioruracil – PTU): for hyperthyroidism in children and

teens, pregnancy (must watch carefully – can induce hypothyroid in fetus), severe Grave’s disease2) Radioactive iodine: management of choice for Grave’s disease, recurrent hyperthyr after antithyroid

meds, toxic multinodular goiter, toxic nodule in pt >40o Avoid children 2-4 days after tmto Avoid pregnancy 6 months

3) Subtotal thyroidectomy: for pregnant women, children intolerant to antithyroid meds, toxic nodule <40, large thyroid goiter causing local compression

Demonstrate a physical examination focused on jaundice

Jaundice (yellowish colour to skin and eyes) is cause by the accumulation of bilirubin in the body. This becomes visible when the bilirubin level is about 2 to 3 mg/dL. In short, bilirubin is produced when hemoglobin is broken down into heme and globin. The heme is processed to become unconjugated bilirubin which binds to albumin and is transported to the liver. In the liver, bilirubin becomes conjugated with glucuronic acid to become water soluble. The c-bilirubin is excreted into the bile which is excreted into the duodenum. In the intestine, the c-bilirubin is metabolized to become urobilinogen (some excreted into urine for the yellow colour and furthered transformed into stercobiligen for stools).

Physical exam for jaundice:- Vital signs for fever (hepatitis), toxicity (hypotension, tachycardia)- Cachexia, muscle wasting – suggests EtOH abuse, chronic disease- Sclera icterus or skin icterus- Kayser Fleischer rings for Wilson’s disease- Breath noted for fetor hepaticus- Ascites- portal hypertension (cirrhosis), chronic liver disease, cancer- Liver palpation – hepatomegaly, masses, tenderness- Presence of Murphy’s sign: cholecystitis- Spleen palpation – splenomegaly - Castell’s sign- DRE – looking for occult blood (hemolysis, GI bleed)- Testicular atrophy, gynecomastia- Dupuytryen’s contracture- Asterexis, altered mental status- Palmar erythema- Needle track marks (IVDU, hepatitis, drugs)- Xanthomas (primary biliary cirrhosis)- Skin – ecchymoses, petechia, purpura – coagulopathy- Skin hyperpigmentation – hemochromatosis, primary biliary cirrhosis

Page 96: Objectives - Internal Medicine Define diastolic and systolic heart

- Scratch marks – pruritis- Lymphadenopathy – lymphoma, leukemia, infectious mono

Establish a differential diagnosis for a patient with jaundice

PREHEPATIC (uncongugated hyperbilirubinemia)Resulting from Overproduction

o Hemolysis (spherocytosis, sickle cell disease, autoimmune disorders)o Ineffective erythropoiesis (megaloblastic anemia)o Hematomaso Pulmonary emboli

HEPATIC (unconjugated hyperbilirubinemia)Resulting from Decreased Hepatic Uptake

o Gilbert Syndrome (recessive disorder; decreased glucuronyl transferase)o Drugs (rifampin)o Neonatal jaundiceo Prolonged fasting

Resulting from Decreased Conjugation (limited glucuronyl transferase activity)o Gilbert Syndromeo Crigler-Najjar Syndrome (type 1 and 2)o Chronic, persistent hepatitiso Wilson disease

HEPATIC (conjugated hyperbilirubinemia)Resulting from Impaired Hepatic Excretion

o Familial disorders (Dubin-Johnson, Rotor, benign recurrent cholestasis, cholestasis of pregnancy)o Hepatocellular infiltrative disorderso Liver metastaseso Cirrhosiso Hepatits (viral, bacterial, autoimmune, ethanol and drug-induced)o Drug-induced cholestasis (many)o Primary biliary cirrhosiso Primary sclerosing cholangitiso Congestive heart failureo Others: shock, toxemia of pregnancy, sarcoidosis, amyloidosis, sepsis

EXTRAHEPATIC (conjugated hyperbilirubinemia)Resulting from extrahepatic biliary obstruction

o Gallstones, choledocholithiasiso Cholecystitiso Tumour of head of pancreas o Tumour of bil ducts (ex. Cholangiocarcinoma)o Tumour of duodenumo Biliary strictures

Interpret hepatic function tests

Page 97: Objectives - Internal Medicine Define diastolic and systolic heart

LFTs

INR/PT Marker of hepatic protein synthesis

o All clotting factors except factor VIII are exclusively synthesized in the liver Increased when hepatic protein synthesis is impaired Must exclude co-existing vitamin K deficiency

Serum Bilirubin Product of heme metabolism in the reticuloendothelial system of liver and spleen Must exclude extrahepatic causes of hyperbilirubinemia

Serum Albumin Level Detects prolonged hepatic dysfunction (weeks) Must exclude malnutrition and renal /GI losses, acute illness

Order of deterioration of LFTs1. Increase INR/PT2. Increase bilirubin3. Decrease albumin

Hepatobiliary Disease Lab Tests

AST & ALT Increase indicates hepatocellular damage

o Hepatitis (inflammation) or vascular injury (ischemia) If AST and ALT > 1000 = common bile duct stone, virus, drugs, ischemia, autoimmune hepatitis ALT>AST = viral hepatitis AST/ALT > 2 and AST< 300 = alcoholic liver disease

ALP & GGT Increases indicates cholestatic disease

o Biochemical cholestasis (drugs), systemic disease (sepsis), pregnancy, infiltrative disease (tumour, fat, lymphoma), mass lesions (stone, tumour, abscess)

Discuss the diagnosis of viral hepatitis

Acute viral hepatits = sudden onset of significant aminotransferase elevationvsChronic viral hepatitis = presence of persistent (> 6 m) necroinflammatory and fibrotic injury

Signs and symptoms1) Acute infection = asymptomatic2) Prodromal/pre-icteric phase

Anorexia, malaise, N/V, fever, RUQ pain (nonspecific) Occasionally, urticaria and arthralgia (especially in HBV)

3) Icteric phase (3-10 days) – lasts 1-2 weeks Dark urine Jaundice

Page 98: Objectives - Internal Medicine Define diastolic and systolic heart

Regression of systemic symptoms (pt feels better) Enlarged and tender liver (but the edge remains soft and smooth)

4) Recovery phase (lasts 2-4 weeks) Slow resolution of jaundice

** Acute viral hepatitis usually resolves spontaneously 4-8 weeks after symptoms onset

5) Chronic hepatitis Clinical features vary widely, often asymptomatic (especially in chronic HCV infection) Malaise, anorexia, and fatigue are common, sometimes with low-grade fever and nonspecific

upper abdominal discomfort Jaundice is usually absent First findings are signs of chronic liver disease: eg, splenomegaly, spider nevi, palmar

erythema (particularly with HCV) Sometimes cholestasis (rare) Chronic HCV occasionally associated with lichen planus, mucocutaneous vasculitis,

glomerulonephritis, porphyria cutanea tarda, and, perhaps, non-Hodgkin B-cell lymphoma.

Investigations1) Liver function test results compatible with hepatitis2) Viral serologic tests (see below)3) Possibly autoantibodies, immunoglobulins, and α1-antitrypsin level and other tests4) Serum albumin and PT5) Usually, biopsy (only for chronic hepatitis)

Hepatitis A serologyIgM anti-HAV IgG anti-HAV

Acute HAV + -Prior HAV* - +

* Previous HAV infections. HAV does not cause chronic hepatitis

Hepatitis B serology (1)- IgM antibody usually develops early in the infection and peaks at 1-2 weeks - IgG develops after 2 weeks and usually persist for life (indicate previous exposure)- HBsAg appears 1-6 weeks before clinical or biochemical illness develops, usually transient (implies infectivity

of the blood)- Anti-HBs appears weeks of months after infection and persist for life- Anti-HBc appears at the time of infection (reflects the viral core)

HBsAg Anti-HBs IgM anti-HBc

IgG anti-HBc HBeAg Anti-HBe HBV DNA (4)

Acute HBV + - + - ± - +Chronic HBV + - - + ± ± +Prior HBV (2) - + (3) - ± - ± -

(1) Antibody to hepatitis D virus (anti-HDV) levels should be measured if serologic tests confirm HBV and infection is severe(2) Previous HBV infection with recovery(3) Anti-HBs is also seen as the sole serologic marker after HBV vaccination(4) In pt with active HBV infection, HBV-DNA can be detected in the serum with special testing although this test is not routinely available** In 5-10% of pt, HBsAg persists and antibodies do not develop; these pt become asymptomatic carrier of the virus or develop chronic hepatitis

Hepatitis C serology- Anti-HCV usually appears within 2 weeks of acute infection but is sometime delayed

Page 99: Objectives - Internal Medicine Define diastolic and systolic heart

Anti-HCV (2) HCV-RNAAcute HCV + +Chronic HCV + +Prior HCV (1) + -

(1) Previous HCV infection with spontaneous recovery or successful treatment(2) Anti-HCV almost always implies active infection

Hepatitis D serology:- Presence of anti-HDV (which almost always implies active infection)- May not be detectable until weeks after the acute illness

Hepatitis E serology:- Test for IgM anti-HEV not routinely available- In pt with endemic exposure and compatible clinical findings, anti-HEV suggests acute HEV infection

Simplified diagnostic approach to possible acute viral hepatitis

List the main causes of cirrhosisCirrhosis represents the final common histologic pathway for a wide variety of chronic liver diseases. It is defined histologically as a diffuse hepatic process characterized by fibrosis and the conversion of normal liver architecture into structurally abnormal nodules. The progression of liver injury to cirrhosis may occur over weeks to years.

Most common causes of cirrhosis in the USA: Hepatitis C (26%)

Page 100: Objectives - Internal Medicine Define diastolic and systolic heart

Alcoholic liver disease (21%) Hep C + Alcoholic liver disease (15%) Cryptogenic causes (18%) Hepatitis B, which may be coincident with Hep D (15%) Miscellaneous (5%)

o Autoimmune hepatitiso Primary biliary cirrhosis o Primary sclerosing cholangitiso Hematochromatosiso Wilson’s Diseaseo Alpha-1 antitrypsin deficiency o Sarcoidosiso Drug-induced liver disease (e.g methotrexate, amiodarone)o Chronic right-sided heart failure

NASH (Non Alcoholic Fatty Liver Disease): Term refers to spectrum of disease – steatosis, steatohepatosis, fibrosis, cirrhosis Will proceed to cirrhosis in 10-20% of cases if you do nothing. Generally only a small number with

steatohepatosis progress to cirrhosis – possibly resulting from second insult). Associated with DM, obesity, hyperlipidemia; secondary causes include drugs (steroids, amiodarone, AZT,

CCBs), surgery, other (abetalipoproteinemia, SB bacterial overgrowth) No common mechanism has been identified – essentially a shift from lipolysis in liver to lipogenesis

(favoured with excess glucose in DM, TPN).

Describe the scope of complications related to cirrhosisSigns of cirrhosis: weak, sickly, fatigue, ↓ appetite, WL, clubbing, itchiness, yellow nodules (esp around eyelids), jaundice (if bile chronically blocked), ↓ absorption of fats and vitamins, ↓ libido, sclera icterus, Kayser-Fleischer ringsNote: many people asymp for yearsSigns of severe liver failure: muscle atrophy, palmar erythema, Dupuytrens contracture, spider nevi/Caput Medusa, enlargement of salivary glands in cheeks, peripheral neuropathy, gynecomastia, testicular atrophy, hair loss, splenomegaly, ascites, Lyndsey nails, asterixis, tremor, fetor hepaticus (sweet, pungent breath)Complications:- Portal HTN → esophageal/gastric/rectal varices- Cirrhotic ascites- Spontaneous bacterial peritonitis- Kidney failure- Hepatic encephalopathy- Osteoporosis: bc ↓ absorption of Vit D- Coagulation abnormalities: bc ↓ absorption of Vit K + ↓ platelets- Hepatocellular carcinoma: esp when cirrhosis due to chronic Hep B or C or alcoholism- Cholelithiasis (gallstone formation)- Pericardial effusion

Discuss the various treatments available for cirrhosis

Treat the underlying disorder EtOH cessation

Page 101: Objectives - Internal Medicine Define diastolic and systolic heart

Liver transplantation for end-stage liver disease if no EtOH for more than 6 months, use MELD stratification – Model for End Stage Liver Disease – predicts the 3 month survival based on creatinine, INR and total bilirubin

Follow patient for complications (think VARICES: Varices, Anemia, Renal failure, Infection, Coagulopathy, Encephalopathy, Sepsis)

To treat the following complications:o Portal hypertension: beta blockero Ascites: salt restriction, diuretic (blood has trouble passing through the liver and fluid leaks out of

blood vessels and into other tissues), paracentesis – may be therapeutic, maybe shunt or TIPSo Hepatic encephalopathy: lactulose, low-protein diet, maybe antibioticso Esophageal varices: octreotide to help stop bleed, balloon tamponnage, TIPS, antibioticso Hepatorenal syndrome: hemodialysis, colloid replacemento Hepatocellular carcinoma: resect, inject EtOH, cryosurgery, liver transplant

Interpret the results of paracentesis for ascites

General Pointso Serum ascites-albumin gradient has replaced the exudative-transudative classificationo An elevated serum ascites-albumin gradient = portal hypertensiono Ascites becomes clinically detectable with fluid accumulation greater than 500mL

Serum Ascites-Albumin GradientHigh (>1.1 g/dl) Low (<1.1 g/dl)CirrhosisChronic hepatic congestionRight ventricular heart failureBudd-Chiari syndromeConstrictive pericarditisMassive liver metastases

Peritoneal carcinomatosisPeritoneal tuberculosisPancreatic and biliary diseaseNephrotic syndrome

Discuss the diagnosis of pancreatitis

Acute H&P

o Hx. of common causes (See following objective)o Epigastric / left flank/back paino N & V, peritoneal signs, jaundice (compression / obstruction of bile duct), fevero Cullen / Grey-Turner signso Inglefingers sign: pain worse when supine, better sitting forwardo Tender rigid abdomen, guarding

Investigationso Increased amylase, lipase, liver enzymes, and leukocytosiso U/S may show multiple stones, edematous pancreaso AXR: dilated proximal jejunemo CT w contrast: contrast seen only in viable pancreatic tissue, non-viable areas can be biopsiedo ERCP / MRCP: searching for duct stone, tumour, pancreas divisum

Chronic H&P

Page 102: Objectives - Internal Medicine Define diastolic and systolic heart

o Usually hx. of alcoholism (less commonly CF, hereditary pancreatits)o Never gallstones – Acute pancreatitis onlyo Manifestations: abdominal pain, diabetes, steatorrhea, calcification of pancreas

Investigationso Flat plate: calcificationso U/S and CT: calcification, dilated pancreatic ducts, pseudocysto MRCP / ERCP: pancreatic duct narrowing or dilatationo 72h fecal fat test, fecal enzyme measurement

Secretin test, CCK test (exocrine function)

List the common causes of pancreatitis

Acute pancreatitsI GET SMASHED

Chronic pancreatitis

Most common

- Idiopathic - Gallstone (45%)- Ethanol (35%) - Drugs

o Furosemideo Methyldopao H2-blockerso Valproic Acido Antibioticso Acetaminopheno ASAo Etc

- Alcohol

Other - Tumors (pancrease, ampulla, choledochocele)

- Scorpion stings- Microbiological (bacterial, viral, parasites)

o Bacterial: mycoplasma, TB, Campylobacter, Legionella, etc.

o Viral: mumps, rubella, varicella, viral hepatitis, CMV, EBV, HIV, Coxsackie virus, etc.

o Parasites- Autoimmune

o SLE, Polyarthritis nodosa, Crohn’s- Surgery/trauma

o Manipulation sphincter of Oddi (ERCP), post-cardiac surgery, penetrating peptic ulcer, etc.

- Hyperlipidemia- Hypercalcemia- Hypothermia- Emboli or ischemia

- Cystic fibrosis- Severe protein-calorie malnutrition- Hereditary- Idiopathic

** Never gallstones, cause acute pancreatitis only

Page 103: Objectives - Internal Medicine Define diastolic and systolic heart

Describe an approach to a patient with chronic diarrheaApproach to chronic diarrhea.

The laboratory tools available to evaluate the very common problem of chronic diarrhea are extensive, and many are costly and invasive. As such, the diagnostic evaluation must be rationally directed by a careful history and physical examination, and simple triage tests are often warranted before complex investigations are launched.

History, physical examination, and routine blood studies:- characterize the mechanism of diarrhea- identify diagnostically helpful associations- assess the patient’s fluid/electrolyte and nutritional status. - question onset, duration, pattern, aggravating (especially diet) and relieving factors, and stool characteristics - determine presence or absence of fecal incontinence, fever,weight loss, pain, certain exposures (travel, medications, contacts with diarrhea), and common extraintestinal manifestations (skin changes, arthralgias, oral aphtha). - physical findings may offer clues such as a thyroid mass, wheezing, heart murmurs, edema, hepatomegaly, abdominal masses, lymphadenopathy, mucocutaneous abnormalities, perianal fistulae, or anal sphincter laxity- peripheral blood counts may reveal leukocytosis that suggests inflammation; anemia that reflects blood loss or nutritional deficiencies; or eosinophilia that may occur with parasitoses, neoplasia, collagen-vascular disease, allergy, or eosinophilic gastroenteritis.- blood chemistries may demonstrate electrolyte, hepatic, or other metabolic disturbances.

Patients suspected of having irritable bowel syndrome should be initially evaluated with proctosigmoidoscopy andmucosal biopsies; those with normal findings might be reassuredand, as indicated, treated empirically with antispasmodics, antidiarrheals,bulk agents, anxiolytics, or antidepressants.

Any patient who presents with chronic diarrhea and hematochezia should beevaluated with stool microbiologic studies and colonoscopy. In an estimated two-thirds of cases, the cause for chronic diarrhea remains unclear after the initial encounter, and further testing is required.

Quantitative stool collection and analyses If stool weight is >200 g/d, additional stool analyses should be performed that might include:

- electrolyte concentration,- pH- occult blood testing- leukocyte inspection (or leukocyte proteinassay) - fat quantitation- laxative screens- microbiologic studies should be done including fecal bacterial cultures (including media for Aeromonas

and Pleisiomonas),inspection for ova and parasites, and Giardia antigen assay (the most sensitive test for giardiasis)

- when suggested by history or other findings, screens for peptide hormones should be pursued (e.g., serum gastrin, VIP, calcitonin, and thyroid hormone/thyroid stimulating hormone, or urinary 5-hydroxyindolacetic acid and histamine)

- upper endoscopy and colonoscopy with biopsies and small-bowel barium x-rays are helpful to rule out structural or occult inflammatory disease.

-Further evaluation of osmotic diarrhea:

- tests for lactose intolerance and magnesium ingestion

Page 104: Objectives - Internal Medicine Define diastolic and systolic heart

Describe the investigations required in the assessment of a patient with possible celiac disease / malabsorptionDx: - Indications for testing: celiac disease in 1st or 2nd degree relative, thyroid disease, DM-I, Down Syndrome or

Turner’s syndrome, infertility, IBS, iron deficient anemia, chronic diarrhea, chronic fatigue, unintentional WL, short stature, osteoporosis, LFT abnormalities

- Antibody testing: anti-tissue transglutaminase Ab (TTG), IgA anti-endomysial Ab (EMA)If one or both tests positive = highly suggestive of celiac disease

- If both tests negative, but still suspect celiac disease → biopsy: flattened villi, with improvement after D/C gluten

Implement emergency treatment for upper gastrointestinal bleeding

- Stabilize patient: IV fluids, cross and type, 2 large bore IVs, monitor: “ABCs, IVs, O2, Monitor”- CBC, platelets, PT, PTT, electrolytes, BUN, creatinine, LFTs- NPO, maybe NG tube- Endoscopy (OGD): find site and treat lesion- epinephrine around bleeding points, injection of sclerosants,

argon plasma coagulation, bipolar electrocoagulation, endoclips, glue- Transfusion if required: PRBC, platelets- If stable non-variceal bleed and endoscopy not availabable treat with PPI (usually 80 mg IV load of

Pantoloc, OR high dose PO)- For variceal bleed, 50 ug octreotide as a loading dose followed by a 50ug/hr drip- Consider IV erythromycin prior to gastroscopy to remove clots from stomach

Page 105: Objectives - Internal Medicine Define diastolic and systolic heart

Establish the differential diagnosis for upper gastrointestinal bleeding based on age group

Upper GI BleedCondition Features Age groupEsophagitis Heartburn, dysphagia, odynophagia Middle-aged to older adultsEsophageal cancer Progressive dysphagia, weight loss Older adultsGastritis, Gastric Ulcer Aspirin, NSAID use Any ageDuodenitis, Dueodenal Ulcer

Abdominal pain, dyspepsia, H Pylori infection Any age

Gastric Cancer Early satiety, weight loss, abdominal pain Middle-aged to Older adultsEsophagogastric varices History chronic liver disease, stigmata on exam Middle-aged to Older adultsMallory-Weiss Tear History of retching before hematemesis Usually younger, but can be middle-

aged

Discuss the use of appropriate investigations for upper gastrointestinal bleeding

Vital signs: hypotension related to important blood lossCBC : Hemoglobin decrease can give indications of severity of extent of bleedingBUN : increase suggests upper GI bleed rather than lower GI bleedUpper endoscopy: to find source of bleeding

Distinguish between upper and lower GI bleeding

Upper GI bleeding Lower GI bleedingDefinition Bleed proximal to ligament of Treitz Bleed distal to ligament of TreitzClinical presentation

In order of severity of bleed:1) Hematochezia2) Hematemesis3) Melena4) Occult blood in stool

HematocheziaAnemiaOccult blood in stool(Rarely melena)

Etiology Esophagal varicesMallory-Weiss tearGastric ulcerGastritis

Other:- Epitaxis- Esophagitis- Esophageal/gastric cancer- Aortoenteric fistula- Coagulopathy- Vascular malformation

Colitis (radiation, infectious, ischemic)IBDHemorrhoids/fissureAngiodysplasiaNeoplasticDiverticular diseases

* Ligament of Treitz = suspensory ligament @ junction of duodenum and jejunum

Discuss the various pharmacological treatments available to prevent the recurrence of bleeding

Page 106: Objectives - Internal Medicine Define diastolic and systolic heart

Upper GI Hemorrhage:Treatment should be directed at underlying cause however a PPI +/- somatostatin (octreotide) is usually started empirically until endoscopy can performed to identify an UGI source

IV proton pump inhibitors (i.e. pantaprozole) – Increase pH >6 and thus promote clot stability (mechanism is through decreasing acid/pepsin on lesion and increasing platelet aggregation and fibrin formation)

IV somatostatin – only in cases of potential variceal bleeds (i.e. known cirrhosis, stigmata of chronic liver disease on exam). Mechanism of action – splanchnic vasoconstriction decreases portal hypertension

H2-recetor antagonists are also given for PUD Esophagastroduodenoscopy should be performed in ≤24 hours after presentation to diagnose source of

bleeding and determine potential risk of rebleeding. Hemostatic therapies depend on underlying pathology.

o Injection therapy – vasoconstrictors (I.e. epinephrine), saline, sclerosants, tissue adhesiveso Thermal therapy – mono/bipolar electrocoagulation, argon plasma coagulation, laser

photocoagulationo Mechanical therapy – clips, rubber bands

Forrest Class

Features Risk of rebleeding

Treatment?

1a Spurting 55% Hemostasis

1b Oozing 55% Hemostasis

IIa Visible vessel 45% Hemostasis

IIb Adherent Clot 20% Remove clot; treat underlying pathology

IIc Flat pigmented spot

10% Admit for monitoring over 24 hours

III 5% If healthy, discharge home

Dual therapies are superior to monotherapy (i.e. injection + clips vs. injection alone) Endoscopic intervention on peptic ulcer disease decreases risk of rebleeding, need for surgery and

mortality. If endoscopic therapy, performed patient admitted to hospital and treated with IV PPI x 72 hours (94% of re-bleeding occurs within 72 hours)then placed on oral PPI for 4-8 weeks. If patient is on NSAIDs, they should be stopped. Anticoagulants should also be held (if safe from CVS standpoint)

If endoscopy fails, do angiography (through interventional radio) where a catheter is introduced through femoral artery, passed to celiac trunk SMA and IMA branches to diagnose source of bleeding

For esophageal varices – treat with band ligation or injection of sclerosants. o Octreotide 50µg IV bolus and 50µg perfused IV/hr.

For gastric varices treat with “glue” or banding. If endoscopy fails, consider TIPS or liver transplant. Lower GI hemorrhage:

Diverticular bleed: Painless hematochezia usually from R-sided diverticula. 80% stop spontaneously and risk of re-bleed is 25% and risk of 2nd is 50% so surgical resection should be considered

Ischemic Colitis: painful hematochezia – treatment is supportive care. IV fluids can be given for dehydration and the patient is placed on bowel rest until symptoms resolve. Antibiotics are sometimes given to moderate or severe cases. The use of prophylactic antibiotics has not been prospectively evaluated in humans, but many authorities recommend their use based on animal data.

Angiodysplasia: Related to age and is usually asymptomatic. Treatment involves iron replacement, APC(argon plasma coagulation) if bleeding or anemic and estrogen/progesterone

Radiation Proctitis: APC

Page 107: Objectives - Internal Medicine Define diastolic and systolic heart

Hemorrhoids: The best way to prevent hemorrhoids is to keep stool soft so they can easily pass, thus decreasing pressure and straining. Recommend that patient takes Metamucil (fibe) capsules to soften their stool. Conservative treatment typically includes increasing dietary fiber, oral fluids to maintain hydration, NSAIDs, sitz baths, and rest. Preparation H (yeast extract is believed by many to remove wrinkles from skin and heal dry, cracked, and irritated skin) may improve local symptoms but does not improve the underlying disorder and longterm use is discouraged due to local irritation of the skin. Rubber band ligation or sclerotherapy may be recommended if hemorrhoids are grade 3-4

**Chron’s/Ulcerative colitis: Buts du traitement:

1. Induire la rémission2. Maintenir la rémission3. Augmenter la qualité de vie4. Éviter la toxicité à long terme

CONVENTIONAL DRUG THERAPIESFor many years, drug therapy of inflammatory bowel disease was limited to sulfasalazine and corticosteroids. Now a variety of oral and rectal aminosalicylates are available for first line therapy. Corticosteroids remain valuable agents for remission induction in moderate to severely ill patients, while budesonide, a rapidly metabolized steroid, may avoid many of the adverse effects of standard agents. For patients with Crohn’s disease, antibiotics have long been used to induce and at times maintain remission. These include metronidazole, quinolones, and others such as amoxacillin and clarithromycin. For patients who are refractory to the

above agents or who become steroid-dependent, immunomodulators are increasingly being used; many practitioners are beginning to use these drugs earlier in the course of therapy. 6- mercaptopurine and azathioprine are the best studied of these agents, but methotrexate, cyclosporine, and anti-TNF agents are used as well in selected patients. Finally physicians should not overlook the role of supportive agents to control diarrhea, abdominal pain, and the stress induced by these disorders.

Colite Ulcérative:Induire la rémission: 5-ASA, steroids, infliximab, cyclosporinMaintenir la rémission : 5-ASA, infliximab, azathioprine

Crohn’s : Induire la rémission: Steroids, anti-TNF, azathioprine, antibioticsMaintenir la rémission : anti-TNF, azathioprine, antibiotics

Define Crohn’s disease, ulcerative colitis and pseudomembranous colitisChron’s disease: chronic inflammation of intestinal wall that may affect any part of digestive tract- S&S: chronic diarrhea (can be bloody), crampy abdo pain, fever, ↓ appetite, WL, rectal bleeding when severeComplications:

Page 108: Objectives - Internal Medicine Define diastolic and systolic heart

- Scarring that can produce obstruction- Deep ulcers → abscesses- Fistulas with intestine, bladder, skin – especially around anus- Gallstones, ↓ absorption of nutrients, UTI, kidney stones, amyloidosis - Arthritis, episcleritis, aphtous stomatitis (mouth sores), erythema nodosum, pyoderma gangrenosumDx: colonoscopy and barium xrays

Ulcerative colitis: chronic disease in which large intestine becomes inflamed and ulcerated (pitted or eroded), leading to flare-ups (bouts or attacks), rarely affects small intestine, usually begins in rectum and spreads along intestineUlcerative proctitis: confined to rectum, common and benign- RF: 15-30, smoking ↓ risk, ↑ long-term risk of colon cancer- S&S (of flare-ups): bloody diarrhea, abdo cramps and feverComplications: - Bleeding: most common complication, → iron deficient anemia- Toxic colitis: damage to entire intestinal wall → ileus (intestinal paralysis) → distension- Toxic megacolon: distension of intestine, fever, pain and tenderness in abdo, ↑ WBC, risk of rupture- Colon CA- Other: arthritis, episcleritis (inflammation of whites of eyes), erythema nodosum, pyoderma gangrenosum

(blue-red skin sores containing pus)Dx: sigmoidoscopy

Pseudomembranous colitis= C diff induced colitis; usually caused by taking antibiotics (esp clindamycin, penicillins, cephalosporins, erythromycin, sulfonamides, chloramphenicol, tetracycline, quinolones)- S&S: usually begin 5-10 days after starting Ab, but can start after tmt stopped (up to 2 months); loose

stools/bloody diarrhea, abdo pain, fever - Dx: stool sample or sigmoidoscope- Tmt:

o If colitis while on Ab, D/C drugso Severe colitis: metronidazole, vancomycin for resistant or very severe cases

Describe the extra intestinal manifestations of inflammatory bowel disease

An acronym to remember these is ULCERATIVE COLITIS- Urinary calculi (will usually be an oxalate calculi)- Liver problems (fatty liver)- Cholelithiasis- Epithelial problems- Retardation of growth/sexual maturation- Arthralgias- Thrombophlebitis (thrombosis)- Iatrogenic complications- Vitamin deficiencies (osteoporosis)- Eyes- Colorectal cancer- Obstruction - Leakage (perforation)- Iron deficiency (anemia)- Toxic megacolon- Inanition (wasting)- Strictures, fistulae

Page 109: Objectives - Internal Medicine Define diastolic and systolic heart
Page 110: Objectives - Internal Medicine Define diastolic and systolic heart

Implement emergency treatment for an acute neurological deficit

MANAGEMENT OF ACUTE STROKE

Use of Anticoagulantso Anti-platelet therapy is treatment of choice (aspirin, dipyridamole, clopidogrel)o Anticoagulation (heparin) may be appropriate in certain patients (cervical artery dissection, basilar

thrombus, stroke in evolution)o No evidence to implement heparin in acute stroke situation (potential benefits offset by high risk of

intracranial hemorrhage)o Tests needed before instituting treatment

o CT (head) – rule out hemorrhageo Baseline PT, PTT, platelet count

Thrombolysiso If onset of stroke < 3 hours: consider IV recombinant tissue-type plasminogen activator (rt-PA) 0.9mg/kg

(max = 90mg)o Risk: 6% get symptomatic intracranial hemorrhageo Benefits: 50% chance of little or no disability at 3 months (38% chance without rt-PA)o (NB: benefits can be seen up to 4.5 hours post-stroke)

Hypertensiono Antihypertensive therapy if BP persistently > 220 systolic or > 120 diastolic

o More stringent guidelines if patients have received rt-PAo IV labetolol used because of short half-life

Page 111: Objectives - Internal Medicine Define diastolic and systolic heart

Cerebral Edemao If large territory is affected (MCA territory), transtentorial herniation can occur; this is called `Malignant

MCA Syndrome`o Treatment: airway intubation + mechanical hyperventilation to produce transient cerebral

vasoconstriction and may reduce ICP; mannitol can be beneficial, but is short acting; corticosteroids have no benefit

o Decompressive hemicraniotomy may be helpful

List and classify the main etiologies of stroke

Ischemic (80%) Thrombotic :

o Cerebrovascular thrombosis: in-situ clot on atherosclerotic lesiono Large vessel thrombosis (carotid, basilar, intracranial)

Embolic:o Thrombus most often formed in left atrium (A. fib), rare causes include fat or air

Lacunar: in small deep penetrating branches Other: hypercoagulable states, sickle cell, vasospasm

Hemorragic (20%) Spontaneous bleed Intracranial or subarachnoid hemorrhage

Demonstrate the ability to perform a neurological examination focused on the problem presented

CRANIAL NERVE EXAMINATIONCN1 – Olfactory

o Generally not tested. Can ask pt to close his eyes and test each nostril separately with coffee or mintCNII- Optic

o Visual acuity: Snellen charto Color vision and red desaturationo Visual fields: in each of four quadrantso Pupillary response: direct and consensual response, accommodationo Funduscopy: presence of red reflex (absence = corneal or lens opacity)

Optic disc (color, cup-to-dic ratio, symmetry, borders Retinal vessels (arteries and veins) Retina (flame hemorrhages?, white/gray/black spots?) Papilledema (optic disc congestion) – an early sign of ↑ ICP

CN III – Occulomotor, CN IV – Trochlear, CN VI - Abducenso Extraoccular mouvements (positions of gaze, horizontal and vertical)o Double vision? Saccadic mouvements? Nystagmus?o Horner’s (ptosis, miosis, facial anhidrosis)

CN V – TrigeminalMOTORo Observe for temporal wasting and jaw deviationo Palpate masseter and temporalis muscleo Ask pt to open his moutn against resistance and move jaw from side to side (pterygoinds)

Page 112: Objectives - Internal Medicine Define diastolic and systolic heart

o Jaw jerkSENSORYo Light touch and painV1, V2, V3 (and compare to other side)o Corneal reflex (afferent)

CN VII – FacialMOTORo Observe nasolabial folds for flattening, palpebral fissure for sagging and mouth for droopingo Ask pt to raise his eyebrows (frontalis), close his eyes tightly (orbicularis oculi), show his teeth

(buccinators), puff out the cheeks (orbicularis oris) and tense the neck muscles (platysma)SENSORYo Anterior 2/3 of tongue using sugar, salt and vinegar

CN VIII – Vestibulocochlearo Auditory acuityo Webber (lateralization to impaired ear for conductive and the opposite for sensorineural loss) o Rinne ( conductive loss, sound best transmitted by bone conduction, in sensorineural loss, sound best

transmitted through air)CN IX – Glassopharyngeal, CN X Vagus

o Palate elevationo Gag reflexo Swallowing o Dysarthria (also caused by lesions in CNs V, VII, XII, motor cortex, cerebellum, or basal ganglia)o Taste in posterior 1/3 of tongue

CN XI – Accessoryo Ask pt to shrug his shoulders against resistance (trapezius)o Ask the pt to turn his head to each side against resistance (SCM)

CN XII – Hypoglossalo Ask pt to stick his tongue out and move it from side to side (tongue pointing side of lesion)o Note fasciculation and atrophy

CEREBELAR DYSFUNCTIONHead

o Nystagmuso Oscillations in saccades or impaired smooth trackingo Assessment of speech o Head tilt

Trunko Truncal ataxiao Truncal tremor

Armso Tremor

Ask pt to stetch arms palms up and note if tremor worsen/improveso Finger to nose testing

For intention tremor or dysmetriao Holmes’ rebound phenomenono Rapid alternating movements o Assess writing

Becomes larger (cerebellar) or smaller (parkinsonian)o Motor: decreased tone and normal powero Reflexes: hyporeflexia, possibly with a pendular quality

Legso Heal-knee-shin

Page 113: Objectives - Internal Medicine Define diastolic and systolic heart

o Patellar tapo Ask pt to tap his foot on the ground (look for dysdiadochokinesia)o Motor: decreased tone and normal powero Reflexes: hyporeflexia, possibly with a pendular quality

Gait- Wide-based gait, with irregular steps and veering or falling to one side- Rhomberg

REFLEXESo 0 (none), 1+ (diminished), 2+ (N), 3+ (increased), 4+ (hyperactive, clonus)o Achilles /heel(S1-S2), Patellar/knee (L3-L4), Hamstring (L5), Brachioradialis (C5-C6), Biceps (C5-C6),

Triceps (C7)o Babinski’s sign (UMNL)o Primitive reflexes (Glabellar tap, Snout/suck/and root, Palmomental, Grasp-place, Hoffmann’s)o Cutaneous (Abdominal, Beevor’s sign, Cremasteric, Bulbocavernous, Anal wink)o Other: jaw jerk, corneal reflex

DIFFERENTIATE BETWEEN UMNL AND LMNLUMNL LMNL

Cranial nerves - Facial paralysis, forehead spared- Horner’s syndrome with anhidrosis

- Facial palsy involving forehead- Horner’s syndrome without anhidrosis

Motor bulk - None or slight atrophy - Focal atrophy, pronouncedMotor tone - ↑

- Rigidity (lead pipe, cogwheel)- Spasticity (clasp knife)

- ↓- Flaccidity

Motor power - Weak or absent (groups of muscles)- Pronator drift

- Weak or absent (focal)- Fasciculations

Reflexes - ↑- Plantar upgoing- Hoffmann’s sign- Posturing

- ↓- Plantar downgoing

Further examination that may assist differentiationSensory - ↓

- Stereoagnosis- Agraphesthesia- Neglect

- ↓ focally (root or peripheral nerve distribution)

Coordination - Ataxia- Dysmetria- Intention tremor

- Not affected

Gait - Romberg positive (vestibular) - Romberg positive (sensory)

Localize the anatomical lesion based on the clinical presentationThis objective encompasses a review of neuroanatomy and could be very long. A review of cortical and spinal anatomy is recommended but I will not outline it in detail below.

Page 114: Objectives - Internal Medicine Define diastolic and systolic heart
Page 115: Objectives - Internal Medicine Define diastolic and systolic heart

Discuss the use of appropriate investigations for strokeGeneral assessment: - ECG, telemetry and trop- Labs: CPC w platelet, chem8 (basic metabolic profile), PT, PTT- Other labs in select pts: LFT, urine tox, blood alcohol, pregnancy test, ABG, CXRNeuro assessment:- Alert possible thrombolytic candidate- Obtain Hx: determine onset of symptoms: consider thrombolytics w/in 3 hours of onset- Physical exam- Neuro exam:

o Assess LOC (Glascow Coma Scale)o Assess stroke severity: NIH stroke scale or Hunat and Hess scale

Rule-out hemorrhagic stroke:- Imaging:

o Urgent noncontrast head CT or MRIo Lateral neck xray if altered LOC or trauma

- Head CT suggests intracranial bleeding: immediate angiography, neurosurgery consult- Head CT negative: lumbar puncture (CI to thrombolytics) → to assess for subarachnoid blood (12 hours after

onset of symp)- Head CT negative suggesting ischemic CVA: consider thrombolytic tmt

Describe the role of antiplatelets, anticoagulants and thrombolysis in the treatment of an acute neurological deficit

- Thrombolysis: IV tissue plasminogen activator (tPA): within 3 hours of onset of symptoms as long as there are no contraindications, Dose: IV 0.9 mg/kgClass I recommendation by the American Stroke Association- Antiplatelets (aspirin, clopidogrel, dipyridamole) – shown to improve outcome in acute stroke if taken within 48 hours, can play a role in preventing a stroke, inhibit the production of thromboxane (produced by platelets) so platelets won’t come together- Anticoagulants: can play a role in preventing a stroke, these target clotting factors, competes with vitamin K hence preventing the formation of a clot

Discuss secondary prevention following an acute neurological deficit

Antiplatelet Therapyo Aspirin + / - dipyridamole or clopidogrelo NB: adding more agents doesn’t provide extra benefit for stroke prevention due to increased risk of

hemorrhage

Anticoagulationo Warfarin indicated for secondary prevention if

o Atrial fibrillationo Severely reduced ejection fractiono Valvular heart disease

Page 116: Objectives - Internal Medicine Define diastolic and systolic heart

Hypertensiono Target BP < 140/90o Best evidence for Ace-inhibitor + thiazide diuretic

Lipid-Lowering Agentso HMG-CoA reductase inhibitors (statins, ex. Lipitor, crestor)o All patients post-stroke should be treated regardless of cholesterol level

Perform a physical examination focusing on the cranial nerves, describing the significance of any findings

Nerve Name Function FindingsI Olfactory OlfactionII Optic Visual acuity, visual fields

Afferent limb of the pupillary light reflexIII Oculomotor Innervates

- medial/superior/inferior rectus muscle- inferior oblique muscle- levator palpebrae muscle

Efferent limb of pupillary light reflexInvolved in accommodation reflex

III, IV, VINystagmus should be notedEyes deviate towards the lesion

IV Trochlear Innnervated superior oblique musclesVI Abducens Innervates lateral rectus muscleV Trigeminal Muscles of mastication (masseter and temporal

muscles)V1 (ophthalmic)V2 (maxillary)V3 (mandibular)

VII Facial Innervates muscles of facial expressionTaste anterior 2/3 tongueSensory around pinna and external auditory canalEfferent limb of corneal reflex

Facial paralysis-entire side : LMN lesion (Bell’s Palsy)-lower face only (forehead sparing) – UMN lesion

VIII Vestibulo-cochlear

Hearing and vestibular function

IX Glosso-pharyngeal

Movement of the palateAfferent limb of gag reflexTaste posterior 1/3 tongue

The palate should be observed for uniform movementGag reflex should be tested on both sides

X Vagus Movement of the palateEfferent limb of gag reflexSwallowing

XI Accessory vagus

Innervates SCM and trapezius muscles

XII Hypoglossal Movement of the tongue AtrophyDeviation away from side of brain lesion

Perform a physical examination focusing on the peripheral nervous system, describing the significance of any findings

Page 117: Objectives - Internal Medicine Define diastolic and systolic heart

Physical examination of peripheral nervous system = motor and sensory assessment + reflexeso Single and multiple neuropathies = pain, weakness and parasthesia in the distribution of the affected

nerve or nerveso Pure motor nerve involvement = begins with painless weaknesso Pure sensory nerve involvement = begins with sensory disturbances and no weaknesso Multiple neuropathy = often asymmetric at onset, nerves can be involved all at once or progressively

Most common mononeuropathies + S&SULNAR NERVE PALSY- Paresthesias and sensory deficit in the 5th digit and medial half of the 4th digit- Thumb adductor, 5th digit abductor, and interosseus muscles are weak and may be atrophied- Severe chronic ulnar palsy causes a clawhand deformity** Sensory symptoms due to this syndrome are similar to those due to C8 root dysfunction secondary to cervical radiculopathy; however, radiculopathy normally affects the more proximal aspects of the C8 dermatome.

CARPAL TUNNEL SYNDROME (compression of the median nerve)- may be unilateral or bilateral- Paresthesias in the radial-palmar aspect of the hand and pain in the wrist and palm- Pain may be referred to the forearm and shoulder and may be more severe at night- Pain may be more severe at night- Sensory deficit in the palmar aspect of the first 3 fingers may follow, and the muscles that control thumb

abduction and opposition may become weak and atrophied

PERONEAL NERVE PALSY - Most common among emaciated bedbound patients and thin people who habitually cross their legs- Causes footdrop (weakened dorsiflexion and eversion of the foot)- Occasionally, a sensory deficit in the anterolateral aspect of the lower leg and the dorsum of the foot or in the

web space between the 1st and 2nd metatarsals.

RADIAL NERVE PALSY (Saturday night palsy) - Wristdrop (weakness of the wrist and finger extensors) - Sensory loss in the dorsal aspect of the first dorsal interosseous muscle.

Multiple mononeuropathy (mononeuritis multiplex) is usually secondary to connective tissue disorders (eg, polyarteritis nodosa, SLE, other types of vasculitis, Sjögren's syndrome, RA), sarcoidosis, metabolic disorders (eg, diabetes, amyloidosis), or infectious disorders (eg, Lyme disease, HIV infection, leprosy). However, diabetes usually causes sensorimotor distal polyneuropathy

PolyneuropathiesDiffuse peripheral nerve disorder, usually bilateral and symmetrical and thus not confined to the distribution of a single nerve or a single limb- Some polyneuropathies (eg, due to lead toxicity, dapsone use, tick bite, porphyria, or Guillain-Barré syndrome)

affect primarily motor fibers- Others (eg, due to dorsal root ganglionitis of cancer, leprosy, AIDS, diabetes mellitus, or chronic pyridoxine

intoxication) affect primarily sensory fibers- Some disorders (eg, Guillain-Barré syndrome, Lyme disease, diabetes, diphtheria) can also affect cranial nerves- Certain drugs and toxins can affect sensory or motor fibers or both

Polyneuropathies usually classified by area of dysfunction:

MYELIN DYSFUNCTION NEUROPATHIES

Page 118: Objectives - Internal Medicine Define diastolic and systolic heart

- Most often result from a parainfectious immune response triggered by an encapsulated bacterium (eg, Campylobacter sp), virus (eg, enteric or influenza viruses, HIV), or vaccine (eg, influenza vaccine).

- Acute (eg, in Guillain-Barré syndrome) = rapidly progressive weakness and respiratory failure may develop- Chronic inflammatory demyelinating polyneuropathy (CIDP) = symptoms may recur or progress over months

and years- S&S:

o Large-fiber sensory disturbances (paresthesias)o Significant muscle weakness greater than expected for degree of atrophyo Greatly diminished reflexeso Trunk musculature and cranial nerves may be involvedo Distal and proximal signs and symptoms (demyelination typically occurs along the entire length of a

nerve) o May be asymmetricalo Upper body may be affected before the lower body, or vice versao Muscle bulk and tone are relatively preserved.

VASA NERVORUM COMPROMISE- Chronic arteriosclerotic ischemia, vasculitis, and hypercoagulable states can compromise the vascular supply

to nerves- S&S:

o Usually, small-fiber sensory and motor dysfunction occurs firsto Painful, often burning sensory disturbanceso Abnormalities tend to be asymmetric early in the disorder and rarely affect the proximal 1⁄3 of the

limb or trunk muscleso Cranial nerve involvement is rare, except in diabetes, which commonly affects the 3rd cranial

(oculomotor) nerveo Dysautonomia and skin changes (eg, atrophic, shiny skin) sometimes occuro Muscle weakness tends to be proportional to atrophyo Reflexes are rarely lost completely.

AXONOPATHIES - Tend to be distal- May be symmetric or asymmetric

o Symmetric axonopathies result most often from toxic-metabolic disorders: Diabetes mellitus Chronic renal insufficiency Adverse effects of chemotherapy drugs (eg, vinca alkaloids) Nutritional deficiencies (most commonly, of thiamin or vitamin B6, B12, or E Excess intake of vitamin B6 or EtOH Less common: hypothyroidism, porphyria, sarcoidosis, and amyloidosis. Other causes include certain infections (eg, Lyme disease), drugs (eg, nitrous oxide), and

exposure to certain chemicals (eg, Agent Orange, n-hexane) or heavy metals (eg, lead, arsenic, mercury). In a paraneoplastic syndrome associated with small-cell lung cancer, loss of dorsal root ganglia and their sensory axons results in subacute sensory neuropathy

- S&S:o May begin with symptoms of large- or small-fiber dysfunction or botho Usually, the resulting neuropathy has a distal symmetric, stocking-glove distributiono Evenly affects the lower extremities before the upper extremities and progresses symmetrically from

distal to proximal areas- Asymmetric axonopathy can result from parainfectious or vascular disorders

Page 119: Objectives - Internal Medicine Define diastolic and systolic heart

Demonstrate an approach to history and physical examination for a patient with a mononeuropathyApproche clinique d'une neuropathie

Avant de considérer des étiologies précises, le clinicien doit obtenir l'histoire du cas et procéder à un examen physique pour pouvoir caractériser le processus neuropathique en ce qui concerne les aspects suivants :

1. Profil temporelselon le temps écoulé entre l'apparition de la maladie et le point de déficit maximal : profil aigu (de quelques jours à 6 semaines), subaigu (de 6 semaines à 6 mois), chronique (de 6 mois à plusieurs années).

2. Distribution anatomiqueradiculopathie ou polyradiculopathie (une ou plusieurs racines affectées), mononeuropathie (un seul nerf périphérique), mononeuropathie multiple (plusieurs nerfs individuels, souvent dans des membres différents), plexopathie, neuropathie multiple.Le terme neuropathie multiple (par opposition à mononeuropathie multiple) est à employer pour une atteinte diffuse et symétrique de nerfs périphériques. Règle générale, les axones les plus longs sont affectés les premiers, ce qui entraîne une perte de sensation dans la partie antérieure du pied et une atrophie des muscles intrinsèques du pied. Le déficit se déplace progressivement à mesure que des axones de plus en plus courts sont affectés. Habituellement, les mains sont touchées lorsque le déficit sensoriel atteint les genoux (distribution en gants et en chaussettes ). Dans les cas graves, même le tronc peut être atteint, selon une distribution en bouclier , (à partir de la ligne médiane antérieure, où se terminent les branches des rameaux dorsaux segmentaires circonférentiels).

3. Types de fibres nerveuses affectéesfibres motrices (atrophie et faiblesse musculaires, fasciculations), sensitives (fibres de grand diamètre : les sensations de vibration, de position des articulations et de toucher discrimatif sont affectées ; fibres de petit diamètre : les sensations de douleur et de température sont affectées), fibres sympathiques sudomotrices (anhidrose), fibres sympathiques vasomotrices (vasodilation).

4. Affection des cellules de Schwann (démyélinisation) ou des axones (axonopathie)Cette distinction est difficile à établir sur des bases purement cliniques. Les neuropathies démyélinisantes tendent à présenter assez tôt une perte de réflexes, à affecter surtout les grosses fibres et à entraîner moins d'atrophie musculaire (atrophie de non-utilisation plutôt que de dénervation). Des études de la conduction nerveuse sont particulièrement utiles dans ce contexte : la vitesse de conduction diminue en cas de démyélinisation, alors que l'amplitude est faible mais que la vitesse de conduction est relativement normale en cas d'axonopathie.

D. Exemples cliniques de neuropathie

1. RadiculopathieLes causes les plus importantes de radiculopathie sont l'hernie d'un disque intervertébral ou une maladie dégénérative chronique de la moelle épinière avec formation d'ostéophyte. Une radiculopathie peut également résulter d'un traumatisme, d'une infection ou d'une invasion néoplasique. Le syndrome typique est une douleur radiculaire, une perte de sensation ou une paresthésie dans un dermatome et une faiblesse dans le myotome correspondant. La douleur irradiante ne permet pas de localiser une radiculopathie d'une manière aussi certaine : elle est souvent plus diffuse et peut être due à une spondylarthrose de la colonne vertébrale sans atteinte directe des racines. La douleur radiculaire est aggravée par les mouvements de la colonne vertébrale, la toux et l'éternuement. La paresthésie d'un dermatome est davantage le signe de lésions radiculaires, même si la perte de sensation peut être difficile à établir à l'examen. Dans la région cervicale, 90 % des modifications dégénératives des disques se produisent entre C5 et C6 ou entre C6 et C7 (et affectent donc respectivement les racines C6 et C7). Dans la région lombaire, 90 % des protrusions se produisent entre L4 et L5 ou entre L5 et S1 (et affectent donc

Page 120: Objectives - Internal Medicine Define diastolic and systolic heart

respectivement les racines L5 et S1). Noter que dans la région lombaire la racine affectée par une hernie discale est généralement celle qui sort sous le disque en cause.

C6 Paresthésie : pouce et indexParésie : biceps, muscle deltoïde, muscle rond pronateurDiminution des réflexes : biceps et muscle brachio-radial

C7 Paresthésie : majeur (mais douleur radiculaire diffuse dans le membre supérieur)Faiblesse : triceps, muscles extenseurs du poignet et des doigtsDiminution des réflexes : triceps

L5 Paresthésie : dos du pied, gros orteil, côté extérieur du molletFaiblesse : dorsiflexion de la cheville (muscle tibial antérieur), éversion de la cheville (muscles fibulaires), rotation de la cheville vers l'intérieur (muscle tibial postérieur), abduction de la hanche (muscle moyen fessier)Réflexes : normaux

S1 Paresthésie : arrière du mollet, côté extérieur de la cheville et du piedParésie : flexion plantaire, extension de la hanche (muscle grand fessier)Diminution des réflexes : cheville

2. Mononeuropathie aiguë

a) Nerf radial dans le haut du brasLe nerf radial est typiquement comprimé contre l'humérus pendant le sommeil profond (une fatigue extrême et l'alcool sont des facteurs prédisposants, d'où l'expression populaire *paralysie des amoureux +). Le patient peut se plaindre d'une perte de sensation ou de paresthésie sur le dos de la main et présente une faiblesse du muscle brachio-radial ainsi que des muscles extenseurs du poignet et des doigts. Le triceps est généralement épargné puisque sa branche nerveuse est proximale par rapport au site de compression. La lésion est du premier degré (neurapraxie) et guérit en 6 à 12 semaines.

b) Nerf fibulaire commun à la tête de la fibula (péroné)

À cet endroit, le nerf est superficiel et comprimé par le croisement des jambes, un repos prolongé au lit ou un accroupissement prolongé. Il y a perte de sensation du côté antérolatéral de la partie inférieure de la jambe et sur le dos du pied. Il y a faiblesse de dorsiflexion et d'éversion du pied ainsi que d'extension des orteils, ce qui se traduit par un pied tombant et une démarche claquante. La rotation de la cheville vers l'intérieur (principalement par le muscle tibial postérieur) est épargnée, contrairement à que l'on observe en cas de radiculopathie L5.

Page 121: Objectives - Internal Medicine Define diastolic and systolic heart

3. Mononeuropathie chronique

Il s'agit le plus souvent de *syndromes canalaires +, ou lésions nerveuses par englobement d'un nerf, dues à la compression chronique d'un nerf dans un canal fibro-osseux étroit.

a) Nerf médian au poignet : syndrome du canal carpien Le patient se plaint de paresthésie intermittente, souvent douloureuse, des 3 premiers doigts, qui affecte typiquement en premier lieu la main dominante et se produit la nuit. Dans les cas graves, elle s'accompagne d'une atrophie de l'éminence thénar et d'une faiblesse du muscle court extenseur du pouce (abduction palmaire du pouce) et du muscle opposant du pouce. Le nerf médian peut présenter un signe de Tinel. La section chirurgicale du rétinaculum des fléchisseurs, qui constitue la limite antérieure du canal carpien, corrige généralement le problème.

b) Nerf cubital au coudeDans ce cas-ci, il y a compression chronique du nerf ulnaire dans le sillon condylien ou à l'endroit où il entre dans l'avant-bras sous l'aponévrose du muscle fléchisseur ulnaire du carpe (canal du nerf cubital). Il y a perte de sensation dans les 2 derniers doigts et du côté ulnaire de la paume. Il y a également atrophie et parésie des muscles interosseux et des 2 muscles lombricaux médiaux, ce qui peut se traduire par une main en griffe (doigts fléchis) du côté médial.

c) Nerf cutané latéral de la cuisseCe nerf purement sensitif transporte les fibres sensitives cutanées du côté latéral de la cuisse. Il entre dans la cuisse sous la partie latérale du ligament inguinal, où il peut devenir comprimé (p. ex. grossesse, obésité). Le patient se plaint de paresthésie douloureuse du côté latéral de la cuisse X méralgie paresthésique.

4. Mononeuropathie multiple

L'étiologique la plus importante à considérer est une ischémie nerveuse à foyers multiples qui peut se produire en cas de vascularite (p. ex., polyartérite noueuse, vascularite rhumatoïde, granulomatose de Wegener) ou de micro-angiopathie du diabète.

Demonstrate an approach to history and physical examination for a patient with a polyneuropathyPolyneuropathy: disease of multiple peripheral nerves simultaneously, usually symmetrical, bilat distal motor and sensory impairment; can be acute or chronic

Important questions to ask in Hx:- Medications: phenytoine, antibiotics (chloramphenicol, nitrofurantoin, sulfonamindes), chemotherapy

(vinblastine, vincristine), sedatives (barbital, hexobarbital)- Past medical Hx:

o Diabetes – most commono Excessive use of alcohol/excessive nutritional deficiencies (thiamin)o Anemia due to vitamin B12 deficiencyo Hypothyroidismo Liver or kidney failureo CA, esp lung cancer an multiple myeloma (direct nerve damage)o Vitamine B5 (pyridoxine) in excess

Page 122: Objectives - Internal Medicine Define diastolic and systolic heart

- Review of systems: check symptoms in autonomic nervous system – constipation, fecal/urinary incontinence, sexual dysfunction, fluctuating BP (esp orthostatic hypotension), skin pale and dry, ↓ sweating

Physical exam: - Determine if motor or sensory or both are involved, +/- cranial nerves- Acute polyneuropathy (eg. Guillain-Barre): begins suddenly in both legs and progresses rapidly upward to arms

→ look for weakness, pins and needles or loss of sensation- Chronic: most commonly only sensation affected, usually feet fist → look for pins and needles, numbness,

burning pain, loss of vibration sense and position sense

Define monoarthritis, oligoarthritis and polyarthritis

Monoarthritis: inflammation of 1 joint at a time (ex: gout, pseudogout, trauma, infection, septic arthritis)Oligoarthritis: inflammation (arthritis), affecting 1 to 4 joints, most commonly large joints are involvedPolyarthritis: arthritis involving 5 or more joints (SLE, RA, psoriasis = a lot of auto-immune disorders, viral, lyme, bacterial endocarditis)

Describe the signs and symptoms of monoarthritis / Distinguish the differences between mechanical and inflammatory arthritis (2 obj combinés)

NB: non-inflammatory arthritis = degenerative, traumatic, mechanical

INFLAMMATORY NON-INFLAMMATORYPain (NB: pain from small joints – localized; pain from large joints – diffuse)

at rest and with motion; worse at the beginning of usage

pain mainly during motion and improves with rest; NB: patients with advanced degenerative disease of large joints may have pain at rest and at night

Stiffness present upon waking and lasts 30-60 minutes or longer

brief stiffness (15 min) upon waking or after periods of inactivity

Swelling joint swelling relates to synovial hypertrophy, synovial effusion and inflammation of periarticular structures; degree of swelling varies over time

formation of osteophytes leads to bony swelling (gnarled fingers, knobby knees); mild degree of soft-tissue swelling

Limitation of Motion Present PresentFatigue With polyarthritis: in afternoon and

early eveningAbsent

Temporal Pattern Insidious – develop over weeks to months

OA – develop over weeks to monthsTrauma, crystalline, infection – acute over minutes to hours

Duration of Symptoms Chronic – more than 6 weeks Acute or chronicSymmetry – Joint Involvement RA, SLE – Symmetrical

Reiter’s, Psoriatic – asymmetricAsymmetric

Distribution of Affected Joints Ulnar deviation at MCP; swan neck (DIP), boutonnière (PIP); wrist (fork-like deformity); Bouchard’s node (PIP); carpal tunnel; interosseous

DIP and PIP involvement; Heberden node (DIP), Bouchard’s nodes (PIP); weight-bearing joints

Page 123: Objectives - Internal Medicine Define diastolic and systolic heart

atrophy; spondylarthropathies (back involvement)

Extra-articular manifestations Constitutional – fatigue, malaise, weight lossSkin lesions – rashes, nodulesOcular symptoms – scleritis, uveitis

Usually absent

Synovial Hypertrophy Present AbsentErythema / Warmth Rare – RA

Common – gout, septicUsually absent

Establish the differential diagnosis for monoarthritis

Trauma Gout Pseudogout Infectious/septic (most often gonococcal, staph) Osteoarthritis Psoriatic

List the risk factors for gout

Risk factors = everything that may lead to hyperuricemia + other

1) Obesityo Being overweight is a risk factor for developing gout. Being overweight can lead to excess

uric acid production.2) Eating Purine-Rich Foods3) Heavy or even moderate alcohol 4) HTN5) Lead Poisoning6) Older age7) Family History

o Up to 18% of people with gout have a family history associated with gou8) Gender

o More men than women have gout. Women generally do not develop gout until they are postmenopausal

9) Abnormal Kidney Function10) Drugs

o thiazide diuretics, ASA, cyclosporine, Niacin, Levodopa, Aminophylline, medications to treat tuberculosis

11) Certain Medical Conditionso Leukemia, lymphoma, hemoglobin disorders, hypothyroidism

Discuss the differences between gout and pseudogoutGout is inflammation caused by monosodium urate monohydrate (MSU) crystals.

MSU crystals are formed in synovial fluid when the fluid becomes supersaturated with MSU. This can result from overproduction or reduced excretion of MSU. A genetic predisposition for the disease exists.

Page 124: Objectives - Internal Medicine Define diastolic and systolic heart

Crystals of MSU appear as needle-shaped intracellular and extracellular crystals. When examined with a polarizing filter, they are yellow when aligned parallel to the axis of the red compensator, but they turn blue when aligned across the direction of polarization (ie, they exhibit negative birefringence).

Gouty attacks are triggered by crystal formation in synovial fluid. They are not related to serum levels of uric acid. Thus, a normal serum uric acid level does not exclude the diagnosis of acute gout, and an elevated level does not prove the diagnosis.

Form of arthritis that occurs suddenly, usually affecting the big toe, and causes severe painful attacks, tenderness and redness in the affected joint. An attack can last days or weeks.

A buildup of uric acid can result from: high alcohol consumption, eating excessive food rich in purines such as meats, crash diets, surgery, chemotherapy

Pseudogout is inflammation caused by calcium pyrophosphate (CPP) crystals and is sometimes referred to as calcium pyrophosphate disease (CPPD).

CPP crystals are produced by nucleoside triphosphate pyrophosphohydrolase (NTPPPH), a catalytic enzyme found in vesicles that develop within osteoarthritic cartilage. A genetic predisposition exists for the condition, but any process that leads to osteoarthritis also can be associated with subsequent pseudogout.

It causes a sudden extreme attack of pain and swelling to specific joints, which can last days- weeks. It is usually present in the knees, but can also affect the shoulder, elbows, wrists, hands, or ankles. It can become a chronic arthritic condition that feels more like osteoarthritis or rheumatoid arthritis.

Related to hypothyroidism, hemochromatosis, overactive parathyroid, hypercalcemia, joint injury, surgery CPP crystals appear shorter and often rhomboidal. Under a polarizing filter, CPP crystals do not change

color depending upon their alignment relative to the direction of the red compensator. Attacks can be triggered by many metabolic abnormalities.

List the different etiologies of septic arthritis based on age group- Newborn (contiguous spread from osteomyelitis): staph aureus, enterobacter, group B strep, N gonorrhea- Children (<15): staph aureus, strep pyogenes, strep pneumonia, H influenza, gram negative bacilli- Young sexually active adults: N gonorrhea (esp common in women), staph aureus, strep- Older adults: staph aureus (50%), strep, gram negative bacilli

Discuss antibiotic treatment of septic arthritis.

This is a MEDICAL EMERGENCY!!!! Start IV antibiotics empirically – delay may result in joint destruction! Use cloxacillin or cefazolin + ciproflaxin or gentamicin is risk for gram negative before culture results come

back Gram stain Gonococcal: ceftriaxone 1 g q24h IM or IV; if penicillin sensitive: ampicillin 1 g q 6h IV, usually 2-4 days IV

and then 7 days PO Non-gonococcal: antibiotics against S.Aureus, strep x 2 week IV then 2-4 weeks PO Surgical drainage if: persistent positive joint cultures, hip involvement Daily aspirations until culture sterile physiotherapy

Interpret synovial fluid results from a joint aspiration

Page 125: Objectives - Internal Medicine Define diastolic and systolic heart

Normal Non-inflamm Inflammatory SepticAppearance Clear Clear/yellow Clear to opaque,

yellow/whiteOpaque

WBC/mm3 <200 <2000 >2000 >2000 (>50k)Polys <25% <25% >50% >75%Culture Neg Neg Neg PositiveConditions OA RA, crystal,

spondyloarth.infection

Discuss appropriate treatment for inflammatory monoarthritis

Treatment depends on etiology. Most common causes of inflammatory monoarthritis are listed below.

Infectious arthritis- Confirmed bacterial arthritis should be treated with prompt antibiotic therapy + repeat joint drainage to

minimize joint destruction- The choice of antibiotic is based on risk factors, such as the patient's age and immunocompetence, whether

the infection was hospital acquired, as well as the results of Gram's staining, if availableo If nongonococcal arthritis is suspected → antibiotics against S. aureus and streptococci should be

usedo In the immunocompromised patient Gram-negative and anaerobic bacteria should also be targetedo All antibiotics should be given intravenouslyo Generally, IV antibiotics should be continued ≥ 2 weeks and followed by 2-4 weeks of oral antibioticso Suspected gonococcal arthritis is treated with ceftriaxone (1 g/day, intramuscularly or intravenously).

If the organism is reported to be sensitive to penicillin, either ampicillin (1 g every 6 h, intravenously) or penicillin G (10 million U daily in divided doses, intravenously) should be given

Parenteral therapy should be maintained until there is clinical improvement of skin and joints (usually 2-4 days) and followed by oral antibiotics to complete a 7-day treatment.9

Crystal-induced arthritis- If acute: 1) NSAIDs 2) corticosteroids or, rarely, 3) colchicines

o The sooner the drug therapy is initiated, the quicker the patient will respondo NSAIDs are used as first-line agentso If NSAIDs are contraindicated corticosteroids (usually intra-articular) are a safe alternativeo A short course of oral prednisone can be given at a dosage of 30-50 mg daily, tapered rapidly over 5-

10 days- Long-term management of gout involves treating hyperuricemia through risk modification and, once the acute

episode of gout has been well controlled, drug therapyo Allopurinol is the drug of choice — 300 mg daily with dose reduction for renal insufficiency. o Colchicine prophylaxis consists of 0.6 mg once or twice daily and should be continued until the

patient has been attack free for 6 months or until the tophi have lysed. o For patients with allopurinol allergy, uricosuric agents such as probenecid or sulfinpyrazone could be

considered if the urinary uric acid excretion is less than 800 mg/day

Distinguish the findings in the examination of the hands of a patient with rheumatoid arthritis and osteoarthritisRheumatoid Arthritis:

Page 126: Objectives - Internal Medicine Define diastolic and systolic heart

Stiffness, swelling, and pain are symptoms common to all forms of arthritis in the hand. In RA, some joints may be more swollen than others. There is often a sausage-shaped (fusiform) swelling of the finger. Other symptoms include:

Soft lump over back of hand that moves with tendons that straighten fingers Firm, non-tender, subcutaneous nodules develop. These are commonly found at pressure points like the

elbow, back of the forearm, and MCP joint. The rheumatoid nodule, which is often subcutaneous, is the cutaneous feature most characteristic of rheumatoid arthritis. The initial pathologic process in nodule formation is unknown but may be essentially the same as the synovitis, since similar structural features occur in both. The nodule has a central area of fibrinoid necrosis that may be fissured and which corresponds to the fibrin-rich necrotic material found in and around an affected synovial space. Surrounding the necrosis is a layer of palisading macrophages and fibroblasts, corresponding to the intimal layer in synovium and a cuff of connective tissue containing clusters of lymphocytes and plasma cells, corresponding to the subintimal zone in synovitis. The typical nodule may be a few millimetres to a few centimetres in diameter and is usually found over bony prominences, such as the olecranon, the calcaneal tuberosity, the metacarpophalangeal joint, or other areas that sustain repeated mechanical stress. Nodules are associated with a positive RF (rheumatoid factor) titer and severe erosive arthritis. Rarely, these can occur in internal organs or at diverse sites on the body.

Palmar erythema Erythema nodosum Crepitus with movement A shift in position of fingers as they drift away from direction of the thumb (ulnar deviation) Swelling and inflammation of the tendons that bend the fingers , resulting in clicking or triggering of the

finger as it bends, and sometimes causing carpal tunnel Boutonnière deformity: deformity in which the PIP of the finger becomes bent and the DIP hyperextended Swan neck deformity "Z-thumb" or "Z-deformity" may occur in rheumatoid arthritis. It consists of hyperextension of the

interphalangeal joint, and fixed flexion and subluxation of the metacarpophalangeal joint and gives a "Z" appearance to the thumb; however, this finding is also typical of osteoarthritis of the thumb as well.

Increased stiffness early in the morning is often a prominent feature of the disease and typically lasts for more than an hour. Gentle movements may relieve symptoms in early stages of the disease. These signs help distinguish rheumatoid from non-inflammatory problems of the joints, osteoarthritis.

RA affects MCP and PIP joints!

Osteoarthritis: OA affects DIP joints and is frequently associated with Heberden’s nodules. Osteoarthritic enlargements of the DIP and PIP are referred to as Herberden’s and Bouchard’s nodes,

respectively. The first carpometacarpal joint is also a common area affected in OA. Enlargement of this joint results in a squared appearance in the hand.

The carpometacarpal joint of the thumb is typically involved in OA, rather than the proximal interphalangeal joint in RA.

Page 127: Objectives - Internal Medicine Define diastolic and systolic heart

Swelling of the joints is hard and bony in OA; in comparison, soft, warm, and tender joint swelling is typical of RA.

Stiffness of the joint is a very common feature of RA, but is a relatively rare feature of osteoarthritis. Furthermore, the stiffness of RA is characteristically worse after resting the joint (eg, morning stiffness), while the stiffness of osteoarthritis (if present) is typically worse after any effort, and is often described as evening stiffness

OA is characterized radiographically by narrowing of the joint space due to cartilage loss and osteophytes due to bone remodeling, but not erosions or cysts.

Demonstrate maneuvers to elicit the findings of a joint effusion1) Shoulder: glenohumeral joint effusions may cause bulge between coracoids process and humeral head →

palpate2) Knee:

a. Bulge sign: knee fully extended, stroke medial aspect of knee to express any fluid away from areab. Balloting patella for larger effusions

3) Arthrocentesis: withdraw fluid from effusion + fluid assessmenta. Gross examination: color, turbidity, viscosity (normal length of viscous string of joint fluid dropped

from syringe >3cm; inflammation ↓ viscosity = shorter string length)b. Lab: culture, PMN, WBC count

Examination Normal Hemorrhagic Infectious Inflammatory Noninflammatory

Gross examination

Appearance Clear Bloody Turbid or purulent

Yellow, cloudy Straw-colored, clear

Viscosity High Variable Variable Low High

Routine laboratory examination

Culture Negative Negative Often positive Negative Negative

PMN %* < 25 — Usually >85 > 50 < 25

WBC count* <200/μL Affected by amount of blood

5000 to >100,000/μL

1000 to 50,000/μL

200 to 1000/μL

List the diagnostic criteria for rheumatoid arthritis

At least 4 of the following 7 criteria:1. morning stiffness: lasting at least 1 hour before maximal improvement2. at least 3 joint areas have simultaneously had soft tissue swelling or fluid (not bony overgrowth) observed

by a physician3. arthritis of hand joints: at least 1 joint area swollen in wrist, MCP or PIP joint4. symmetrical arthritis: bilateral involvement of PIP, MCP, or MTP joints is acceptable without absolute

symmetry5. rheumatoid nodules: over bony prominences or extensor surfaces or in juxta-articular regions that are

observed by a physician6. RF positive (rheumatoid factor)

Page 128: Objectives - Internal Medicine Define diastolic and systolic heart

7. radiographic changes: changes typical of rheumatoid arthritis include erosions or unequivocal bony decalcification localized to or most marked adjacent to involved joints

Establish the differential diagnosis for polyarthritis

Rheumatoid arthritis Osteoarthritis Osteomalacia Gout / pseudogout Septic arthritis SLE Reactive arthritis (Reiter’s) Post-viral arthritis Trauma / injury Spondyloarthropathy (assoc: ankylosing spondylitis, Reiter’s, IBD, psoriatic) Lyme disease Sarcoidosis

Discuss the approach to weight loss in the patient with undiagnosed cancer

Timeline and amount lost Changes in diet/appetite/exercise Other constitutional symptoms : night sweats, fatigue, fever Complete history and review of systems

Risk factors: alcohol, smoking System-related history and symptoms:

o Lung: hx of smoking, environmental exposure, cough, sputum, hemoptysiso Colorectal: melena, hematochezia, changes in stool consistencyo Breast: lump, skin changes, dischargeo Prostate: urinary symptomso Brain: nausea, vomiting, new onset seizures, headache

Complete physical exam Lab workup (CBC, lytes, BUN, creatinine, LFT) CXR CT

Establish a differential diagnosis for splenomegaly and lymphadenopathyItalic = splenomegaly + lymphadenopathy

Splenomegaly1) INCREASED DEMAND FOR SPLENIC FCN

HEMATOLOGICALo Spherocytosiso Hemoglobinopathieso Nutritional anemiaso Elliptocytosis

Page 129: Objectives - Internal Medicine Define diastolic and systolic heart

INFECTIOUSo Bacterial (TB)o Viral (EBV, CMV, HIV)o Malariao Fungal (histoplasmosis)o LeishnmaniasisAUTOIMMUNEo RA (Felty’s)o SLEo Sarcoidosis

2) CONGESTIVEo Cirrhosiso Splenic vein obstructiono Portal vein obstructiono CHF

3) INFILTRATIVENon-malignanto Amyloidosiso Lysosomal storage diseases (Gaucher’s, Niemann-Pick)Malignanto Leukemia (CML)o Lymphomao Hodgkin’s diseaseo Myeloproliferative disorderso Metastatic tumour

LymphadenopathyREACTIVE

o Bacterial (TB, Lyme, Cat-scach disease)o Viral (EBV, CMV, HIV)o Parasitic (toxoplasmosis)o Fungal (histoplasmosis)

INFLAMMATORYo Autoimmunine (RA, SLE)o Drug hypersensitivityo Sarcoidosis, amyloidosis

NEOPLASTICo Lymphomao Lymphocytic leukemiao Metastatic cancero Hystocytosis X

Othero Serum sickness

Demonstrate physical examination of the lymphatic system

Inspection: Size Number

Page 130: Objectives - Internal Medicine Define diastolic and systolic heart

Presence of redness

Palpation: Describe characteristics of enlarged nodes and location Consistency: rubbery, hard, fluctuant, and matted; hard nodes are found in cancers that induce fibrosis

(scirrhous changes) and when previous inflammation has left fibrosis. Firm, rubbery nodes are found in lymphomas and chronic leukemia; nodes in acute leukemia tend to be softer.

Size: abnormal nodes are generally greater than 1 cm in diameter Location – localized lymphadenopathy suggests local causes and should prompt a search for pathology in

the area of node drainage. Tenderness – tenderness suggests recent, rapid enlargement that has put pain receptors in the capsule

under tension. This typically occurs with inflammatory processes, but can also result form hemorrhage into a node, immunologic stimulation, or malignancy.

Mobility – Normal nodes are freely moveable in the subcutaneous space. Abnormal nodes can become fixed to adjacent tissues (eg deep fascia) by invading cancers or inflammation in tissues surrounding the nodes.

Warmth-also palpate spleen and liver

Palpate using the pads of the fingers in a rotary motion (moving the skin over the underlying tissues). You may examine both sides of the head and neck simultaneously. Palpate the following nodes:

Occipital: Base of skull posteriorly Postauricular: Superficial to mastoid process Preauricular: anterior to ear Tonsillar: angle of mandible Submandibular: Midway between the angle

and the tip of the mandible Submental: Midline behind tip of mandible Anterior cervical: Along the anterior border of

the sternocleidomastoid muscle. Superficial cervical: superficial to SCM

muscle Post cervical: along anterior edge of trapezius

muscle Deep cervical: Deep to SCM muscle and often

inaccessible Supraclavicular: Deep in the angle between clavicle

and SCM muscle

Recognize the signs and symptoms of multiple myeloma- Pain in lower back, ribs or sternum: osteolytic lesions (70% of myeloma pts 1st present w pain)- Fatigue: ↑ myeloma cells can ↓ production of RBC in bone marrow → anemia- Recurrent infections: pneumonia, sinusitis, bladder or kidney infections, skin infections, shingles, cold sores: bc

crowding in marrow → ↓ immune system- Hypercalcemia: tiredness, thirst, frequent urination, nausea, muscle weakness, constipation, ↓appetite- WL- Weakness or numbness in legs- Renal probs: excessive light chains in urine or high levels of Ca in blood damages kidneys- M proteins (monoclonal) in blood and/or urine

Page 131: Objectives - Internal Medicine Define diastolic and systolic heart

Recognize, and implement treatment for, the following oncology emergencies: hypercalcemia, superior vena cava syndrome, cerebral metastasis, spinal cord compression

1. hypercalcemia : quick reminder, in cases of hypercalcemia the symptoms and signes include “Bones, stones, psychosis-based mones and abdominal groans”,

a. increase urinary Ca excretion with isotonic-saline (4-5L) + loop diurectics if hypovolemic (furosemide)

b. bisphosphonates are the treatment of choicec. IV pamidronate is most commonly used 2-4 days until full effect, but effect lasts 2-4 weeksd. Corticosteroids in hypervitaminosis and hematologic malignanciese. EDTA or IV phosphate (rare!)

2. superior vena cava syndrome (SVCS): obstruction of SVC by malignancies or mediastinal lymphadenopathy, get SOB + face/arm swelling: Tx = elevation of the head of the bed and supplemental oxygen, corticosteroids (dexamethasone 8-40 mg IV once initially, followed by 4-6 mg IV q6-8h, acts as anti-inflammatory and immunosupressant) and diuretics are often used to relieve laryngeal or cerebral edema, radiotherapy or chemotherapy in certain cases, surgery to bypass the SVC (palliation in select patients)

3. cerebral metastasis a. phenytoin for seizure prophylaxisb. dexamethasone to reduce edema…given with ranitidinec. chemo with small cell lung cancerd. radiation: whole brain radiation therapy or stereotactic radiosurgery if more than 3 lesions: can

help reduce symptoms in inoperable casese. surgery for single/solitary lesions, with radiation

4. spinal cord compression

Define pain and explain the various contributing factors in the cancer patient

Pain: physical, psychological, spiritual, social aspects Physical pain

o Somatic, visceral, neuropathico Other contributing factors: constipation, nausea, urinary retention, pruritis, dyspnea, anorexia-

cachexia, fatigue, bleeding, agitation, apathy, myoclonus Psychological Pain

o Depression, anxiety, deliriumo Grief and normal sadness: experienced in waves, retained capacity for pleasureo Depression: enduring, persistent, intense; associated with: helplessness, hopelessness,

worthlessness, guilt; *in context of cancer, don’t use somatic symptoms (fatigue, anorexia, sleep disturbance) to aid diagnosis as these can be associated with the cancer itself

o Delirium: fluctuating cognition (many causes) Spiritual Pain

o Spiritual issues with cancer usually centre on questions of meaning, value and relationshipso Dying patients want to be assured of their value; can help find hope in despair

Describe the means used to assess pain in the patient with cancer

Page 132: Objectives - Internal Medicine Define diastolic and systolic heart

Assessment of pain requires investigation of its history, location, character, provocative and palliative factors. Patients should be given a 10 division visual analogue scale on which to indicate the severity of the pain.

Ongoing pain is assessed by response to treatment. Pharmacologic intervention is most commonly used but antitumor therapy, neurostimulatory techniques, regional analgesia or neuroablative procedures can also be used in pain management.

WHO ladder – titration of oral analgesia

Non-opiod: Acetaminophen, aspirin, or NSAID Adjuvant: glucocorticoid, antidepressant Opioid for mild-moderate pain: codeine or hydrocodone (dose roughly equivalent to 10mg IV morphine)

or tramadol Opioid for moderate-severe pain: morphine 15-30mg q3-4h or controlled-release morphine 90-120mg bid

Demonstrate an understanding of the approach to treatment of cancer pain

Step-wise approach according to the level of painStep 1 - NSAIDS or ASA

Consider Tramadol if not responding to NSAIDSStep 2 Weak opioids:

- Codeine- Hydrocodone- OxycodoneConsider Tramadol if not responding to NSAIDS Alone or in combination with (1)

Step 3 - Morphine- Hydromorphone- Methadone- Transdermal fentanyl

Page 133: Objectives - Internal Medicine Define diastolic and systolic heart

Coanagesics to consider(explanation below)

- Systemic corticosteroids- Tricyclic antidepressants

o Nortriptyline- Anticonvulsants

o Gabapentin- Biphosphonates

o Zoledronic acid or Pamidronate- Radiolabeled agents

o Scrotinum-89 or samarium-153

- Oral route usually preferred - Pt with moderate to severe pain (2 and 3) must be reevaluated within 24-48 hr- Dose titration should be done aggressively to alleviate symptoms rapidly- Coanalgesics should be considered

o Systemic corticosteroids in pain from bone met, ↑ICP, nerve compression, infiltrationo Tricyclic antidepressants and anticonvulsants in neuropathic pain (less responsive to opioids)o Biphosphonates and Radiolabeled agents for pain from bone met

** Important to consider side effects of opioids: constipation, N/V, sedation/drowsiness, etc.

Describe the mechanisms of action and side effects of the main medications used to treat pain

Acetomininophen: inhibits cyclo-oxygenase and recent findings suggest that it is highly selective for COX-2. While it has analgesic and antipyretic properties comparable to those of ASA or other NSAIDS, its peripheral anti-inflammatory activity is usually limited by several factors, one of which is high level of peroxides present in inflammatory lesions. Because of its selectivity for COX-2, it does not significantly inhibit the production of the pro-clotting thromboxanes. The COX family of enzymes are responsible for the metabolism of arachidonic acid to prostaglandin H2, an unstable molecule that is, in turn, converted to numerous other pro-inflammatory compounds. Classical anti-inflammatories block this step. Tylenol reduces the oxidized form of the COX enzyme, preventing it from forming pro-inflammatory chemicals – therefore reduced prostaglandin E2 in CNS. Tylenol also modulates the endogenous cannabinoid system – it is metabolized to AM404, a compound with several actions; most important, it inhibits the uptake of

Page 134: Objectives - Internal Medicine Define diastolic and systolic heart

endogenous cannabinoid/vanilloid anandamide by neurons. Anandamide uptake would result in the activation of the main pain receptor in the body, TRPV1. Furthermore, AM404 inhibits sodium channels, as do the anesthetics lidocaine and procaine. Either of these actions by themselves has been shown to reduce pain, and are a possible mechanism for paracetamol (acetominiphen). However, it has been demonstrated that, after blocking cannabinoid receptors with synthetic antagonists, paracetamol’s analgesic effects are prevented, suggesting its pain-relieving action involves activation of the endogenous cannabinoid system.

o Acetominophen induced liver toxicity is due to the cellular damage caused by the reactive intermediate metabolite NAPQI. . Glutathione is produced in the liver and responsible for “mopping-up” NAPQI. Problems occur when the amount of NAPQI produced exceeds the “mopp-up” crews capacity. The recommended max dose of acetaminophen is 4 grams daily for acute pain and for long term administration consideration to decrease this to 2.6 gram daily. Max daily dose for children 90 mg/kg per day for children. (The FDA just voted to decrease the maximum daily dose in adults, but they haven’t decided what that dose should be). Liver toxicity is rare if dosage recommendations are followed. Toxicity is common if more than 10 gms (150 mg/kg in kids) is ingested. N-acetylcysteine(mucomyst) is the anti-dote.

o Risk of hepatotoxicity increases when patient surpasses maximum secure dose (automedication/polypharmacy) or if minor oxidative pathway becomes the major one (i.e. genetic polymorphism increasing activity of CYP 2E1 or inducers of CYP 2E1 such as Et-OH and smoking). If we increase the activity of CYP 2E1 – will have increased production of NAPQ1. Risk also increases with decrease in glutathione reserves (age and poor nutrition).

o At recommended doses, does NOT irritate the lining of the stomach, affect blood coagulation as much as NSAIDs, or affect kidney function. High doses increase risk of GI bleed.

Cyclo-oxygenase inhibitors: Naproxen, celecoxib, ketorolac, etc

o While the incidence of NSAID complication deaths is not high, due to the millions of patients on these drugs, the numbers are actually quite high. However chronic opioid administration is not as safe as we have been thinking. A major review published in the CMAJ December 2009 by Dhalla et al. reported that the death rate from medically

Page 135: Objectives - Internal Medicine Define diastolic and systolic heart

prescribed opioids is 30 patients per million per year. This extrapolates to 30 X 300 for the US (around 9,000) or more than half the rate of NSAIDs, yet one can be sure that many more patient are on NSAID making the relative risk for opioid even higher.

o The primary incentive for the drug industry to develop the COX-2 was to avoid the huge morbidity and mortality related to the upper GI problems associated with the chronic use of non-specific COX inhibitors. The fact that the COX-2 inhibitors are an even better solution to the anti-platelet problem is a coincidental winfall for the acute post-operative pain patient, where the market is relatively small compared to the arthritic patient population.

o Benefit of COX-2 inhibitor:1. Equal analgesic efficacy of non-selective COX inhibitors2. No effect on platelets!3. Better GI tolerance

o Contraindications to NSAIDs and celecoxib – patients with ASA triad: development of nasal polyps after taking ASA develop asthmatic crisis and angio-edema of upper respiratory tracts. (Samter’s syndrome). Other contraindications include gatro-duodenal ulcers or CHF.

Opiate: bind stereoscopically to opiod receptors localized in dorsal horn, brain, gut, smooth muscles sphincters.

o 3 main receptors: 1. Mu receptors: found primarily in brainstem and medial thalamus. Mu receptors are

responsible for supraspinal analgesia, respiratory depression, euphoria, sedation, decreased gastrointestinal motility, and physical dependence.mu 1 = analgesiamu 2 = sedation, vomiting, respiratory depression, pruritus, euphoria, anorexia, urinary retention, physical dependence

2. Kappa recetors: found in limbic system and other diencephalic areas – responsible for spinal analgesia, sedation, dyspnea, dependence, psychomimetic effects, and cause resp depression.

3. Delta receptors – largely in brain and effects not well studied.o Side effects – sedation, respiratory depression, impaired cognition miosis, N/V, loss of appetite,

pruritis, urinary retention, constipation, hypogonadism, immune modulation, hyperalgesia, toxicity (myoclonus), tolerance, dependence addiction.

Codeine: has exceptionally low affinity for opioid receptors and the analgesic effect of codeine is due to its conversion to morphine (O-demethylation, about 10% of codeine is O-demethylated to morphine). The conversion of codeine to morphine is effected by the cytochrome P450 enzyme CYP2D6. Well characterized genetic polymorphisms in this enzyme lead to the inability of convert codeine to morphine, thus making codeine in-effective as an analgesic for about 10% of the Caucasian population.

o Constipation***o Itching, nausea, vomiting, drowsiness, dry mouth, orthostatic hypotension, urinary retention

Tramadol: SNRI 325mg acetaminophen + 37.5 g de tramadol = tramacet; 1-2 comprimés toutes les 4 heures, au besoin. L’efficacité est limitée par la dose maximale d’acétaminophène. La force du tramadol repose dans sa faiblesse en tant qu’opioïde (faible affinité avec le récepteur de type µ). Donc moins de constipation et de N/V, aucune dépression respiratoire importante, aucun risque important d’abus.

o Tramadol ne s’oppose pas à l’action des antagonistes des récepteurs µ classiques comme la morphine, l’hydromorphone (Dilaudid®) ou le fentanyl

o On peut ajouter un autre agoniste µo Précautions – syndrome sérotoninergique (quand administré avec inhibiteurs de la MAO, ISRI,

IRSN, trazodone) TCAs : none of the TCAs carries an indication for pain management. Nevertheless, they remain

pharmacologic mainstay in a variety of chronic pain states. The mechanism of analgesic action is uncertain; it has been theorized that the analgesic properties are associated with their action as serotonin and norepinephrine reuptake inhibitors. There is also some evidence that TCAs potentiate the endogenous opioid system.

Page 136: Objectives - Internal Medicine Define diastolic and systolic heart

o Wide range of adverse effects: ANTICHOLINERGIC effects (hot as a hare, red as a beet, mad as a hatter, blind as a bat, dry as a bone), most of the troubling side effects involve the GI, cardio, and neuro systems. **widening of QRS on ECG!

Anticonvulsants: (phenytoin, carbamazepine, valproic acid, gabapentin, pregabalin=lyrica) o Mechanisms of actions are different for each and not fully understood. Phenytoin's activity

seems to be related to its membrane stabilizing action. Carbamazepine, which is pharmacologically related to the TCAs, prevents repeated discharges in neurons, an action that is consistent with its ability to relieve lancinating pain. Pregabalin is similar in structure to gabapentin. Valproic acid and clonazepam are believed to work through enhancement of GABA inhibitory systems. Beyond the consistent selection of carbamazepine for trigeminal neuralgia, physicians vary widely in their preferences for the use of these drugs. Oxcarbazepine provides similar analgesia to carbamazepine and has fewer adverse side effects, although it may cause more hyponatremia in susceptible patients. Many physicians will begin with gabapentin since it appears to be well tolerated, even at high doses, and plasma levels do not need to be followed as they do with phenytoin and carbamazepine.

o CBC and baseline LFTs should be obtained for patients starting on older anticonvulsants like dilantin, carbamazepine, and valproic acid.

Discuss end of life care goals with patients and their families- Discussion of likely course and possible complications- Advise pts and family when death imminent - Explain changes that may occur in pts body before and after death: irreg breathing, cool extremities,

confusion, purplish skin, somnolence in last hours, bronchial congestion, palatal relaxation (death rattle)- Advance directives: DNR- Relieving symptoms- Maximize function- Attention to spiritual issues- If pt to die at home, family should know who to call- Dying pt should be in area that is peaceful, quiet, comfortable, maintain physical contact

Describe the indications for desensitization therapyIt`s unclear what this question is specifically asking for, so I`m assuming it`s a pharm and allergy related thing…so here goes!

Desensitization: procedure which alters the immune response to an antibiotic and results in temporary tolerance, allowing the patient with IgE-mediated allergy to receive a course of medication safely, for example penicillin (desensitization renders mast cells unresponsive to penicillin).

Indications: patients who are proven – by positive skin testing or in vitro tests- OR are strongly suspected to have an IgE-mediated penicillin allergy AND for whom there are no acceptable alternate antibiotics

Discuss the side effects, indications for and titration of corticosteroids

Indications-Any inflammatory condition

Page 137: Objectives - Internal Medicine Define diastolic and systolic heart

-Ex. Rheumatoid arthritis, hepatitis, IBD (ulcerative colitis, Crohn’s), asthma, adrenal insufficiency, meningitis, multiple sclerosis, acute cerebral edema, anaphylaxis, eczema, rhinitis

Side Effects-increased susceptibility to infection (because they suppress inflammation)-may worsen: hypertension, heart failure, diabetes, peptic ulcers, osteoporosis-long-term use (especially at higher doses and if taken PO or IV):

-thinning of skin with bruising and stretch marks-high blood pressure-hyperglycemia/ diabetes-cataracts-fat deposits: chest, face upper back, stomach; puffiness in the face (moon face) -water and salt retention leading to swelling and edema-thinning of arms and legs-acne-poor wound healing-stunted growth in children-loss of calcium from bones (can lead to osteoporosis)-hunger-weight gain-mood swings / depression-adrenal suppression and adrenal crisis

*topical creams or inhaled corticosteroids act locally and therefore have a small side effect profile

TitrationShort-term treatment: -usually prescribed at a moderate dose and tapered over 1-2 week period; goal: rapid symptom of improvement

Long-term treatment: -high-dose steroids (1mg / kg, approx 60 mg / day given in divided doses) reserved for severe cases of inflammatory disease; steroids are tapered as soon as possible-always use the lowest dose possible to achieve an effect to minimize side effects

Tapering: -exogenous corticosteroids suppress production of endogenous cortisol (by adrenal glands), so when taken for more than 2 weeks, dose should be decreased gradually – so that adrenals have time to recover production of cortisol


Top Related